Computational Economic Analysis for Engineering and Industry [1 ed.] 0849374774, 9780849374777, 9781420007589

Recent global anxiety indicates that more focus needs to be directed at economic issues related to industry. Conventiona

753 12 5MB

English Pages 304 [306] Year 2007

Report DMCA / Copyright

DOWNLOAD FILE

Polecaj historie

Computational Economic Analysis for Engineering and Industry  [1 ed.]
 0849374774, 9780849374777, 9781420007589

Table of contents :
Front cover......Page 1
Acknowledgments......Page 6
Dedication......Page 8
Table of Contents......Page 10
Preface......Page 16
The Authors......Page 18
chapter one. Applied economic analysis......Page 20
chapter two. Cost concepts and techniques......Page 36
chapter three. Fundamentals of economic analysis......Page 52
chapter four. Economic methods for comparing investment alternatives......Page 80
chapter five. Asset replacement and retention analysis......Page 104
chapter six. Depreciation methods......Page 118
chapter seven. Break-even analysis......Page 132
chapter eight. Effects of inflation and taxes......Page 142
chapter nine. Advanced cash-flow analysis techniques......Page 154
chapter ten. Multiattribute investment analysis and selection......Page 170
chapter eleven. Budgeting and capital allocation......Page 198
chapter twelve. The ENGINEA software: a tool for economic evaluation......Page 208
chapter thirteen. Cost benchmarking case study*......Page 224
appendix A: Definitions and terms......Page 240
appendix B: Engineering conversion factors......Page 256
appendix C: Computational mathematical formulae......Page 264
appendix D: Units of measure......Page 270
appendix E: Interest factors and tables......Page 272
Index......Page 302
Back cover......Page 306

Citation preview

7477_C000.fm Page i Wednesday, May 2, 2007 3:59 PM

Computational Economic Analysis for Engineering and Industry

7477_C000.fm Page ii Wednesday, May 2, 2007 3:59 PM

Book series on industrial innovation Series editor Adedeji B. Badiru Department of Systems and Engineering Management Air Force Institute of Technology (AFIT) – Dayton, Ohio

Published titles: Handbook of Industrial and Systems Engineering Adedeji B. Badiru Techonomics: The Theory of Industrial Evolution H. Lee Martin

Forthcoming titles: Computational Economic Analysis for Engineering and Industry Adedeji B. Badiru & Olufemi A. Omitaomu Industrial Project Management: Concepts, Tools and Techniques Adedeji B. Badiru, Abi Badiru, & Ade Badiru Beyond Lean: Elements of a Successful Implementation Rupy (Rapinder) Sawhney Triple C Model of Project Management: Communication, Cooperation, Coordination Adedeji B. Badiru Process Optimization for Industrial Quality Improvement Ekepre Charles-Owaba & Adedeji B. Badiru Systems Thinking: Coping with 21st Century Problems John Turner Boardman & Brian J. Sauser

7477_C000.fm Page iii Wednesday, May 2, 2007 3:59 PM

Computational Economic Analysis for Engineering and Industry ADEDEJI B. BADIRU O L U F E M I A . O M I TA O M U

Boca Raton London New York

CRC Press is an imprint of the Taylor & Francis Group, an informa business

7477_C000.fm Page iv Wednesday, May 2, 2007 3:59 PM

CRC Press Taylor & Francis Group 6000 Broken Sound Parkway NW, Suite 300 Boca Raton, FL 33487-2742 © 2007 by Taylor & Francis Group, LLC CRC Press is an imprint of Taylor & Francis Group, an Informa business No claim to original U.S. Government works Printed in the United States of America on acid-free paper 10 9 8 7 6 5 4 3 2 1 International Standard Book Number-13: 978-0-8493-7477-7 (Hardcover) This book contains information obtained from authentic and highly regarded sources. Reprinted material is quoted with permission, and sources are indicated. A wide variety of references are listed. Reasonable efforts have been made to publish reliable data and information, but the author and the publisher cannot assume responsibility for the validity of all materials or for the consequences of their use. No part of this book may be reprinted, reproduced, transmitted, or utilized in any form by any electronic, mechanical, or other means, now known or hereafter invented, including photocopying, microfilming, and recording, or in any information storage or retrieval system, without written permission from the publishers. For permission to photocopy or use material electronically from this work, please access www. copyright.com (http://www.copyright.com/) or contact the Copyright Clearance Center, Inc. (CCC) 222 Rosewood Drive, Danvers, MA 01923, 978-750-8400. CCC is a not-for-profit organization that provides licenses and registration for a variety of users. For organizations that have been granted a photocopy license by the CCC, a separate system of payment has been arranged. Trademark Notice: Product or corporate names may be trademarks or registered trademarks, and are used only for identification and explanation without intent to infringe. Visit the Taylor & Francis Web site at http://www.taylorandfrancis.com and the CRC Press Web site at http://www.crcpress.com

7477_C000.fm Page v Wednesday, May 2, 2007 3:59 PM

Acknowledgments We thank all those who contributed to the completion and quality of this book. We thank our students at the University of Tennessee for their insights and ideas for what to include and what to delete. We especially thank Sirisha Saripali-Nukala and Bukola Ojemakinde for their help in composing many of the examples and problems included in the book. We also thank Jeanette Myers and Christine Tidwell for their manuscript preparation and administrative support services throughout the writing project. Many thanks go to Em Chitty Turner for her expert editorial refinement of the raw manuscript. We owe a debt of gratitude to Cindy Renee Carelli, senior acquisitions editor at CRC Press, and her colleagues for the excellent editorial and production support they provided for moving the book from idea to reality.

7477_C000.fm Page vi Wednesday, May 2, 2007 3:59 PM

7477_C000.fm Page vii Wednesday, May 2, 2007 3:59 PM

Dedication This book is dedicated to our families, from whom we directed our attention occasionally while we labored on this manuscript.

7477_C000.fm Page viii Wednesday, May 2, 2007 3:59 PM

7477_C000.fm Page ix Wednesday, May 2, 2007 3:59 PM

Table of Contents Preface.....................................................................................................................xv The Authors ........................................................................................................ xvii Chapter 1 Applied economic analysis............................................................1 1.1 Cost- and value-related definitions ............................................................1 1.2 Economics of worker assignment ...............................................................4 1.3 Economics of resource utilization ...............................................................6 1.4 Minimum annual revenue requirement (MARR) analysis .....................7 1.4.1 Flow-through method of MARR ..................................................10 1.4.2 Normalizing method of MARR .................................................... 11 Chapter 2 Cost concepts and techniques .....................................................17 2.1 Project cost estimation.................................................................................19 2.1.1 Optimistic and pessimistic cost estimates ..................................20 2.2 Cost monitoring ...........................................................................................21 2.3 Project balance technique ...........................................................................21 2.4 Cost and schedule control systems criteria .............................................22 2.5 Sources of capital .........................................................................................24 2.6 Commercial loans ........................................................................................25 2.7 Bonds and stocks .........................................................................................25 2.8 Interpersonal loans ......................................................................................25 2.9 Foreign investment ......................................................................................25 2.10 Investment banks .........................................................................................26 2.11 Mutual funds ................................................................................................26 2.12 Supporting resources...................................................................................26 2.13 Activity-based costing.................................................................................26 2.14 Cost, time, and productivity formulas.....................................................27 Practice problems for cost concepts and techniques ......................................32 Chapter 3 Fundamentals of economic analysis..........................................33 3.1 The economic analysis process..................................................................33 3.2 Simple and compound interest rates........................................................34 3.3 Investment life for multiple returns .........................................................36 3.4 Nominal and effective interest rates.........................................................36

7477_C000.fm Page x Wednesday, May 2, 2007 3:59 PM

3.5 Cash-flow patterns and equivalence ........................................................39 3.6 Compound amount factor..........................................................................41 3.7 Present worth factor ....................................................................................41 3.8 Uniform series present worth factor ........................................................42 3.9 Uniform series capital recovery factor .....................................................43 3.10 Uniform series compound amount factor ...............................................43 3.11 Uniform series sinking fund factor...........................................................44 3.12 Capitalized cost formula.............................................................................45 3.13 Permanent investments formula ...............................................................46 3.14 Arithmetic gradient series ..........................................................................47 3.15 Increasing geometric series cash flow ......................................................49 3.16 Decreasing geometric series cash flow.....................................................50 3.17 Internal rate of return..................................................................................52 3.18 Benefit/cost ratio .........................................................................................52 3.19 Simple payback period ...............................................................................54 3.20 Discounted payback period .......................................................................55 3.21 Fixed and variable interest rates ...............................................................57 Practice problems for fundamentals of economic analysis ...........................58 Chapter 4 Economic methods for comparing investment alternatives ....................................................................................................61 4.1 Net present value analysis .........................................................................61 4.2 Annual value analysis.................................................................................66 4.3 Internal rate of return analysis ..................................................................70 4.3.1 External rate of return analysis ....................................................74 4.4 Incremental analysis ....................................................................................75 4.5 Guidelines for comparison of alternatives ..............................................76 Practice problems: Comparing investment alternatives ................................81 Chapter 5 Asset replacement and retention analysis................................85 5.1 Considerations for replacement analysis .................................................85 5.2 Terms of replacement analysis...................................................................86 5.3 Economic service life (ESL) ........................................................................87 5.4 Replacement analysis computation ..........................................................88 Practice problems for replacement and retention analysis............................97 Chapter 6 Depreciation methods...................................................................99 6.1 Terminology of depreciation ......................................................................99 6.2 Depreciation methods ...............................................................................100 6.2.1 Straight-line (SL) method.............................................................101 6.2.2 Declining balance (DB) method..................................................102 6.2.3 Sums-of-years’ digits (SYD) method .........................................105 6.2.4 Modified accelerated cost recovery system (MACRS) method ............................................................................................106 Practice problems: Depreciation methods ......................................................109

7477_C000.fm Page xi Wednesday, May 2, 2007 3:59 PM

Chapter 7 Break-even analysis..................................................................... 113 7.1 Illustrative examples ................................................................................. 113 7.2 Profit ratio analysis.................................................................................... 117 Practice problems: Break-even analysis ..........................................................120 Chapter 8 Effects of inflation and taxes .....................................................123 8.1 Mild inflation ..............................................................................................126 8.2 Moderate inflation .....................................................................................126 8.3 Severe inflation...........................................................................................127 8.4 Hyperinflation ............................................................................................127 8.5 Foreign-exchange rates .............................................................................129 8.6 After-tax economic analysis .....................................................................130 8.7 Before-tax and after-tax cash flow ..........................................................132 8.8 Effects of taxes on capital gain ................................................................132 8.9 After-tax computations .............................................................................132 Practice problems: Effects of inflation and taxes...........................................133 Chapter 9 Advanced cash-flow analysis techniques ...............................135 9.1 Amortization of capitals ...........................................................................135 9.1.1 Equity break-even point...............................................................139 9.2 Introduction to tent cash-flow analysis..................................................142 9.3 Special application of AGS.......................................................................142 9.4 Design and analysis of tent cash-flow profiles .....................................143 9.5 Derivation of general tent equation........................................................148 Chapter 10 Multiattribute investment analysis and selection ..............151 10.1 The problem of investment selection .....................................................151 10.2 Utility models .............................................................................................151 10.2.1 Additive utility model..................................................................154 10.2.2 Multiplicative utility model ........................................................155 10.2.3 Fitting a utility function...............................................................155 10.2.4 Investment value model ..............................................................161 10.2.4.1 Capability........................................................................162 10.2.4.2 Suitability ........................................................................162 10.2.4.3 Performance....................................................................162 10.2.4.4 Productivity ....................................................................163 10.2.5 Polar plots.......................................................................................163 10.3 The analytic hierarchy process ................................................................170 10.4 Investment benchmarking........................................................................175 References.............................................................................................................177 Chapter 11 Budgeting and capital allocation............................................179 11.1 Top-down budgeting.................................................................................179 11.2 Bottom-up budgeting ................................................................................179 11.3 Mathematical formulation of capital allocation....................................180

7477_C000.fm Page xii Wednesday, May 2, 2007 3:59 PM

Chapter 12 The ENGINEA software: a tool for economic evaluation ....................................................................................................189 12.1 The ENGINEA software ...........................................................................189 12.1.1 Instructional design ....................................................................190 12.1.2 Cash-flow analysis ......................................................................190 12.1.3 Replacement analysis .................................................................191 12.1.4 Depreciation analysis .................................................................193 12.1.5 Interest calculator........................................................................194 12.1.6 Benefit/cost (B/C) ratio analysis .............................................195 12.1.7 Loan/mortgage analysis............................................................196 12.1.8 Rate-of-return (ROR) analysis...................................................197 12.2 The Excel spreadsheet functions .............................................................199 12.2.1 DB (declining balance) ...............................................................199 12.2.2 DDB (double declining balance) ..............................................199 12.2.3 FV (future value).........................................................................199 12.2.4 IPMT (interest payment)............................................................200 12.2.5 IRR (internal rate of return) ......................................................200 12.2.6 MIRR (modified internal rate of return) .................................200 12.2.7 NPER (number of periods) .......................................................200 12.2.8 NPV (net present value) ............................................................201 12.2.9 PMT (payments)..........................................................................201 12.2.10 PPMT (principal payment)........................................................201 12.2.11 PV (present value) ......................................................................202 12.2.12 RATE (interest rate) ....................................................................202 12.2.13 SLN (straight-line depreciation) ...............................................202 12.2.14 SYD (sum-of-year digits depreciation)....................................203 12.2.15 VDB (variable declining balance).............................................203 Reference ..............................................................................................................203 Chapter 13 Cost benchmarking case study ...............................................205 13.1 Abstract........................................................................................................205 13.2 Introduction ................................................................................................205 13.3 Basis of the methodology .........................................................................207 13.3.1 Data gathering and analysis .....................................................208 13.3.2 New project financing options .................................................208 13.3.3 Analysis of funding options .....................................................209 13.3.4 Cost comparison basis ...............................................................209 13.3.5 Analysis of the cost drivers.......................................................210 13.4 Evaluation baselines .................................................................................. 211 13.5 Data collection processes ..........................................................................213 13.6 Analysis of the factors...............................................................................214 13.6.1 F1: Impact of Davis Bacon Wages............................................214 13.6.1.1 Data findings................................................................214 13.6.2 F2: Impact of PLA.......................................................................214 13.6.2.1 Data findings................................................................214

7477_C000.fm Page xiii Wednesday, May 2, 2007 3:59 PM

13.6.3 F3: Additional ESH&Q requirements ........................................215 13.6.3.1 Data findings ..................................................................215 13.6.4 F4: Impact of excessive contract terms and conditions ..........216 13.6.5 F5: Impact of procurement process............................................216 13.7 Conclusions and recommendations........................................................218 13.7.1 Recommendations for reducing future project costs ..............219 13.7.1.1 Overall project recommendation ................................220 Reference ..............................................................................................................220 Appendix A

Definitions and terms.............................................................221

Appendix B

Engineering conversion factors ............................................237

Appendix C

Computational and mathematical formulae......................245

Appendix D

Units of measure .....................................................................251

Appendix E

Interest factors and tables......................................................253

Index .....................................................................................................................283

7477_C000.fm Page xiv Wednesday, May 2, 2007 3:59 PM

7477_C000.fm Page xv Wednesday, May 2, 2007 3:59 PM

Preface The bottom line, expressed in terms of cost and profits, is a major concern of many organizations. Even public institutions that have traditionally been nonchalant about costs and profits are now beginning to worry about economic justification. Formal economic analysis is the only reliable mechanism through which all the cost ramifications of a project, public or private, can be evaluated. Good economic analysis forms the basis for good decision making. Modern decisions should have sound economic basis. Decisions that don't have good economic foundations will eventually come back to haunt the decision maker. This is true for all types of decisions including technology decisions, engineering decisions, manufacturing decisions, and even social and political decisions. Decision making is the principal function of an engineer or manager. Over two-thirds of engineers will spend over two-thirds of their careers as managers and decision makers. Ordinary decision making involves choosing between alternatives. Economic decision making involves choosing between alternatives on the basis of monetary criteria. Economic analysis is a fundamental tool of the decision-making process. Traditionally, the application of economic analysis techniques to engineering problems has been referred to as engineering economy or engineering economic analysis. However, the increasing interest in economic analysis in all disciplines has necessitated a greater use of the more general term, economic analysis. Over the past few decades, the interest in engineering economy has increased dramatically. This is mainly due to a greater awareness and consciousness of the cost aspects of projects and systems. In the management of technology, it is common for top management people to consist largely of former engineers. It has, consequently, become important to train engineers in the cost aspects of managing engineering and manufacturing systems. Colleges and universities have been responding to this challenge by incorporating engineering economy into their curricula. All engineering and technology disciplines now embrace the study of engineering economy. This has, consequently, created a big and delineated market with a growing demand for engineering economy books. The rapid development of new engineering technologies and the pressure to optimize systems output while minimizing cost has continued to fuel the market momentum. Unfortunately, the pace of generating text materials for engineering economy has not kept up with the

7477_C000.fm Page xvi Wednesday, May 2, 2007 3:59 PM

demand. This is even more so in the very complex industrial environment, where integrated computational analyses are required. This book on Computational Economic Analysis for Engineering and Industry provides direct computational tools, techniques, models, and approaches for economic analysis with a specific focus on industrial and engineering processes. The book integrates mathematical models, optimization, computer analysis, and the managerial decision process. Industry is a very dynamic and expansive part of the national economy that is subject to high levels of investment, risks, and potential economic rewards. To justify the investments, special computational techniques must be used to address the various factors involved in an industrial process. A focused compilation of formulations, derivations, and analyses that have been found useful in various economic analysis applications will be of great help to industry professionals. This book responds to the changing economic environment of industry. Recent global economic anxiety indicates that more focus needs to be directed at economic issues related to industry. The book provides a high-level technical presentation of economic analysis of the unique aspects of industrial processes. Existing conventional techniques, while well proven, do not adequately embrace the integrated global factors affecting unique industries. Conceptual and philosophical publications are available on the worldwide developments in industry. But industryfocused computational tools are not readily available. This book fills that void. The contents of the book include new topics such as: New economic analysis models and techniques Tent-shaped cash flows Industrial economic analysis Project-based economic measures Profit ratio analysis Equity break-even point Utility based analysis Project-balance analysis Customized ENGINEA software tool The book will provide students, researchers, and practitioners with a comprehensive treatment of economic analysis, considering the specific needs of industry. Topics such as investment justification, break-even analysis, and replacement analysis are covered in an updated manner. The book provides a pragmatic alternative to conventional economic analysis books. Readers will find useful general information in the Appendixes, which contain engineering conversion factors and formulae. Adedeji B. Badiru Olufemi A. Omitaomu 2007

7477_C000.fm Page xvii Wednesday, May 2, 2007 3:59 PM

The Authors Adedeji “Deji” B. Badiru is the department head of systems and engineering management at the U.S. Air Force Institute of Technology (AFIT), Wright Patterson Air Force Base, Ohio. Previously head of industrial and information engineering at the University of Tennessee in Knoxville, he served as professor of industrial engineering and dean of University College at the University of Oklahoma. He is a registered professional engineer, a fellow of the Institute of Industrial Engineers, and a fellow of the Nigerian Academy of Engineering. He holds a B.S. degree in industrial engineering, an M.S. in mathematics, an M.S. in industrial engineering from Tennessee Technological University, and a Ph.D. in industrial engineering from the University of Central Florida. His areas of expertise and courses taught cover mathematical modeling, project management, systems analysis, and economic analysis. He is the author of several technical papers and books, and is the editor of the Handbook of Industrial and Systems Engineering. He is a member of several professional associations, including the Institute of Industrial Engineers (IIE), Society of Manufacturing Engineers (SME), Institute for Operations Research and Management Science (INFORMS), American Society for Engineering Education (ASEE), American Society for Engineering Management (ASEM), and the Project Management Institute (PMI). He has served as a consultant to several organizations around the world, including Russia, Mexico, Taiwan, Venezuela, South Africa, Nigeria, Ghana and South Korea. He has conducted customized training workshops for numerous organizations including Sony, AT&T, Seagate Technology, the U.S. Air Force, Oklahoma Gas and Electric, Oklahoma Asphalt Pavement Association, Hitachi, Nigeria National Petroleum Corporation, and ExxonMobil. He is the recipient of several honors including the IIE Outstanding Publication Award, University of Oklahoma Regents' Award for Superior Teaching, School of Industrial Engineering Outstanding Professor of the Year, Eugene L. Grant Award for Best Paper in Volume 38 of The Engineering Economist journal, University of Oklahoma College of Engineering Outstanding Professor of the Year, Ralph R. Teetor Educational Award from the Society of Automotive Engineers, Award of Excellence as chapter president from the Institute of Industrial Engineers, UPS Professional Excellence Award, Distinguished Alumni Award from Saint Finbarr's College, Lagos, Nigeria, and Distinguished

7477_C000.fm Page xviii Wednesday, May 2, 2007 3:59 PM

Alumni Award from the Department of Industrial and Systems Engineering, Tennessee Tech University. He holds a leadership certificate from the University of Tennessee Leadership Institute. Dr. Badiru has served as a technical project reviewer for the Third-World Network of Scientific Organizations, Italy. He has also served as a proposal review panelist for the National Science Foundation and National Research Council, and a curriculum reviewer for the American Council on Education. He is on the editorial and review boards of several technical journals and book publishers, and was an industrial development consultant to the United Nations Development Program. Olufemi Abayomi Omitaomu received a B.S. degree in mechanical engineering from Lagos State University, Nigeria in 1995, an M.S. degree in mechanical engineering from the University of Lagos, Nigeria in 1999, and a Ph.D. in industrial engineering from University of Tennessee, Knoxville, in 2006. He won several academic prizes during his undergraduate and graduate programs. After his B.S., he worked as a project engineer for Mobil Producing Nigeria between 1995 and 2001. During his Ph.D. program, he taught engineering economic analysis course for several semesters. Olufemi has published several journal and conference articles in international journals including The Engineering Economist. He has also published book chapters on economic analysis and data mining techniques. He jointly published two computer software programs including the ENGINEA, which is included in this book. Olufemi is a member of several professional bodies including the Institute of Industrial Engineers (IIE), Institute of Electrical and Electronic Engineers (IEEE), Institute for Operations Research and the Management Sciences (INFORMS), and the American Society of Mechanical Engineers (ASME). He was a board member of the Engineering Economy Division, Institute of Industrial Engineers (IIE) for 4 years. He is listed in the 2006 edition of Who's Who in America and 2007 edition of Who’s Who in Science and Engineering. He is currently a research associate in the Computational Sciences and Engineering Division at Oak Ridge National Laboratory, Tennessee. His research interests include computational economic analysis and online knowledge discovery and data mining. He is married to Remilekun Enitan, and they have two children, Oluwadamilola and Oluwatimilehin. December 2006

7477_book.fm Page 1 Tuesday, March 13, 2007 3:34 PM

chapter one

Applied economic analysis Industrial enterprises have fundamentally unique characteristics and require unique techniques of economic analysis. Thus, although the methodologies themselves may be standard, the specific factors or considerations may be industrially focused. Fortunately, most of the definitions used in general economic analyses are applicable to industrial economic analysis. This chapter presents computational definitions, techniques, and procedures for applied industrial economic analysis. As in the chapters that follow, a project basis is used for most of the presentations in this chapter.

1.1 Cost- and value-related definitions We need to define and clarify some basic terms often encountered in economic analysis. Some terms appear to be the same but are operationally different. For example, the term economics must be distinguished from the term economic analysis, and even more specifically from the term engineering economic analysis. Economics is the study of the allocation of the scarce assets of production for the purpose of satisfying some of the needs of a society. Economic analysis, in contrast, is an integrated analysis of the qualitative and quantitative factors that influence decisions related to economics. Finally, an engineering economic analysis is an analysis that focuses on the engineering aspects. Examples of the engineering aspects typically considered in an economic design process include the following: • • • • • •

Product conceptualization Research and development Design and implementation Prototyping and testing Production Transportation and delivery

Industrial economics is the study of the relationships between industries and markets with respect to prevailing market conditions, firm behavior,

1

7477_book.fm Page 2 Tuesday, March 13, 2007 3:34 PM

2

Computational Economic Analysis for Engineering and Industry

and economic performance. In a broader sense, the discipline of industrial economics focuses on a broad mix of industrial operations involving real-world competition, market scenarios, product conceptualization, process development, design, pricing, advertising, supply chain, delivery, investment strategies, and so on. Although this book may touch on some of the cost aspects, the full range of industrial economics is beyond its scope. Instead, the book focuses on the computational techniques that are applied to economic analysis in industrial settings. Earned value analysis is often used in industrial project economic analysis to convey the economic status of a project. Planned value (PV) refers to the portion of the approved cost that is planned to be spent during a specific period of the project. Actual cost (AC) is the total direct and indirect costs incurred in accomplishing work over a specific period of time. Earned value (EV) is defined as the budget for the work accomplished in a given period. Formulas relating to these measures are used to assess the overall economic performance of a project. Specific definitions are presented below: • Cost variance (CV) equals EV minus AC. The cost variance at the end of the project is the difference between the budget at completion (BAC) and the actual amount spent: CV = EV – AC A positive CV value indicates that costs are below budget. A negative CV value indicates a cost overrun. • Schedule variance (SV) equals EV minus PV. Schedule variance will ultimately equal zero when the project is completed, because all of the planned values will have been earned: SV = EV – PV A positive SV value indicates that a project is ahead of schedule. A negative SV value indicates that the project is behind schedule. • Cost performance index (CPI) equals the ratio of EV to AC. A CPI value less than 1.0 indicates a cost overrun of estimates. A CPI value greater than 1.0 indicates a cost underrun of estimates. CPI is the most commonly used cost-efficiency indicator: CPI = EV/AC A CPI greater than 1.0 indicates costs are below budget. A CPI less than 1.0 indicates costs are over budget. • Cumulative CPI (CPIC) is used to forecast project costs at completion. CPIC equals the sum of the periodic earned values (EVC) divided by the sum of the individual actual costs to date (ACC): CPIC = EV/ACC

7477_book.fm Page 3 Tuesday, March 13, 2007 3:34 PM

Chapter one:

Applied economic analysis

3

• A schedule performance index (SPI) is used, in addition to the schedule status, to predict completion date and is sometimes used in conjunction with CPI to generate project completion estimates. SPI equals the ratio of EV to PV: SPI = EV/PV An SPI greater than 1.0 indicates that a project is ahead of schedule. An SPI less than 1.0 indicates that a project is behind schedule. • BAC, ACC, and cumulative cost performance index (CPIC) are used to calculate the estimated total cost (ETC) and the estimated actual cost (EAC), where BAC is equal to the total PV at completion for a scheduled activity, work package, control account, or other WBS component: BAC = total cumulative PV at completion • ETC, based on atypical variances, is an approach that is often used when current variances are seen as atypical and the project management team expects that similar variances will not occur in the future. ETC equals BAC minus the cumulative earned value to date (EVC): ETC = (BAC – EVC) • Estimate at completion (EAC), also used interchangeably with estimated actual cost, is the expected total project cost upon completion with respect to the present time. There are alternate formulas for computing EAC depending on different scenarios. In one option, EAC equals ACC plus a new ETC that is provided by the project organization. This approach is most often used when past performance shows that the original estimating assumptions are no longer applicable due to a change in conditions: EAC = ACC + ETC • EAC using remaining budget. EAC equals ACC plus the budget required to complete the remaining work, which is BAC minus EV. This approach is most often used when current variances are seen as atypical and the project management team expects that similar variances will not occur in the future: EAC = ACC + BAC – EV • EAC using CPIC. EAC equals ACC to date plus the budget required to complete the remaining project work, which is BAC minus EV, modified by a performance factor (often CPIC). This approach is most often used when current variances are seen as typical of future variances: EAC = ACC + ((BAC – EV)/CPIC)

7477_book.fm Page 4 Tuesday, March 13, 2007 3:34 PM

4

Computational Economic Analysis for Engineering and Industry • Present Value (PV) is the current value of a given future cash-flow stream, discounted at a given rate. The formula for calculating a present value is: PV = FV/(1+r)(n)

1.2 Economics of worker assignment Operations research techniques are often used to enhance resource allocation decisions in engineering and industrial projects. One common resourceallocation methodology is the resource-assignment algorithm. This algorithm can be used to enhance the quality of resource-allocation decisions. Suppose there are n tasks that must be performed by n workers. The cost of worker i performing task j is cij. It is desirable to assign workers to tasks in a fashion that minimizes the cost of completing the tasks. This problem scenario is referred to as the assignment problem. The technique for finding the optimal solution to the problem is called the assignment method. The assignment method is an iterative procedure that arrives at the optimal solution by improving on a trial solution at each stage of the procedure. The assignment method can be used to achieve an optimal assignment of resources to specific tasks in an industrial project. Although the assignment method is cost-based, task duration can be incorporated into the modeling in terms of time–cost relationships. The objective is to minimize the total cost of the project. Thus, the formulation of the assignment problem is as shown below: Let xij = 1 if worker i is assigned to task j, j = 1, 2, …, n xij = 0 if worker i is not assigned to task j cij = cost of worker i performing task j z=

Minimize:

n

n

i=1

j=1

∑∑c x ij

ij

n

Subject to:

∑x

ij

= 1,

i = 1, 2 , … , n

ij

= 1,

j = 1, 2 , … , n

j=1 n

∑x i=1

x ij ≥ 0, i , j = 1, 2 , … , n The preceding formulation uses the non-negativity constraint, xij ≥ 0, instead of the integer constraint, xij = 0 or 1. However, the solution of the

7477_book.fm Page 5 Tuesday, March 13, 2007 3:34 PM

Chapter one:

Applied economic analysis

5

model will still be integer-valued. Hence, the assignment problem is a special case of the common transportation problem in operations research, with the number of sources (m) = number of targets (n), Si = 1 (supplies), and Di = 1 (demands). The basic requirements of an assignment problem are as follows: 1. There must be two or more tasks to be completed. 2. There must be two or more resources that can be assigned to the tasks. 3. The cost of using any of the resources to perform any of the tasks must be known. 4. Each resource is to be assigned to one and only one task. If the number of tasks to be performed is greater than the number of workers available, we will need to add dummy workers to balance the problem. Similarly, if the number of workers is greater than the number of tasks, we will need to add dummy tasks to balance the problem. If there is no problem of overlapping, a worker’s time may be split into segments so that the worker can be assigned more than one task. In this case, each segment of the worker’s time will be modeled as a separate resource in the assignment problem. Thus, the assignment problem can be extended to consider partial allocation of resource units to multiple tasks. The assignment model is solved by a method known as the Hungarian method, which is a simple iterative technique. Details of the assignment problem and its solution techniques can be found in operations-research texts. As an example, suppose five workers are to be assigned to five tasks on the basis of the cost matrix presented in Table 1.1. Task 3 is a machinecontrolled task with a fixed cost of $800 regardless of the specific worker to whom it is assigned. Using the assignment method, we obtain the optimal solution presented in Table 1.2, which indicates the following: x15 = 1, x23 = 1, x31 = 1, x44 = 1, and x52 = 1 Thus, the minimum total cost (TC) is given by TC = c15 + c23 + c31 + c44 + c52 = $(400 + 800 + 300 + 400 + 350) = $2,250

Table 1.1 Cost Matrix for Resource Assignment Problem Worker

Task 1

Task 2

Task 3

Task 4

Task 5

1 2 3 4 5

300 500 300 400 700

200 700 900 300 350

800 800 800 800 800

500 1250 1000 400 700

400 700 600 400 900

7477_book.fm Page 6 Tuesday, March 13, 2007 3:34 PM

6

Computational Economic Analysis for Engineering and Industry Table 1.2 Solution to Resource Assignment Problem Worker

Task 1

Task 2

Task 3

Task 4

Task 5

1 2 3 4 5

0 0 1 0 0

0 0 0 0 1

0 1 0 0 0

0 0 0 1 0

1 0 0 0 0

1.3 Economics of resource utilization In industrial operations that are subject to risk and uncertainty, probability information can be used to analyze resource utilization characteristics of the operations. Suppose the level of availability of a resource is probabilistic in nature. For simplicity, we will assume that the level of availability, X, is a continuous variable whose probability density function is defined by f(x). This is true for many resource types, ranging from funds and natural resources to raw materials. If we are interested in the probability that resource availability will be within a certain range of x1 and x2, then the required probability can be computed as follows:

(

)

P x1 ≤ X ≤ x2 =

∫ f ( x) dx x2

x1

Similarly, a probability density function can be defined for the utilization level of a particular resource. If we denote the utilization level by U and its probability density function by f(u), then we can calculate the probability that the utilization will exceed a certain level, u0, by the following expression:

(

)

P U ≥ u0 =



∫ f ( u) du u0

Suppose that a critical resource is leased for a large project. There is a graduated cost associated with using the resource at a certain percentage level U. The cost is specified as $10,000 per 10% increment in utilization level above 40%. A flat cost of $5,000 is charged for utilization levels below 40%. The utilization intervals and the associated costs are as follows: U < 40%, $5,000 40% ≤ U < 50%, $10,000 50% ≤ U < 60%, $20,000 60% ≤ U < 70%, $30,000 70% ≤ U < 80%, $40,000 80% ≤ U < 90%, $50,000 90% ≤ U < 100%, $60,000

7477_book.fm Page 7 Tuesday, March 13, 2007 3:34 PM

Chapter one:

Applied economic analysis

7

Thus, a utilization level of 50% will cost $20,000, whereas a level of 49.5% will cost $10,000. Suppose the utilization level is a normally distributed random variable with a mean of 60% and a variance of 16% squared and that we are interested in finding the expected cost of using this resource. The solution procedure involves finding the probability that the utilization level will fall within each of the specified ranges. The expected value formula will then be used to compute the expected cost as shown below: E C  =

∑ x P(x ) k

k

k

where xk represents the kth interval of utilization. The standard deviation of utilization is 4%. Thus, we have the following:  40 − 60  P U < 40 = P  z ≤ = P z ≤ −5 = 0.0 4  

(

)

(

)

( ) P ( 50 ≤ U < 60 ) = 0.4938 P ( 60 ≤ U < 70 ) = 0.4938 P ( 70 ≤ U < 80 ) = 0.0062 P ( 80 ≤ U < 90 ) = 0.0 E ( C ) = $5 , 000( 0.0 ) + $10 , 000( 0.0062 ) + $20 , 000( 0.4938 )

P 40 ≤ U < 50 = 0.0062

+ $30 , 000( 0.4938 ) + $40 , 000( 0.0062 ) + $50 , 000( 0.0 ) = $25 , 000 Based on these calculations, it can be expected that leasing this critical resource will cost $25,000 in the long run. A decision can be made as to whether to lease the resource, buy it, or substitute another resource for it, based on the information gained from this calculation.

1.4 Minimum annual revenue requirement (MARR) analysis Companies evaluating capital expenditures for proposed projects must weigh the expected benefits against the initial and expected costs over the life cycle of the project. One method that is often used is MARR analysis. Using the information about costs, interest payments, recurring expenditures,

7477_book.fm Page 8 Tuesday, March 13, 2007 3:34 PM

8

Computational Economic Analysis for Engineering and Industry

and other project-related financial obligations, the minimum annual revenue required by a project can be evaluated. We can compute the break-even point of the project. The break-even point is then used to determine the level of revenue that must be produced by the project in order for it to be profitable. The analysis can be done with either the flow-through method or the normalizing method. The factors to be included in MARR analysis are initial investment, book salvage value, tax salvage value, annual project costs, useful life for bookkeping purposes, book depreciation method, tax depreciation method, useful life for tax purposes, rate of return on equity, rate of return on debt, capital interest rate, debt ratio, and investment tax credit. Computational details on these factors are presented in subsequent chapters of this book. This section presents an overall illustrative example of how companies use MARR as a part of their investment decisions. The minimum annual revenue requirement for any year n may be determined by means of the net cash flows expected for that year: Net Cash Flow = Income – Taxes – Principal Amount Paid That is, Xn = (G – C – I) – I – P where Xn = annual revenue for year n G = gross income for year n C = expenses for year n I = interest payment for year n t = taxes for year n P = principal payment for year n Rewriting the equation yields G = Xn + C + I + t +P The preceding equation assumes that there are no capital requirements, salvage value considerations, or working capital changes in year n. For the minimum annual gross income, the cash flow, Xn, must satisfy the following relationship: Xn = De + fn where De = recovered portion of the equity capital fn = return on the unrecovered equity capital

7477_book.fm Page 9 Tuesday, March 13, 2007 3:34 PM

Chapter one:

Applied economic analysis

9

It is assumed that the total equity and debt capital recovered in a year are equal to the book depreciation, Db, and that the principal payments are a constant percentage of the book depreciation. That is, P = c(Db) where c is the debt ratio. The recovery of equity capital is, therefore, given by the following: De = (1 – c)Db The annual returns on equity, fn, and interest, I, are based on the unrecovered balance as follows: fn = (1 – c)ke (BVn–1) I = ckd (BVn–1) where c = debt ratio ke = required rate of return on equity kd = required rate of return on debt capital BVn–1 = book value at the beginning of year n Based on the preceding equations, the minimum annual gross income, or revenue requirement, for year n can be represented as R = Db + fn + C + I + t An expression for taxes, t, is given by t = (G – C – Dt – I) T where Dt = depreciation for tax purposes T = tax rate If the expression for R is substituted for G in the preceding equation, the following alternate expression for t can be obtained: t = [T/(1 – T)](Db + fn – Dt ) The calculated minimum annual revenue requirement can be used to evaluate the economic feasibility of a project. An example of a decision criterion that may be used for that purpose is presented as follows:

7477_book.fm Page 10 Tuesday, March 13, 2007 3:34 PM

10

Computational Economic Analysis for Engineering and Industry Decision Criterion: If expected gross incomes are greater than MARR, then the project is considered to be economically acceptable, and the project investment is considered to be potentially profitable. Economic acceptance should be differentiated from technical acceptance, however. If, of course, other alternatives being considered have similar results, a comparison based on the margin of difference (i.e., incremental analysis) between the expected gross incomes and minimum annual requirements must be made. There are two extensions to the basic analysis procedure presented in the preceding text. They are the flow-through method and the normalizing method.

1.4.1

Flow-through method of MARR

This extension of the basic revenue requirement analysis allocates credits and costs in the year that they occur. That is, there are no deferred taxes, and the investment tax credit is not amortized. Capitalized interest is taken as an expense in the first year. The resulting equation for calculating the minimum annual revenue requirements is: R = Db + fe + I + gP + C + t where the required return on equity is given by the following: fe = ke(1 – c)Kn–1 where ke = c= Kn–1 = Kn = g=

implied cost of common stock debt ratio chargeable investment for the preceding year Kn–1 – Db (with K0 = initial investment) capitalized interest rate

The capitalized interest rate is usually set by federal regulations. The debt interest is given by I = (c)kd Kn–1 where kd = after-tax cost of capital. The investment tax credit is calculated as follows: Ct = itP

7477_book.fm Page 11 Tuesday, March 13, 2007 3:34 PM

Chapter one:

Applied economic analysis

11

where i is the investment tax credit. Costs, C, are estimated totals that include such items as ad valorem taxes, insurance costs, operation costs, and maintenance costs. The taxes for the flow-through method are calculated as t=

1.4.2

T C ( fe + Db − Di ) − 1−T 1−T

Normalizing method of MARR

The normalizing method differs from the flow-through method in that deferred taxes are utilized. These deferred taxes are sometimes included as expenses in the early years of the project and then as credits in later years. This normalized treatment of the deferred taxes is often used by public utilities to minimize the potential risk of changes in tax rules that may occur before the end of the project but are unforeseen at the start of the project. Also, the interest paid on the initial investment cost is capitalized. That is, it is taken as a tax deduction in the first year of the project and then amortized over the life of the project to spread out the interest costs. The resulting minimum annual revenue requirement is expressed as R = Db + dt + Ct – At + I + fe + t + C where the depreciation schedules are based on the following capitalized investment cost: K = P + gP with P and g as previously defined. The deferred taxes, d, are the difference in taxes that result from using an accelerated depreciation model instead of a straight-line rate over the life of the project. That is, dt = (Dt – Ds)T where Dt = accelerated depreciation for tax purposes Ds = straight-line depreciation for tax purposes The amortized investment tax credit, At, is spread over the life of the project, n, and is calculated as follows:

At =

Ct n

7477_book.fm Page 12 Tuesday, March 13, 2007 3:34 PM

12

Computational Economic Analysis for Engineering and Industry

The debt interest is similar to the earlier equation for capitalized interest. However, the chargeable investment differs by taking into account the investment tax credit, deferred taxes, and the amortized investment tax credit. The resulting expressions are: I = kd(c)Kn–1 Kn = Kn–1 – Db – Ct – dt – At In this case, the expression for taxes, t, is given by the following:

t=

(

)

C T f e + Db + dt + Ct − At − Dt − gP − t 1− T 1− T

The differences between the procedures for calculating the minimum annual revenue requirements for the flow-through and the normalizing methods yield some interesting and important details. If the MARRs are converted to uniform annual amounts (leveled), a better comparison between the effects of the calculations for each method can be made. For example, the MARR data calculated by using each method are presented in Table 1.3. The annual MARR values are denoted by Rn, and the uniform annual amounts are denoted by Ru. The uniform amounts are found by calculating the present value for each early amount and then converting that total amount to equal yearly amounts over the same span of time. For a given investment, the flow-through method will produce a smaller leveled minimum annual revenue requirement. This is because the normalized data include an amortized investment tax credit as well as deferred taxes. The yearly data for the flow-through method should give values closer to the actual cash flows because credits and costs are assigned in the year in which they occur and not up front, as in the normalizing method. Table 1.3 Normalizing vs. Flow-through Revenue Analysis Year 1 2 3 4 5 6 7 8

Normalizing Rn Ru

Flow-through Rn Ru

7135 6433 5840 5297 4812 4380 4005 3685

5384 6089 5913 5739 5565 5390 5214 5040

5661 5661 5661 5661 5661 5661 5661 5661

5622 5622 5622 5622 5622 5622 5622 5622

7477_book.fm Page 13 Tuesday, March 13, 2007 3:34 PM

Chapter one:

Applied economic analysis

13

The normalizing method, however, provides for a faster recovery of the project investment. For this reason, this method is often used by public utility companies when establishing utility rates. The normalizing method also agrees better, in practice, with the required accounting procedures used by utility companies than does the flow-through method. Return on equity also differs between the two methods. For a given internal rate of return, the normalizing method will give a higher rate of return on equity than will the flow-through method. This difference occurs because of the inclusion of deferred taxes in the normalizing method. Illustrative Example Suppose we have the following data for a project. It is desired to perform a revenue requirement analysis using both the flow-through and the normalizing methods. Initial project cost = $100,000 Book salvage value = $10,000 Tax salvage value = $10,000 Book depreciation model = Straight line Tax depreciation model = Sum-of-years digits Life for book purposes = 10 years Life for tax purposes = 10 years Total costs per year = $4,000 Debt ratio = 40% Required return on equity = 20% Required return on debt = 10% Tax rate = 52% Capitalized interest = 0% Investment tax credit = 0% Table 1.4, Table 1.5, Table 1.6, and Table 1.7 show the differences between the normalizing and flow-through methods for the same set of data. The different treatments of capital investment produced by the investment tax credit can be seen in the tables as well as in Figure 1.1, Figure 1.2, Figure 1.3, and Figure 1.4. There is a big difference in the distribution of taxes because most of the taxes are paid early in the investment period with the normalizing method, but taxes are deferred with the flow-through method. The resulting MARR requirements are larger for the normalizing method early in the period. However, there is a more gradual decrease with the flow-through method. Therefore, the use of the flow-through method does not place as great a demand on the project to produce high revenues early in the project’s life cycle as does the normalizing method. Also, the normalizing method produces a lower rate of return on equity. This fact may be of particular interest to shareholders.

7477_book.fm Page 14 Tuesday, March 13, 2007 3:34 PM

14

Computational Economic Analysis for Engineering and Industry Table 1.4 Part One of MARR Analysis Year 1 2 3 4 5 6 7 8 9 10

Tax Depreciation Normalizing Flow-through 16,363.64 14,727.27 13,090.91 11,454.55 9,818.18 8,181.82 6,545.45 4,909.09 3,272.73 1,636.36

16,363.64 14,727.27 13,090.91 11,454.55 9,818.18 8,181.82 6,545.45 4,909.09 3,272.73 1,636.36

Deferred Taxes Normalizing Flow-through 3,829.09 2,978.18 2,127.27 1,276.36 425.45 425.45 1,276.36 2,127.27 2,978.18 3,829.09

None

Table 1.5 Part Two of MARR Analysis Year 1 2 3 4 5 6 7 8 9 10

Capitalized Investment Normalizing Flow-through 100,000.00 87,170.91 75,192.73 64,065.46 53,789.90 44,363.64 35,789.09 28,065.45 21,192.72 15,170.90 10,000.00

100,000.00 91,000.00 92,000.00 73,000.00 64,000.00 55,000.00 46,000.00 37,000.00 28,000.00 19,000.00 10,000.00

Taxes Normalizing Flow-through — 9,170.91 8,354.04 7,647.78 7,052.15 6,567.13 6,192.73 5,928.94 5,775.78 5,733.24 5,801.31

— 5,022.73 5,625.46 6,228.18 6,830.91 7,433.64 8,036.36 8,639.09 9,241.82 9,844.55 10,447.27

Table 1.6 Part Three of MARR Analysis Year 1 2 3 4 5 6 7 8 9 10

Return on Debt Normalizing Flow-through 4,000.00 3,486.84 3,007.71 2,562.62 2,151.56 1,774.55 1,431.56 1,122.62 847.71 606.84

4,000.00 3,640.00 3,280.00 2,920.00 2,560.00 2,200.00 1,840.00 1,480.00 1,120.00 760.00

Return of Equity Normalizing Flow-through 12,000.00 10,460.51 9,023.13 7,687.86 6,454.69 5,323.64 4,294.69 3,367.85 2,543.13 1,820.51

12,000.00 10,920.00 9,840.00 8,760.00 7,680.00 6,600.00 5,520.00 4,440.00 3,360.00 2,280.00

7477_book.fm Page 15 Tuesday, March 13, 2007 3:34 PM

Chapter one:

Applied economic analysis

15

Table 1.7 Part Four of MARR Analysis Year

Minimum Annual Revenues Normalizing Flow-through

1 2 3 4 5 6 7 8 9 10

42,000.00 38,279.56 34,805.89 31,578.98 28,598.84 25,865.45 23,378.84 21,138.98 19,145.89 17,399.56

34,022.73 33,185.45 32,348.18 31,510.91 30,673.64 29,836.36 2,899.09 28,161.82 27,324.55 26,487.27

Tax depreciation/deferred taxes

Part one of MARR analysis 20,000.00 15,000.00 10,000.00 5,000.00 0.00 1

2

3

4

5

6

7

9

8

–5,000.00

10

Year Tax depreciation: Normalizing Deferred taxes: Normalizing

Tax depreciation: Flow-through Deferred taxes: Flow-through

Figure 1.1 Plot of part one of MARR analysis.

Capitalized investment/taxes

Part two of MARR analysis 120,000 100,000 80,000 60,000 40,000 20,000 0 0

1

2

3

4

5 Year

6

7

8

9

Capitalized investment - normalizing Capitalized investment - flow-through Taxes - normalizing Taxes - flow-through

Figure 1.2 Plot of part two of MARR analysis.

10

7477_book.fm Page 16 Tuesday, March 13, 2007 3:34 PM

16

Computational Economic Analysis for Engineering and Industry

Return on debt/ return on equity

Part three of MARR analysis 15,000 10,000 5,000 0 1

2

3

4

5

6

7

8

9

10

Year Return on debt: Normalizing Return on equity: Normalizing

Return on debt: Flow-through Return on equity: Flow-through

Figure 1.3 Plot of part three of MARR analysis.

Minimum annual revenues

Part four of MARR analysis 50,000 40,000 30,000 20,000 10,000 0 1

2

3

4

5

6

7

8

9

10

Year Normalizing

Flow-through

Figure 1.4 Plot of part four of MARR analysis.

This chapter has presented selected general techniques of applied economic analysis for industrial projects. Subsequent chapters present specific topics within the general body of knowledge for industrial economic analysis, focusing primarily on computational techniques. Applied economic analysis techniques, as presented in this chapter, are useful in engineering and industrial projects for making crucial business decisions involving buy, make, rent, or lease options. The next chapter covers cost concepts relevant for computational economic analysis.

7477_book.fm Page 17 Tuesday, March 13, 2007 3:34 PM

chapter two

Cost concepts and techniques The term cost management refers, in a project environment, to the functions required to maintain effective financial control of the project throughout its life cycle. There are several cost concepts that influence the economic aspects of managing engineering and industrial projects. Within a given scope of analysis, there may be a combination of different types of cost aspects to consider. These cost aspects include the ones defined here: Actual cost of work performed: The cost actually incurred and recorded in accomplishing the work performed within a given period of time. Applied direct cost: The amounts recognized in the time period associated with the consumption of labor, material, and other direct resources, without regard to the date of commitment or the date of payment. These amounts are to be charged to work-in-process (WIP) when resources are actually consumed, material resources are withdrawn from inventory for use, or material resources are received and scheduled for use within 60 d. Budgeted cost for work performed: The sum of the budgets for completed work plus the appropriate portion of the budgets for level of effort and apportioned effort. Apportioned effort is that, which by itself is not readily divisible into short-span work packages, but is related in direct proportion to measured effort. Budgeted cost for work scheduled: The sum of budgets for all work packages and planning packages scheduled to be accomplished (including work in process) plus the amount of level of effort and apportioned effort scheduled to be accomplished within a given period of time. Direct cost: Cost that is directly associated with actual operations of a project. Typical sources of direct costs are direct material costs and direct labor costs. Direct costs are those that can be reasonably measured and allocated to a specific component of a project. Economies of scale: A reduction of the relative weight of the fixed cost in total cost by increasing output quantity. This helps to reduce the

17

7477_book.fm Page 18 Tuesday, March 13, 2007 3:34 PM

18

Computational Economic Analysis for Engineering and Industry final unit cost of a product. Economies of scale are often simply referred to as the savings due to mass production. Estimated cost at completion: The actual direct costs, plus indirect costs that can be allocated to the contract, plus estimated costs (direct and indirect) for authorized work remaining. First cost: The total initial investment required to initiate a project or the total initial cost of the equipment needed to start the project. Fixed cost: A cost incurred irrespective of the level of operation of a project. Fixed costs do not vary in proportion to the quantity of output. Examples of costs that make up the fixed cost of a project are administrative expenses, certain types of taxes, insurance cost, depreciation cost, and debt-servicing cost. These costs usually do not vary in proportion to quantity of output. Incremental cost: The additional cost of changing the production output from one level to another. Incremental costs are normally variable costs. Indirect cost: A cost that is indirectly associated with project operations. Indirect costs are those that are difficult to assign to specific components of a project. An example of an indirect cost is the cost of computer hardware and software needed to manage project operations. Indirect costs are usually calculated as a percentage of a component of direct costs. For example, the indirect costs in an organization may be computed as 10% of direct labor costs. Life-cycle cost: The sum of all costs, recurring and nonrecurring, associated with a project during its entire life cycle. Maintenance cost: A cost that occurs intermittently or periodically and is used for the purpose of keeping project equipment in good operating condition. Marginal cost: The additional cost of increasing production output by one additional unit. The marginal cost is equal to the slope of the total cost curve or line at the current operating level. Operating cost: A recurring cost needed to keep a project in operation during its life cycle. Operating costs may consist of such items as labor cost, material cost, and energy cost. Opportunity cost: The cost of forgoing the opportunity to invest in a venture that would have produced an economic advantage. Opportunity costs are usually incurred due to limited resources that make it impossible to take advantage of all investment opportunities. This is often defined as the cost of the best rejected opportunity. Opportunity costs can also be incurred due to a missed opportunity rather than due to an intentional rejection. In many cases, opportunity costs are hidden or implied because they typically relate to future events that cannot be accurately predicted. Overhead cost: A cost incurred for activities performed in support of the operations of a project. The activities that generate overhead costs support the project efforts rather than contribute directly to the

7477_book.fm Page 19 Tuesday, March 13, 2007 3:34 PM

Chapter two:

Cost concepts and techniques

19

project goal. The handling of overhead costs varies widely from company to company. Typical overhead items are electric power cost, insurance premiums, cost of security, and inventory-carrying cost. Standard cost: A cost that represents the normal or expected cost of a unit of the output of an operation. Standard costs are established in advance. They are developed as a composite of several component costs, such as direct labor cost per unit, material cost per unit, and allowable overhead charge per unit. Sunk cost: A cost that occurred in the past and cannot be recovered under the present analysis. Sunk costs should have no bearing on the prevailing economic analysis and project decisions. Ignoring sunk costs is always a difficult task for analysts. For example, if $950,000 was spent 4 years ago to buy a piece of equipment for a technologybased project, a decision on whether or not to replace the equipment now should not consider that initial cost. However, uncompromising analysts might find it difficult to ignore so much money. Similarly, an individual making a decision on selling a personal automobile would typically try to relate the asking price to what was paid for the automobile when it was acquired. This is wrong under the strict concept of sunk costs. Total cost: The sum of all the variable and fixed costs associated with a project. Variable cost: A cost that varies in direct proportion to the level of operation or quantity of output. For example, the costs of material and labor required to make an item are classified as variable costs because they vary with changes in level of output.

2.1 Project cost estimation Cost estimation and budgeting help establish a strategy for allocating resources in project planning and control. There are three major categories of cost estimation for budgeting based on the desired level of accuracy: orderof-magnitude estimates, preliminary cost estimates, and detailed cost estimates. Order-of-magnitude cost estimates are usually gross estimates based on the experience and judgment of the estimator. They are sometimes called “ballpark” figures. These estimates are typically made without a formal evaluation of the details involved in the project. Order-of-magnitude estimates can range, in terms of accuracy, from 50 to +50% of the actual cost. These estimates provide a quick way of getting cost information during the initial stages of a project. 50% (Actual Cost) ≤ Order-of-Magnitude Estimate ≤ 150% (Actual Cost) Preliminary cost estimates are also gross estimates but with a higher level of accuracy. In developing preliminary cost estimates, more attention is paid to some selected details of the project. An example of a preliminary

7477_book.fm Page 20 Tuesday, March 13, 2007 3:34 PM

20

Computational Economic Analysis for Engineering and Industry

cost estimate is the estimation of expected labor cost. Preliminary estimates are useful for evaluating project alternatives before final commitments are made. The level of accuracy associated with preliminary estimates can range from 20 to +20% of the actual cost. 80% (Actual Cost) ≤ Preliminary Estimate ≤ 120% (Actual Cost) Detailed cost estimates are developed after careful consideration is given to all the major details of a project. Considerable time is typically needed to obtain detailed cost estimates. Because of the amount of time and effort needed to develop detailed cost estimates, the estimates are usually developed after there is firm commitment that the project will happen. Detailed cost estimates are also important for evaluating actual cost performance during the project. The level of accuracy associated with detailed estimates normally ranges from 5 to +5% of the actual cost. 95% (Actual Cost) ≤ Detailed Cost ≤ 105% (Actual Cost) There are two basic approaches to generating cost estimates. The first one is a variant approach, in which cost estimates are based on variations of previous cost records. The other approach is the generative cost estimation, in which cost estimates are developed from scratch without taking previous cost records into consideration.

2.1.1

Optimistic and pessimistic cost estimates

Using an adaptation of the PERT formula, we can combine optimistic and pessimistic cost estimates. Let O = optimistic cost estimate M = most likely cost estimate P = pessimistic cost estimate Then, the estimated cost can be estimated as E C  =

O + 4M + P 6

The cost variance can be estimated as P−O V C  =    6 

2

7477_book.fm Page 21 Tuesday, March 13, 2007 3:34 PM

Project expenditure

Chapter two:

Cost concepts and techniques

Cost benefit

Cost overrun

21

Cost benefit

Planned cost Actual cost

Cost overrun

0 Cost review times

Figure 2.1 Evaluation of actual and projected cost.

2.2 Cost monitoring As a project progresses, costs can be monitored and evaluated to identify areas of unacceptable cost performance. Figure 2.1 shows a plot of cost vs. time for projected cost and actual cost. The plot permits quick identification when cost overruns occur in a project. Plots similar to those presented in the figure may be used to evaluate the cost, schedule, and time performances of a project. An approach similar to the profit ratio presented earlier may be used together with the plot to evaluate the overall cost performance of a project over a specified planning horizon. The following is a formula for the cost performance index (CPI):

CPI =

Area of cost benefit Area of cost beneffit + area of cost overrun

As in the case of the profit ratio, CPI may be used to evaluate the relative performances of several project alternatives or to evaluate the feasibility and acceptability of an individual alternative. In Figure 2.2, we present another cost monitoring tool: the cost control pie chart. This type of chart is used to track the percentage of cost going into a specific component of a project. Control limits can be included in the pie chart to identify out-of-control cost situations. The example in Figure 2.2 shows that 10% of total cost is tied up in supplies. The control limit is located at 12% of total cost. Hence, the supplies expenditure is within control (so far, at least).

2.3 Project balance technique One other approach to monitoring cost performance is the project balance technique, one that helps in assessing the economic state of a project at a

7477_book.fm Page 22 Tuesday, March 13, 2007 3:34 PM

22

Computational Economic Analysis for Engineering and Industry

Supplies

Control limit 1

10% 12%

30%

Control limit 2 (Panic situation)

Figure 2.2 Cost control pie chart.

desired point in time in the life cycle of the project. It calculates the net cash flow of a project up to a given point in time. The project balance is calculated as follows:

()

(

)

t

B i = St − P 1 + i + t

t

∑ PW

income

( i)

k

k=1

where B(i)t = project balance at time t at an interest rate of i% per period PW income (i)t = present worth of net income from the project up to time t P = initial cost of the project St = salvage value at time t The project balance at time t gives the net loss or net profit associated with the project up to that time.

2.4 Cost and schedule control systems criteria Contract management involves the process by which goods and services are acquired, utilized, monitored, and controlled in a project. Contract management addresses the contractual relationships from the initiation of a project to its completion (i.e., completion of services and/or hand-over of deliverables). Some of the important aspects of contract management include • Principles of contract law • Bidding process and evaluation

7477_book.fm Page 23 Tuesday, March 13, 2007 3:34 PM

Chapter two: • • • • • • •

Cost concepts and techniques

23

Contract and procurement strategies Selection of source and contractors Negotiation Worker safety considerations Product liability Uncertainty and risk management Conflict resolution

In 1967, the U.S. Department of Defense (DOD) introduced a set of 35 standards or criteria with which contractors must comply under cost or incentive contracts. The system of criteria is referred to as the Cost and Schedule Control Systems Criteria (C/SCSC). Many government agencies now require compliance with C/SCSC for major contracts. The system presents an integrated approach to cost and schedule management, and its purpose is to manage the government’s risk of cost overruns. Now widely recognized and used in major project environments, it is intended to facilitate greater uniformity and provide advance warning about impending schedule or cost overruns. The topics addressed by C/SCSC include cost estimating and forecasting, budgeting, cost control, cost reporting, earned value analysis, resource allocation and management, and schedule adjustments. The important link between all of these is the dynamism of the relationship between performance, time, and cost. Such a relationship is represented in Figure 2.3. This is essentially a multiobjective problem. Because performance, time, and cost objectives cannot be satisfied equally well, concessions or compromises need to be worked out in implementing C/SCSC.

Performance-cost relationship

f(p,c,t)

Cost (c)

Performance-time relationship

Performance (p)

Time (t)

Time-cost relationship

Figure 2.3 Performance–cost–time relationships for C/SCSC.

7477_book.fm Page 24 Tuesday, March 13, 2007 3:34 PM

24

Computational Economic Analysis for Engineering and Industry

Another dimension of the performance–time–cost relationship is represented by the U.S. Air Force’s R&M 2000 Standard, which addresses the reliability and maintainability of systems. R&M 2000 is intended to integrate reliability and maintainability into the performance, cost, and schedule management for government contracts. C/SCSC and R&M 2000 together constitute an effective guide for project design. To comply with C/SCSC, contractors must use standardized planning and control methods that are based on earned value. This refers to the actual dollar value of work performed at a given point in time, compared to the planned cost for the work. It is different from the conventional approach of measuring actual vs. planned, which is explicitly forbidden by C/SCSC. In the conventional approach, it is possible to misrepresent the actual content (or value) of the work accomplished. The work rate analysis technique presented in this book can be useful in overcoming the deficiencies of the conventional approach. C/SCSC is developed on a work content basis, using the following factors: • The actual cost of work performed (ACWP), which is determined on the basis of the data from the cost accounting and information systems • The budgeted cost of work scheduled (BCWS) or baseline cost determined by the costs of scheduled accomplishments • The budgeted cost of work performed (BCWP) or earned value, the actual work of effort completed as of a specific point in time The following equations can be used to calculate cost and schedule variances for a work package at any point in time. Cost variance = BCWP – ACWP Percent cost variance = (Cost variance/BCWP) · 100 Schedule variance = BCWP – BCWS Percent schedule variance = (Schedule variance/BCWS) · 100 ACWP and remaining funds = Target cost (TC) ACWP + cost to complete = Estimated cost at completion (EAC)

2.5 Sources of capital Financing a project means raising capital for the project. “Capital” is a resource consisting of funds available to execute a project, and it includes not only privately owned production facilities but also public investment. Public investments provide the infrastructure of the economy, such as roads, bridges, water supply, and so on. Other public capital that indirectly supports production and private enterprise includes schools, police stations, a central financial institution, and postal facilities. If the physical infrastructure of the economy is lacking, the incentive for private entrepreneurs to invest in production facilities is likely to be lacking

7477_book.fm Page 25 Tuesday, March 13, 2007 3:34 PM

Chapter two:

Cost concepts and techniques

25

also. Government and/or community leaders can create the atmosphere for free enterprise by constructing better roads, providing better public safety and better facilities, and by encouraging ventures that will assure adequate support services. As far as project investment is concerned, what can be achieved with project capital is very important. The avenues for raising capital funds include banks, government loans or grants, business partners, cash reserves, and other financial institutions. The key to the success of the free-enterprise system is the availability of capital funds and the availability of sources to invest the funds in ventures that yield products needed by the society. Some specific ways that funds can be made available for business investments are discussed in the following text.

2.6 Commercial loans Commercial loans are the most common sources of project capital. Banks should be encouraged to lend money to entrepreneurs, particularly those who are just starting new businesses. Government guarantees may be provided to make it easier for an enterprise to obtain the needed funds.

2.7 Bonds and stocks Bonds and stocks are also common sources of capital. National policies regarding the issuance of bonds and stocks can be developed to target specific project types in order to encourage entrepreneurs.

2.8 Interpersonal loans Interpersonal loans are an unofficial means of raising capital. In some cases, there may be individuals with enough personal funds to provide personal loans to aspiring entrepreneurs. But presently, there is no official mechanism that handles the supervision of interpersonal business loans. If a supervisory body existed at a national level, wealthy citizens might be less apprehensive about lending money to friends and relatives for business purposes. Individual wealthy citizens could, thus, become a strong source of business capital. Venture capitalists often operate as individuals or groups of individuals providing financing for entrepreneurial activities.

2.9 Foreign investment Foreign investment can be attracted for local enterprises through government incentives, which may take such forms as attractive zoning permits, foreign exchange permits, or tax breaks.

7477_book.fm Page 26 Tuesday, March 13, 2007 3:34 PM

26

Computational Economic Analysis for Engineering and Industry

2.10 Investment banks The operations of investment banks are often established to raise capital for specific projects. Investment banks buy securities from enterprises and resell them to other investors. Proceeds from these investments may serve as a source of business capital.

2.11 Mutual funds Mutual funds represent collective funds from a group of individuals. Such collective funds are often large enough to provide capital for business investments. Mutual funds may be established by individuals or under the sponsorship of a government agency. Encouragement and support should be provided for the group to spend the money for business investment purposes.

2.12 Supporting resources The government may establish a clearinghouse of potential goods and services that a new project can provide. New entrepreneurs interested in providing these goods and services should be encouraged to start relevant enterprises and given access to technical, financial, and information resources to facilitate starting production operations. A good example of this is “partnership” financing whereby cooperating entities come together to fund capital-intensive projects. The case study in Chapter 13 illustrates an example of federal, state, and commercial bank partnership to finance a large construction project.

2.13 Activity-based costing Activity-based costing (ABC) has emerged as an appealing costing technique in industry. The major motivation for adopting ABC is that it offers an improved method to achieve enhancements in operational and strategic decisions. ABC offers a mechanism to allocate costs in direct proportion to the activities that are actually performed. This is an improvement over the traditional way of generically allocating costs to departments. It also improves the conventional approaches to allocating overhead costs. The use of PERT/CPM, precedence diagramming, and critical resource diagramming can facilitate task decomposition to provide information for ABC. Some of the potential impacts of ABC on a production line include the following: • Identification and removal of unnecessary costs • Identification of the cost impact of adding specific attributes to a product • Indication of the incremental cost of improved quality • Identification of the value-added points in a production process • Inclusion of specific inventory-carrying costs

7477_book.fm Page 27 Tuesday, March 13, 2007 3:34 PM

Chapter two:

Cost concepts and techniques

27

• Provision of a basis for comparing production alternatives • The ability to assess “what-if” scenarios for specific tasks ABC is just one component of the overall activity-based management in an organization. Activity-based management involves a more global management approach to planning and control of organizational endeavors. This requires consideration for product planning, resource allocation, productivity management, quality control, training, line balancing, value analysis, and a host of other organizational responsibilities. Thus, although activity-based costing is important, one must not lose sight of the universality of the environment in which it is expected to operate. Frankly, there are some processes whose functions are so intermingled that separating them into specific activities may be difficult. Major considerations in the implementation of ABC include these: • Resources committed to developing activity-based information and cost • Duration and level of effort needed to achieve ABC objectives • Level of cost accuracy that can be achieved by ABC • Ability to track activities based on ABC requirements • Handling the volume of detailed information provided by ABC • Sensitivity of the ABC system to changes in activity configuration Income analysis can be enhanced by the ABC approach as shown in Table 2.1. Similarly, instead of allocating manufacturing overhead on the basis of direct labor costs, an activity-based costing analysis can be done, as illustrated in the example presented in Table 2.2. Table 2.3 shows a more comprehensive use of ABC to compare product lines. The specific ABC cost components shown in Table 2.3 can be further broken down if needed. A spreadsheet analysis would indicate the impact on net profit as specific cost elements are manipulated. Based on this analysis, it is seen that Product Line A is the most profitable. Product Line B comes in second even though it has the highest total line cost. Figure 2.4 presents a graphical comparison of the ABC cost elements for the product lines.

2.14 Cost, time, and productivity formulas This section presents a collection of common formulas useful for cost, time, and productivity analysis in manufacturing projects. Average time to perform a task: Based on learning-curve analysis, the average time required to perform a repetitive task is given by the following: tn = an − b

7477_book.fm Page 28 Tuesday, March 13, 2007 3:34 PM

28

Computational Economic Analysis for Engineering and Industry Table 2.1 Sample of Project Income Statement Statement of Income (In Thousands of Dollars, Except Per-Share Amounts) Two years ended 2006 2007 December 31 Net sales 1,918,265 1,515,861 Costs and expenses Cost of sales Research and development Marketing and distribution General and administrative

Operating income Consolidation of operations Interest and other income, net Income before taxes Provision for income taxes Net income

$1,057,849 72,511 470,573 110,062

$878,571 71,121 392,851 81,825

1,710,995

1,424,268

207,270 (36,981) 9,771 180,060 58,807

91,493 17,722 109,215 45,115

$121,253

$64,100

61,880

60,872

$1.96

$1.05

Equivalent shares Earnings per common share

where tn = cumulative average time resulting from performing the task n times t1 = time required to perform the task the first time k = learning factor for the task (usually known or assumed) The parameter k is a positive real constant whose magnitude is a function of the type of task being performed. A large value of k would cause the overall average time to drop quickly after just a few repetitions of the task. Thus, simple tasks tend to have large learning factors. Complex tasks tend to have smaller learning factors, thereby requiring several repetitions before significant reduction in time can be achieved. Calculating the learning factor: If the learning factor is not known, it may be estimated from time observations by the following formula: k=

log t1 − log tn log n

Calculating total time: Total time, Tn, to complete a task n times, if the learning factor and the initial time are known, is obtained by multiplying the average time by the number of times. Thus, 1− k Tn = t1n( )

7477_book.fm Page 29 Tuesday, March 13, 2007 3:34 PM

Chapter two:

Cost concepts and techniques

29

Table 2.2 Activity-Based Cost Details for Industrial Project Unit Cost ($) Labor Design engineer Carpenter Plumber Electrician IS engineer Labor subtotal Contractor Air conditioning Access flooring Fire suppression AT&T DEC reinstall DEC install VAX mover Transformer mover Contractor subtotal Materials Site preparation Hardware Software Other Materials subtotal Grand Total

Cost Basis

Days Worked

Day Day Day Day Day

34 27 2 82 81

$6,800 4,050 350 14,350 16,200 41,750

10,000 5,000 7,000 1,000 4,000 8,000 1,100 300

Fixed Fixed Fixed Fixed Fixed Fixed Fixed Fixed

5 5 5 50 2 7 7 7

10,000 5,000 7,000 1,000 4,000 8,000 1,100 300 36,400

2,500 31,900 42,290 10,860

Fixed Fixed Fixed Fixed

— — — —

2,500 31,900 42,290 10,860 87,550 165,700

200 150 175 175 200

Cost ($)

Determining time for nth performance of a task: The time required to perform a task the nth time if given by

(

)

xn = t1 1 − k n− k Determining limit of learning effect: The limit of learning effect indicates the number of times of performance of a task at which no further improvement is achieved. This is often called the improvement ratio and is represented as n≥

1 1 − r 1/k

Determining improvement target: It is sometimes desired to achieve a certain level of improvement after so many performances of a task, given a certain learning factor, k. Supposing that it takes so many trials, n1, to achieve a certain average time performance, y1, and that it is

7477_book.fm Page 30 Tuesday, March 13, 2007 3:34 PM

30

Computational Economic Analysis for Engineering and Industry Table 2.3 ABC Comparison of Product Lines ABC Cost Components

Product A

Product B

Product C

Product D

Direct labor Direct materials Supplies Engineering Material handling Quality assurance Inventory cost Marketing Equipment depreciation Utilities Taxes and insurance

27,000.00 37,250.00 1,500.00 7,200.00 4,000.00 5,200.00 13,300.00 3,000.00 2,700.00 950.00 3,500.00

37,000.00 52,600.00 1,300.00 8,100.00 4,200.00 6,000.00 17,500.00 2,700.00 3,900.00 700.00 4,500.00

12,500.00 31,000.00 3,200.00 18,500.00 5,000.00 9,800.00 10,250.00 4,000.00 6,100.00 2,300.00 2,700.00

16,000.00 35,000.00 2,500.00 17,250.00 5,200.00 8,300.00 11,200.00 4,300.00 6,750.00 2,800.00 3,000.00

105,600.00

138,500.00

105,350.00

112,300.00

Annual production Cost/unit Price/unit Net profit/unit

13,000.00 8.12 9.25 1.13

18,000.00 7.69 8.15 0.46

7,500.00 14.05 13.25 0.80

8,500.00 13.21 11.59 1.62

Total line revenue

120,250.00

146,700.00

99,375.00

98,515.00

14,650.00

8,200.00

5,975.00

13,785.00

Total line cost

Net line profit

60 50 40 30 20 10

Figure 2.4 Activity-based comparison of product lines.

Taxes & insurance

Utilities

Equipment dept.

Marketing

Inventory cost

Quality assurance

Material handling

Engineering

Supplies

Direct materials

Direct labor

0

7477_book.fm Page 31 Tuesday, March 13, 2007 3:34 PM

Chapter two:

Cost concepts and techniques

31

desired to calculate how many trials, n2, would be needed to achieve a given average time performance, y2, the following formula would be used:

(

n2 = n1 y11/k y2−1/k = n1 y2 /y1

)

−1/k

= n1r −1/k

where the parameter, r, is referred to as the time improvement factor. Calculation of number of machines needed to meet output: The number of machines needed to achieve a specified total output is calculated from the following formula:

N= where N= t= OT = u= H=

( )

1.67 t OT uH

number of machines processing time per unit (in minutes) total output per shift machine utilization ratio (in decimals) hours worked per day (8 times number of shifts)

Calculation of machine utilization: Machine idle times adversely affect utilization. The fraction of the time a machine is productively engaged is referred to as the utilization ratio and is calculated as follows: u=

ha hm

where ha = actual hours worked hm = maximum hours a machine could work The percent utilization is obtained as 100% times u. Calculation of output to allow for defects: To allow for a certain fraction of defects in total output, use the following formula to calculate starting output: Y= where Y = starting output X = target output f = fraction defective

X 1− f

7477_book.fm Page 32 Tuesday, March 13, 2007 3:34 PM

32

Computational Economic Analysis for Engineering and Industry Calculation of machine availability: The percent of time that a machine is available for productive work is calculated as follows: A=

o−u 100% o

(

)

Practice problems for cost concepts and techniques 2.1 If 70 standard labor hours were required in a manufacturing plant to construct some equipment for the first time, and experience with similar products indicates a learning curve of 90%, how many hours are required for the 150th unit? For the 350th unit? 2.2 The standard screening minutes per patient in a hospital based on a time study of the 42nd patient is 18.7. If the learning curve based on previous experience of patients with a similar illness is 88%, (a) What was the number of minutes required for the first patient? (b) What is the estimated number of minutes needed for the 100th patient? 2.3 If you were considering implementing ABC in an oil-and-gas or manufacturing or service facility, what are some examples of data or information that you would require before full implementation? 2.4 If the data requirements in Problem 2.3 were very costly to obtain, could you consider partially implementing ABC? How would you decide which overhead factors to allocate with ABC? 2.5 What are some advantages and disadvantages of using the cost performance index (CPI) for monitoring project costs? Develop a discounted CPI formula for monitoring project costs.

7477_book.fm Page 33 Tuesday, March 13, 2007 3:34 PM

chapter three

Fundamentals of economic analysis Capital, in the form of money, is one of the factors that sustains business projects or ventures in the enterprise of producing wealth. However, it is necessary to intelligently consider the implications of committing capital to a business over a period of time; the discipline of economic analysis helps us achieve that aim. The time value of money is an important factor in economic consideration of projects. This is particularly crucial for long-term projects that are subject to changes in several cost parameters. Both the timing and quantity of cash flow are important for project management. The evaluation of a project alternative requires consideration of the initial investment, depreciation, taxes, inflation, economic life of the project, salvage value, and cash flow. Capital can be classified into two categories: equity and debt. Equity capital is owned by individuals and invested with the hope of making profit, whereas debt capital is borrowed from lenders such as banks. In this chapter, we explain the nature of capital, interest, and the fundamental concepts underlying the relationship between capital investments and the terms of those investments. These fundamental concepts play a central role throughout the rest of this book.

3.1 The economic analysis process The process of economic analysis has the following basic components. The specific components will differ depending on the nature and prevailing circumstances of a project. This list is, however, representative of what an analyst might expect to encounter: 1. 2. 3. 4. 5. 6. 7.

Problem identification Problem definition Development of metrics and parameters Search for alternate solutions Selection of the preferred solutions Implementation of the selected solution Monitoring and sustaining the project 33

7477_book.fm Page 34 Tuesday, March 13, 2007 3:34 PM

34

Computational Economic Analysis for Engineering and Industry

3.2 Simple and compound interest rates Interest rates are used to quantify the time value of money, which may be defined as the value of capital committed to a project or business over a period of time. Interest paid is the cost on borrowed money, and interest earned is the benefit on saved or invested money. Interest rates can be calculated as simple rates or compound rates. Simple interest is interest paid only on the principal, whereas compound interest is interest paid on both the principal and the accrued interest. Let Fn = future value after n periods P = initial investment amount (the principal) I = interest rate per interest paid n = number of investment or loan periods r = nominal interest rate per year m = number of compounding periods per year ia = effective interest rate per compounding period i = effective interest rate per year The expressions for computing interest amounts based on simple interest and compound interest are as follows: Simple interest:

In = P(i)(n)

Compound interest:

In = iF(n- 1)

F(n- 1) is the future value at period (n – 1), which is the period immediately preceding the one for which the interest amount is being computed. The future values at time n are computed as: Simple interest:

Fn = P(1 + ni)

Compound interest:

Fn = P(1 + i)n

Fn is the accumulated value after n periods. At n = 0 and 1, the future values for both simple interest and compound interest are equal as shown in Table 3.1. In other words, simple and compound interest calculations yield the same results for Fn only when n = 0 or n = 1. Computationally, Fn = P(1 + in) ≡ Fn = P(1 + i)n ⇒ P(1 + in) = P(1 + i)n ⇒ (1 + in) = (1 + i)n ⇒ (1 + i)n − (1 + in) = 0 ⇒ n = 0 or n = 1

7477_book.fm Page 35 Tuesday, March 13, 2007 3:34 PM

Chapter three:

Fundamentals of economic analysis

35

Table 3.1 Comparison of Simple Interest and Compound Interest Computations Cash Flow

N (periods) 0

Simple Interest F0 = P + I 0

F P

1

0 1

F0 = P + I 0

= P + P(i)(0)

= P + P(i)(0)

=P

=P

F1 = P + I1

F

Compound Interest

F1 = F0 + I1

= P + P(i)(1)

= F0 + iF0

= P(1 + i)

= F0 (1 + i) = P(1 + i)

P

2

F2 = P + I 2

F 0 2

1

F2 = F1 + I 2

= P + P(i)(2)

= F1 + iF1

= P(1 + 2i)

F1 (1 + i) P(1 + i)(1 + i)

P

P(1 + i)2 . . .

. . .

n F

. . . Fn = P(1 + ni)

. . . Fn = P(1 + i)n

0 1 2

3……

….n

P

It can be seen that compound interest calculations represent a compound sum of a series of one-period simple interest calculations. Because simple interest is not widely used in economic analysis, it will not be further discussed, other than to point out differences between it and compound interest.

7477_book.fm Page 36 Tuesday, March 13, 2007 3:34 PM

36

Computational Economic Analysis for Engineering and Industry

3.3 Investment life for multiple returns A topic that is often of intense interest in many investment scenarios is how long it will take a given amount to reach a certain multiple of its initial level. The “Rule of 72” is one simple approach to calculating how long it will take an investment to double in value at a given interest rate per period. The Rule of 72 gives the following formula for estimating the doubling period: n=

72 i

where i is the interest rate expressed in percentage. Referring to the singlepayment compound amount factor, we can set the future amount equal to twice the present amount and then solve for n, the number of periods. That is, F = 2P. Thus,

(

2P = P 1 + i

)

n

Solving for n in the preceding equation yields an expression for calculating the exact number of periods required to double P: n=

()

ln 2

(

ln 1 + i

)

where i is the interest rate expressed in decimals. When exact computation is desired, the length of time it would take to accumulate m multiple of P is expressed in its general form as: n=

( )

ln m

(

ln 1 + i

)

where m is the desired multiple. For example, at an interest rate of 5% per year, the time it would take an amount P to double in value (m = 2) is 14.21 years. This, of course, assumes that the interest rate will remain constant throughout the planning horizon. Table 3.2 presents a tabulation of the values calculated from both approaches. Figure 3.1 shows a graphical comparison of the results from use of the Rule of 72 to use of the exact calculation.

3.4 Nominal and effective interest rates The compound interest rate, which we will refer to as simply “interest rate,” is used in economic analysis to account for the time value of money. Interest rates are usually expressed as a percentage, and the interest period (the time

7477_book.fm Page 37 Tuesday, March 13, 2007 3:34 PM

Chapter three:

Fundamentals of economic analysis

37

Table 3.2 Evaluation of the Rule of 72 i%

n (Rule of 72)

n (Exact value)

0.25 0.50 1.00 2.00 5.00 8.00 10.00 12.00 15.00 18.00 20.00 25.00 30.00

288.00 144.00 72.00 36.00 14.20 9.00 7.20 6.00 4.80 4.00 3.60 2.88 2.40

277.61 138.98 69.66 35.00 17.67 9.01 7.27 6.12 4.96 4.19 3.80 3.12 2.64

80

Number of periods (n)

70 60 50 40 Exact calculation

30

Rule of 72

20 10 0 0

5

10 15 Interest rate (i%)

20

25

Figure 3.1 Evaluation of investment life for double return.

unit of the rate) is usually a year. However, interest rates can also be computed more than once a year. Compound interest rates can be quoted as nominal interest rates or as effective interest rates. A nominal interest rate is the interest rate as quoted without considering the effect of any compounding. It is not the real interest rate used for economic analysis; however, it is usually the quoted interest rate because it is numerically smaller than the effective interest rate. It is equivalent to the annual percentage rate (APR), which is usually quoted for loan and creditcard purposes. The expression for calculating the nominal interest rate is as follows:

(

) (

r = interest rate per period × number of perio ods

)

7477_book.fm Page 38 Tuesday, March 13, 2007 3:34 PM

38

Computational Economic Analysis for Engineering and Industry

The format for expressing r is as follows: r% per time period t The effective interest rate can be expressed either per year or per compounding period. It is the effective interest rate per year that is used in engineering economic analysis calculations. It is the annual interest rate taking into consideration the effect of any compounding during the year. It accounts for both the nominal rate and the compounding frequency. Effective interest rate per year is given by:

(

i = 1+ i

)

m

−1 m

 r = 1+  − 1 m 

(

)

= F P , r m %, m − 1 Effective interest rate per compounding period is given by:

(

ia = 1 + i

)

1 m

−1=

r m

When compounding occurs more frequently, the compounding period becomes shorter; hence, we have the phenomenon of continuous compounding. This situation can be seen in the stock markets. The effective interest rate for continuous compounding is given by: i = er − 1 Note that the time period for i and r must be the same in using the preceding equations. Example 3.1 The nominal annual interest rate of an investment is 9%. What is the effective annual interest rate if the interest is 1. Payable, or compounded, quarterly? 2. Payable, or compounded, continuously?

7477_book.fm Page 39 Tuesday, March 13, 2007 3:34 PM

Chapter three:

Fundamentals of economic analysis

39

Solution 1. Using the effective interest rate formula, the effective annual interest rate compounded quarterly = 4

 0.09   1 + 4  − 1 = 9.31% 2. Using the equation for continuous rate, the effective annual interest rate compounded continuously = e 0.09 – 1 = 9.42% The slight difference between each of these values and the nominal interest rate of 9% becomes a big concern if the period of computation is in the double digits. The effective interest rate must always be used in all computations. Therefore, a correct identification of the nominal and effective interest rates is very important. See the following example. Example 3.2 Identify the following interest rate statements as either nominal or effective: 1. 2. 3. 4. 5.

14% per year 1% per month, compounded weekly Effective 15% per year, compounded monthly 1.5% per month, compounded monthly 20% per year, compounded semiannually Solution

1. This is an effective interest rate. This may also be written as 14% per year, compounded yearly. 2. This is a nominal interest rate because the rate of compounding is not equal to the rate of interest time period. 3. This is an effective interest for yearly rate. 4. This is an effective interest for monthly rate. A new rate should be computed for yearly computations. This may also be written as 1.5% per month. 5. This is a nominal interest rate because the rate of compounding and the rate of interest time period are not the same.

3.5 Cash-flow patterns and equivalence The basic reason for performing economic analysis is to provide information that helps in making choices between mutually exclusive projects competing

7477_book.fm Page 40 Tuesday, March 13, 2007 3:34 PM

40

Computational Economic Analysis for Engineering and Industry

for limited resources. The cost performance of each project will depend on the timing and levels of its expenditures. By using various techniques of computing cash-flow equivalence, we can reduce competing project cash flows to a common basis for comparison. The common basis depends, however, on the prevailing interest rate. Two cash flows that are equivalent at a given interest rate are not equivalent at a different interest rate. The basic techniques for converting cash flows from an interest rate at one point in time to the interest rate at another are presented in this section. A cash-flow diagram (CFD) is a graphical representation of revenues (cash inflows) and expenses (cash outflows). If several cash flows occur during the same time period, a net cash-flow diagram is used to represent the differences in cash flows. Cash-flow diagrams are based on several assumptions: • • • • • •

Interest rate is computed once in a time period. All cash flows occur at the end of the time period. All periods are of the same length. The interest rate and the number of periods are of the same length. Negative cash flows are drawn downward from the time line. Positive cash flows are drawn upward from the time line.

Cash-flow conversion involves the transfer of project funds from one point in time to another. There are several factors used in the conversion of cash flows. Let: P = cash flow value at the present time period. This usually occurs at time 0. F = cash flow value at some time in the future. A = a series of equal, consecutive, and end-of-period cash flow. This is also called annuity. G = a uniform arithmetic gradient increase in period-by-period cash flow. t = a measure of time period. It can be stated in years, months, or days. n = the total number of time periods, which can be in days, weeks, months, or years. i = interest rate time period expressed as a percentage. In many cases, the interest rate used in performing economic analysis is set equal to the minimum attractive rate of return (MARR) of the decision maker. MARR is also sometimes referred to as the hurdle rate, the required internal rate of return (IRR), the return on investment (ROI), or the discount rate. The value of MARR is chosen with the objective of maximizing the economic performance of a project.

7477_book.fm Page 41 Tuesday, March 13, 2007 3:34 PM

Chapter three:

Fundamentals of economic analysis

41 Future

Time periods 0 1

3 . . . .

2

n

Present

Figure 3.2 Single-payment compound amount cash flow.

3.6 Compound amount factor The procedure for the single-payment compound amount factor finds a future sum of money, F, that is equivalent to a present sum of money, P, at a specified interest rate, i, after n periods. This is calculated as: F = P(1 + i)n A graphical representation of the relationship between P and F is shown in Figure 3.2. Example 3.3 A sum of $5,000 is deposited in a project account and is left there to earn interest for 15 years. If the interest rate per year is 12%, the compound amount after 15 years can be calculated as follows: F = $5,000(1 + 0.12)15 = $27,367.85

3.7 Present worth factor The present worth factor computes P when F is given. It is obtained by solving for P in the equation for the compound amount factor. That is, P = F(1 + i)n Suppose it is estimated that $15,000 would be needed to complete the implementation of a project five years in the future; how much should be deposited in a special project fund now so that the fund would accrue to the required $15,000 exactly in five years? If the special project fund pays interest at 9.2% per year, the required deposit would be: P = $15,000(1 + 0.092)5 = $9,660.03

7477_book.fm Page 42 Tuesday, March 13, 2007 3:34 PM

42

Computational Economic Analysis for Engineering and Industry A

A

A

A

. . . . 0 1

2

. . . .

3

n

P

Figure 3.3 Uniform series cash flow.

3.8 Uniform series present worth factor The uniform series present worth factor is used to calculate the present worth equivalent, P, of a series of equal end-of-period amounts, A. Figure 3.3 shows the uniform series cash flow. The derivation of the formula uses the finite sum of the present worths of the individual amounts in the uniform series cash flow, as follows. Some formulas for series and summation operations are presented in the Appendixes at the end of the book. n

P=

∑ A (1 + i )

−t

t= 1

(

)

 1+ i n − 1  = A  i 1+ i n   

(

)

Example 3.4 Suppose that the sum of $12,000 must be withdrawn from an account to meet the annual operating expenses of a multiyear project. The project account pays interest at 7.5% per year compounded on an annual basis. If the project is expected to last 10 years, how much must be deposited in the project account now so that the operating expenses of $12,000 can be withdrawn at the end of every year for 10 years? The project fund is expected to be depleted to zero by the end of the last year of the project. The first withdrawal will be made 1 year after the project account is opened, and no additional deposits will be made in the account during the project life cycle. The required deposit is calculated to be:

(

)

 1 + 0.075 10 − 1   P = $12 , 000   0.075 1 + 0.075 10    = $82 , 368.92

(

)

7477_book.fm Page 43 Tuesday, March 13, 2007 3:34 PM

Chapter three:

Fundamentals of economic analysis

43

3.9 Uniform series capital recovery factor The capital recovery formula is used to calculate the uniform series of equal end-of-period payments, A, that are equivalent to a given present amount, P. This is the converse of the uniform series present amount factor. The equation for the uniform series capital recovery factor is obtained by solving for A in the uniform series present amount factor. That is,

(

)

 i 1+ i n   A = P  1+ i n − 1  

(

)

Example 3.5 Suppose a piece of equipment needed to launch a project must be purchased at a cost of $50,000. The entire cost is to be financed at 13.5% per year and repaid on a monthly installment schedule over 4 years. It is desired to calculate what the monthly loan payments will be. It is assumed that the first loan payment will be made exactly 1 month after the equipment is financed. If the interest rate of 13.5% per year is compounded monthly, then the interest rate per month will be 13.5%/12 = 1.125% per month. The number of interest periods over which the loan will be repaid is 4(12) = 48 months. Consequently, the monthly loan payments are calculated to be:

(

)

 0.01125 1 + 0.01123 48   A = $50 , 000   1 + 0.01125 48 − 1   

(

)

= $1353.82

3.10 Uniform series compound amount factor The series compound amount factor is used to calculate a single future amount that is equivalent to a uniform series of equal end-of-period payments. The cash flow is shown in Figure 3.4. Note that the future amount A

A

A

A

. . . . n

0 1

2

3

. . . . F

Figure 3.4 Uniform series compound amount cash flow.

7477_book.fm Page 44 Tuesday, March 13, 2007 3:34 PM

44

Computational Economic Analysis for Engineering and Industry

occurs at the same point in time as the last amount in the uniform series of payments. The factor is derived as follows: n

F=

∑ A (1 + i )

n −t

t= 1

(

)

 1+ i n − 1  = A   i   Example 3.6 If equal end-of-year deposits of $5,000 are made to a project fund paying 8% per year for 10 years, how much can be expected to be available for withdrawal from the account for capital expenditure immediately after the last deposit is made?

(

)

 1 + 0.08 10 − 1   F = $5 , 000    0.08   = $72 , 432.50

3.11 Uniform series sinking fund factor The sinking fund factor is used to calculate the uniform series of equal endof-period amounts, A, that are equivalent to a single future amount, F. This is the reverse of the uniform series compound amount factor. The formula for the sinking fund is obtained by solving for A in the formula for the uniform series compound amount factor. That is,   i  A= F  1+ i n − 1  

(

)

Example 3.7 How large are the end-of-year equal amounts that must be deposited into a project account so that a balance of $75,000 will be available for withdrawal immediately after the 12th annual deposit is made? The initial balance in the account is zero at the beginning of the first year. The account pays 10% interest per year. Using the formula for the sinking fund factor, the required annual deposits are:

7477_book.fm Page 45 Tuesday, March 13, 2007 3:34 PM

Chapter three:

Fundamentals of economic analysis

45

  0.10   A = $75 , 000  1 + 0.10 12 − 1   

(

)

= $3 , 507.25

3.12 Capitalized cost formula Capitalized cost refers to the present value of a single amount that is equivalent to a perpetual series of equal end-of-period payments. This is an extension of the series present worth factor with an infinitely large number of periods. This is shown graphically in Figure 3.5. Using the limit theorem from calculus as n approaches infinity, the series present worth factor reduces to the following formula for the capitalized cost:

(

)

 1+ i n − 1  P = lim A  n→∞  i 1+ i n   

(

)

  1 + i n − 1     = A lim  n n→∞     i 1+ i  

(

(

)

)

 1 = A   i There are several real-world investments that can be computed using this idea of capitalized cost formula. These include scholarship funds, maintenance of public buildings, and maintenance of roads and bridges, among others. Example 3.8 How much should be deposited in a general fund to service a recurring public service project to the tune of $6,500 per year forever if the fund yields A

A

A

A

A

A

A

A

.

.

.

.

.

.

.

.

0 1

2

3

4

C

Figure 3.5 Capitalized cost cash flow.

5

6

7

8

9

7477_book.fm Page 46 Tuesday, March 13, 2007 3:34 PM

46

Computational Economic Analysis for Engineering and Industry

an annual interest rate of 11%? Using the capitalized cost formula, the required one-time deposit to the general fund is as follows: P=

$6500 0.11

= $59 , 090.91 Example 3.9 A football stadium is expected to have an annual maintenance expense of $75,000. What amount must be deposited today in an account that pays a fixed interest rate of 12% per year to provide for this annual maintenance expense forever? The amount of money to be deposited today is 75, 000 = $625, 000 0.12 That is, if $625,000 is deposited today into this account, it will pay $75,000 annually forever. This is the power of the compounded interest rate.

3.13 Permanent investments formula This measure is the reverse of capitalized cost. It is the net annual value (NAV) of an alternative that has an infinitely long period. Public projects such as bridges, dams, irrigation systems, and railroads fall into this category. In addition, permanent and charitable organization endowments are evaluated using this approach. The NAV in the case of permanent investments is given by: A = Pi Example 3.10 If we deposit $25,000 in an account that pays a fixed interest rate of 10% today, what amount can be withdrawn each year to sponsor college scholarships forever? Solution Using the permanent investments formula, the required annual college scholarship worth is: A = $25, 000 × 0.10 = $2 , 500 The formulas presented in the preceding text represent the basic cashflow conversion factors. The factors are tabulated in Appendix E. Variations in the cash-flow profiles include situations where payments are made at the

7477_book.fm Page 47 Tuesday, March 13, 2007 3:34 PM

Chapter three:

Fundamentals of economic analysis

47

beginning of each period rather than at the end, and situations where a series of payments contains unequal amounts. Conversion formulas can be derived mathematically for those special cases by using the basic factors already presented. Conversion factors for some complicated cash-flow profiles are now discussed.

3.14 Arithmetic gradient series The gradient series cash flow involves an increase of a fixed amount in the cash flow at the end of each period. Thus, the amount at a given point in time is greater than the amount during the preceding period by a constant amount. This constant amount is denoted by G. Figure 3.6 shows the basic gradient series, in which the base amount at the end of the first period is zero. The size of the cash flow in the gradient series at the end of period t is calculated as follows:

(

)

At = t − 1 G,

t = 1, 2 , … , n

The total present value of the gradient series is calculated by using the present amount factor to convert each individual amount from time t to time 0 at an interest rate of i% per period and then by summing up the resulting present values. The finite summation reduces to a closed form, as follows: n

P=

∑ A (1 + i )

−t

t

t =1 n

=

∑ (t − 1)G (1 + i)

−t

t =1

n

∑ (t − 1)(1 + i)

=G

−t

t =1

(

) ( )  ( ) 

 1 + i n − 1 + ni = G n  i2 1 + i 

(n–1)G G

2G

0 1

2

3 . . . .

n

P

Figure 3.6 Arithmetic gradient cash flow with zero base amount.

7477_book.fm Page 48 Tuesday, March 13, 2007 3:34 PM

48

Computational Economic Analysis for Engineering and Industry Example 3.11

The cost of supplies for a 10-year project increases by $1,500 every year, starting at the end of the second year. There is no supplies cost at the end of the first year. If the interest rate is 8% per year, determine the present amount that must be set aside at time zero to take care of all the future supplies expenditures. We have G = 1,500, i = 0.08, and n = 10. Using the arithmetic gradient formula, we obtain the following:

(

(

)) ( ( )

 1 − 1 + 10 0.08 1 + 0.08 P = 1500  2  0.08 

(

= $1500 25.9768

)

−10

   

)

= $38, 965.20 In many cases, an arithmetic gradient starts with some base amount at the end of the first period and then increases by a constant amount thereafter. The nonzero base amount is denoted as A1. Figure 3.7 shows this type of cash flow. The calculation of the present amount for such cash flows requires breaking the cash flow into a uniform series cash flow of amount A1 and an arithmetic gradient cash flow with zero base amount. The uniform series present worth formula is used to calculate the present worth of the uniform series portion, and the basic gradient series formula is used to calculate the gradient portion. The overall present worth is then calculated as follows: P = Puniform series + Pgradient series

(

)

(

) ( )  ( ) 

 1+ i n − 1  1 + i n − 1 + ni +G = A1  n   i 1+ i n  i2 1 + i   

(

)

A1 + (n–1)G . . . . A1 . . . . 0 1

2

3

. . . .

n

P

Figure 3.7 Arithmetic gradient cash flow with nonzero base amount.

7477_book.fm Page 49 Tuesday, March 13, 2007 3:34 PM

Chapter three:

Fundamentals of economic analysis

49 An

A3

A2

A1

. . . . 0 1

2

3

. . . .

n

P

Figure 3.8 Increasing geometric series cash flow.

3.15 Increasing geometric series cash flow In an increasing geometric series cash flow, the amounts in the cash flow increase by a constant percentage from period to period. There is a positive base amount, A1, at the end of period one. Figure 3.8 shows an increasing geometric series. The amount at time t is denoted as

(

)

At = At−1 1 + j ,

t = 2 , 3, … , n

where j is the percentage increase in the cash flow from period to period. By doing a series of back substitutions, we can represent At in terms of A1 instead of in terms of At-1, as shown:

( ) = A (1 + j ) = A (1 + j ) (1 + j )

A2 = A1 1 + j A3

2

1



(

At = A1 1 + j

)

t −1

,

t = 1, 2 , 3, … , n

The formula for calculating the present worth of the increasing geometric series cash flow is derived by summing the present values of the individual cash-flow amounts. That is, n

P=

∑ A (1 + i )

−t

t

t =1 n

=

∑  A (1 + j)  (1 + i) t −1

1

t =1

−t

7477_book.fm Page 50 Tuesday, March 13, 2007 3:34 PM

50

Computational Economic Analysis for Engineering and Industry

A1 = 1+ j

(

n

)∑

 1+ j  1 + i 

(

)(

t =1

t

)

1− 1+ j n 1+ i − n  , = A1    i−j  

i≠j

If i = j, the preceding formula reduces to the limit as i approaches j (i → j), shown as follows: P=

nA1 , 1+ i

i=j

Example 3.12 Suppose that funding for a 5-year project is to increase by 6% every year, with an initial funding of $20,000 at the end of the first year. Determine how much must be deposited into a budget account at time zero in order to cover the anticipated funding levels if the budget account pays 10% interest per year. We have j = 6%, i = 10%, n = 5, A1 = $20,000. Therefore,

(

)(

 1 − 1 + 0.06 5 1 + 0.10 P = 20 , 000   0.10 − 0.06 

(

= $20 , 000 4.2267

)

5

   

)

= $84 , 533.60

3.16 Decreasing geometric series cash flow In a decreasing geometric series cash flow, the amounts in the cash flow decrease by a constant percentage from period to period. The cash flow starts at some positive base amount, A1, at the end of period one. Figure 3.9 shows a decreasing geometric series. The amount at time t is denoted as follows:

(

)

At = At−1 1 − j ,

t = 2 , 3, … , n

where j is the percentage decrease in the cash flow from period to period. As in the case of the increasing geometric series, we can represent At in terms of A1:

(

A2 = A1 1 − j

)

7477_book.fm Page 51 Tuesday, March 13, 2007 3:34 PM

Chapter three:

Fundamentals of economic analysis

51

A1 A2 A3 An . . . . 0 1

2

3

. . . .

n

P

Figure 3.9 Decreasing geometric series cash flow.

(

)

(

)

(

)(

A3 = A2 1 − j = A1 1 − j 1 − j

)

 At = A1 1 − j

t −1

t = 1, 2 , 3, … , n

,

The formula for calculating the present worth of the decreasing geometric series cash flow is derived by finite summation, as in the case of the increasing geometric series. The final formula is:

(

)(

1− 1− j n 1+ i P = A1   i+j 

)

−n

   

Example 3.13 A contract amount for a 3-year project is expected to decrease by 10% every year with an initial contract of $100,000 at the end of the first year. Determine how much must be available in a contract reservoir fund at time zero in order to cover the contract amounts. The fund pays 10% interest per year. Because j = 10%, i = 10%, n = 3, A1= $100,000, we should have

(

)(

 1 + 1 + 0.10 3 1 + 0.10 P = 100 , 000   0.10 + 0.10 

(

= $100 , 000 2.2615 = $226 , 150

)

)

−3

   

7477_book.fm Page 52 Tuesday, March 13, 2007 3:34 PM

52

Computational Economic Analysis for Engineering and Industry

3.17 Internal rate of return The IRR for a cash flow is defined as the interest rate that equates the future worth at time n or present worth at time 0 of the cash flow to zero. If we let i* denote the internal rate of return, we then have the following: n

FWt= n =

∑ ( ± A ) (1 + i * )

n −t

t

=0

t= 0 n

PWt=0 =

∑ ( ± A ) (1 + i * ) t

−t

=0

t= 0

where “+” is used in the summation for positive cash-flow amounts or receipts, and “–” is used for negative cash-flow amounts or disbursements. At denotes the cash-flow amount at time t, which may be a receipt (+) or a disbursement (–). The value of i* is referred to as the discounted cash flow rate of return, internal rate of return, or true rate of return. The procedure just discussed essentially calculates the net future worth or the net present worth of the cash flow. That is, Net Future Worth = Future Worth of Receipts – Future Worth of Disbursements NFW = FW(receipts) – FW(disbursements) Net Present Worth = Present Worth of Receipts – Present Worth of Disbursements NPW = PW(receipts) – PW(disbursements)

Setting the NPW or NFW equal to zero and solving for the unknown variable i determines the internal rate of return of the cash flow.

3.18 Benefit/cost ratio The computational methods described previously are mostly used for private projects because the objective of most private projects is to maximize profits. Public projects, on the other hand, are executed to provide services to the citizenry at no profit; therefore, they require a special method of analysis. Benefit/cost (B/C) ratio analysis is normally used for evaluating public projects. It has its origins in the Flood Act of 1936, which requires that, for a federally financed project to be justified, its benefits must, at minimum, equal its costs. B/C ratio is the systematic method of calculating the ratio of project benefits to project costs at a discounted rate. For over 60 years, the B/C ratio method has been the accepted procedure for making “go” or “no-go” decisions on independent and mutually exclusive projects in the public sector. The B/C ratio of a cash flow is the ratio of the present worth of benefits to the present worth of costs. This is defined as

7477_book.fm Page 53 Tuesday, March 13, 2007 3:34 PM

Chapter three:

Fundamentals of economic analysis n

∑ B (1 + i )

53

−t

t

B/C =

t= 0 n

∑ C (1 + i )

−t

t

t= 0

=

PWbenefits AWbenefits = PWcosts AWcosts

where Bt is the benefit (receipt) at time t and Ct is the cost (disbursement) at time t. If the B/C ratio is greater than one, then the investment is acceptable. If the ratio is less than one, the investment is not acceptable. A ratio of one indicates a break-even situation for the project. Example 3.14 Consider the following investment opportunity by Knox County. Initial cost = $600,000 Benefit per year at the end of Years 1 and 2 = $30,000 Benefit per year at the end of Years 3 to 30 = $50,000 If Knox County set the interest at 7% per year, would this be an economically feasible investment for the county? Solution For this problem, we will use both the PW and the AW approaches in order to show that both equations would give the same results. Using Present Worth

(

)

(

)(

PWbenefits = 30, 000 P/A, 7%, 2 + 50, 000 P/A, 7%, 28 P/F , 7%, 2

)

= $584, 267.16 PWcosts = $600, 000 Using Annual Worth

(

)

(

)(

)(

AWbenefits =  30, 000 P/A, 7%, 2 + 50, 000 P/A, 7%, 28 P/F , 7%, 2  A/P , 7%, 30 = $47 , 086.09

(

AWcosts = $600, 000 A/P , 7%, 30 = $48, 354.00

)

)

7477_book.fm Page 54 Tuesday, March 13, 2007 3:34 PM

54

Computational Economic Analysis for Engineering and Industry

B/C =

584, 267.16 47 , 086.09 = = 0.97 600, 000.00 48, 354.00

The B/C ratio indicates that the investment is not economically feasible because B/C < 1.0. However, one can see that the PW and AW methods both produced the same result.

3.19 Simple payback period The term payback period refers to the length of time it will take to recover an initial investment. The approach does not consider the impact of the time value of money. Consequently, it is not an accurate method of evaluating the worth of an investment. However, it is a simple technique that is used widely to perform a “quick-and-dirty” or superficial assessment of investment performance. The technique considers only the initial cost. Other costs that may occur after time zero are not included in the calculation. The payback period is defined as the smallest value of n(nmin) that satisfies the following expression: nmin

∑R ≥ C t

t =1

where Rt is the revenue at time t and C0 is the initial investment. The procedure calls for a simple addition of the revenues, period by period, until enough total has been accumulated to offset the initial investment. Example 3.15 An organization is considering installing a new computer system that will generate significant savings in material and labor requirements for order processing. The system has an initial cost of $50,000. It is expected to save the organization $20,000 a year. The system has an anticipated useful life of 5 years with a salvage value of $5,000. Determine how long it would take for the system to pay for itself from the savings it is expected to generate. Because the annual savings are uniform, we can calculate the payback period by simply dividing the initial cost by the annual savings. That is: nmin =

$50 , 000 $20 , 000

= 2.5 years Note that the salvage value of $5,000 is not included in the preceding calculation because the amount is not realized until the end of the useful life of the asset (i.e., after 5 years). In some cases, it may be desirable to consider

7477_book.fm Page 55 Tuesday, March 13, 2007 3:34 PM

Chapter three:

Fundamentals of economic analysis

55

the salvage value. In that case, the amount to be offset by the annual savings will be the net cost of the asset, represented here as nmin =

$50 , 000 − $5000 $20 , 000

= 2.25 years If there are tax liabilities associated with the annual savings, those liabilities must be deducted from the savings before calculating the payback period. The simple payback period does not take the time value of money into consideration; however, it is a concept readily understood by people unfamiliar with economic analysis.

3.20 Discounted payback period The discounted payback period is a payback analysis approach in which the revenues are reinvested at a certain interest rate. The payback period is determined when enough money has been accumulated at the given interest rate to offset the initial cost as well as other interim costs. In this case, the calculation is done with the aid of the following expression: nmin

∑ ( ) Rt 1 + i

t= 1

nmin − 1

nmin



∑C

t

t= 0

Example 3.16 A new solar-cell unit is to be installed in an office complex at an initial cost of $150,000. It is expected that the system will generate annual cost savings of $22,500 on the electricity bill. The solar cell unit will need to be overhauled every 5 years at a cost of $5,000 per overhaul. If the annual interest rate is 10%, find the discounted payback period for the solar-cell unit considering the time value of money. The costs of overhaul are to be considered in calculating the discounted payback period. Solution Using the single-payment compound amount factor for one period iteratively, the following solution is obtained: Time 1 2 3 4 5 6

Cumulative Savings $22,500 $22,500 $22,500 $22,500 $22,500 $22,500

+ + + + +

$22,500 (1.10)1 = $47,250 $47,250 (1.10)1 = $74,475 $74,475 (1.10)1 = $104,422.50 $104,422.50 (1.10)1 – $5000 = $132,364.75 $132,364.75 (1.10)1 = $168,101.23

7477_book.fm Page 56 Tuesday, March 13, 2007 3:34 PM

56

Computational Economic Analysis for Engineering and Industry

The initial investment is $150,000. By the end of period 6, we have accumulated $168,101.23, more than the initial cost. Interpolating between period 5 and period 6, we obtain: nmin = 5 +

150 , 000 − 132 , 364.75 6−5 168 , 101.25 − 132 , 364..75

(

)

= 5.49 That is, it will take 5.49 years, or 5 years and 6 months, to recover the initial investment. Example 3.17 For the following cash flows: 1. Calculate the simple payback period. 2. Calculate the discount payback period. A = 30,000

0

1

2

3

4

5

6

MARR = 15% 20,000

100,000

Solution In order to solve this problem, we make a payback table to facilitate easy computation: EOY

Net Cash Flow

Cumulative PV @ i = 0%

PV @ i > 0% (P/F,15%,n)

Cumulative PV @ i = 15%

n

= (n) 0 1 2 3 4 5 6

(a) $100,000 $30,000 $30,000 $30,000 $30,000 $30,000 $10,000

∑a

n

k

k=0

$100,000 $70,000 $40,000 $10,000 $20,000 $50,000 $60,000

= (b) $100,000 $26,088 $22,683 $19,725 $17,154 $14,916 $4,323

∑b

k

k=0

$100,000 $73,912 $51,229 $31,504 $14,350 $566 $4,889

7477_book.fm Page 57 Tuesday, March 13, 2007 3:34 PM

Chapter three:

Fundamentals of economic analysis

57

Based on the computations in the payback table, the simple payback period (SPP) is determined from the third column, and the discounted payback period (DPP) is determined from the fifth column. Therefore, the SPP is 4 years, the fourth year being when the cumulative PV becomes a positive value. The DPP is 5 years because the cumulative PV becomes a positive value in the fifth year. It must be noted that, in computing these values, the cash flows after SPP or DPP are not taken into consideration; therefore, both SPP and DPP techniques are usually used for initial screening of potential investment alternatives. They must never be used for final selection without considering other techniques such as Net Present Value and/or Internal Rate of Returns techniques.

3.21 Fixed and variable interest rates An interest rate may be fixed or may vary from period to period over the useful life of an investment, and companies, when evaluating investment alternatives, need also to consider the variable interest rates involved, especially in long-term investments. Some of the factors responsible for varying interest rates include changes in nominal and international economies, effects of inflation, and changes in market share. Loan rates, such as mortgage loan rates, may be adjusted from year to year based on the inflation index of the U.S. Consumer Price Index (CPI). If the variations in interest rate from period to period are not large, cash-flow calculations usually ignore their effects. However, the results of the computation will vary considerably if the variations in interest rates are large. In such cases, the varying interest rates should be considered in economic analysis, even though such consideration may become computationally involved. Example 3.18 Find the present worth (present value) of the following cash flows if for n < 5, i = 0.5%, and for n > 4, i = 0.25%. 400 350 300 250 200 100 100 100 100 100

0

PW

1

2

3

4

150

5

6

7

8

9

10

7477_book.fm Page 58 Tuesday, March 13, 2007 3:34 PM

58

Computational Economic Analysis for Engineering and Industry Solution

This is typical of several real-world cash flows. The computation must be carefully done in order to avoid errors. The present value is given as follows:

(

PV = A0 + A1− 4 P/A, 0.5%, 4

(

)

) (

)(

+  A5 P/A, 0.25%, 6 + G P/G, 0.25%, 6  P/F , 0.5%, 4

(

)

(

)

(

)(

= 100 + 100 3.950 + 150 5.948 + 50 14.826  0.9802

(

= 100 + 395 + 892.2 + 741.3  0.9802

)

)

)

= $2096.16 Therefore, the present value for these cash flows with two different interest rates is $2,096.16. The interest rate for n > 4 affects only cash flows in periods 5 to 10, whereas the interest rate for n < 5 affects cash flows in periods 1 to 10.

Practice problems for fundamentals of economic analysis 3.1 Calculate how long it would take your current personal or family savings to double at the current interest rate you are being offered by your bank. What will it take for you or your family to reduce the calculated period by one half? 3.2 If the nominal interest rate on a savings account is 0.25% payable, or compounded, quarterly, what is the effective annual interest rate? If $1,000 is deposited into this account quarterly, how much would be available in the account after 10 years? 3.3 A football player signed an $11 million, 10-year contract package with a football team he joined recently. Based on this contract, the football player will receive the following benefits: his yearly salary starts at $300,000 and goes up yearly to $400,000, $500,000, $600,000, $700,000, $1 million, $1.1 million, $1.2 million, $1.3 million and, finally, to $1.4 million in the 10th year. Besides his salary, a $2.5 million bonus is available that will pay him $500,000 immediately and $500,000 each year from the 11th year to the 14th year. Calculate the total present worth of the salaries and bonuses if the prevailing interest rate is 8% compounded per year. Did the footballer get the value of his contract? 3.4 How much must be deposited into a project account today if the project cost for each of the first 5 years is $12,000, and this amount increases by 10% per year for the following 10 years if the account pays 2.5% per year, compounded yearly?

7477_book.fm Page 59 Tuesday, March 13, 2007 3:34 PM

Chapter three:

Fundamentals of economic analysis

59

3.5 In order to maintain RAB University’s football stadium, the athletic department of the university needs annual maintenance costs of $60,000, annual insurance costs of $5,000, and annual utilities costs of $1,500. In addition, the department needs $100,000 worth of donations every 10 years for expansion projects. If the department opens a special account that pays 2.3% per year, how much must be deposited into this account now to pay for these annual costs forever? 3.6 A newly implemented technology in a manufacturing plant pays zero revenue in the first two years but $1,000 revenue in the third and fourth years; this amount increases by $500 annually for the following 5 years. If the company uses a MARR of 5.5% per year, what is the present value of these cash flows? What is the annual equivalent of the benefits over a 7-year period? 3.7 Repeat Problem 3.6 if the MARR for the company is 5.5% in the first 4 years and increases to 6% starting in year five. 3.8 A company borrowed $10,000 at 12% interest per year compounded yearly. The loan was repaid at $2,000 per year for the first 4 years and $2,200 in the fifth year. How much must be paid in the sixth year to pay off the loan? 3.9 A university alumnus wants to save $25,000 over 15 years so that he could start a scholarship for students in industrial engineering. To have this amount when it is needed, annual payments will be made into a savings account that earns 8% interest per year. What is the amount of each annual payment? 3.10 A small-scale industry thinks that it will produce 10,000 t of metal during the coming year. If the processes are controlled properly, then the metal is going to increase 5% per year thereafter for the next 6 years. Profit per ton of metal is $14 for years 1 to 7. If the industry earns 15% per year on this capital, what is the future equivalent of the industry’s cash flows at the EOY 7? 3.11 My grandmother just purchased a new house for $500,000. She made a down payment of 50% of the negotiated price and then makes a payment of $2,000.00 per month for 36 months. Furthermore, she thinks that she can resell the house for $600,000 due to the increasing real estate bubble at the end of 3 years. Draw a cash flow for this from my grandmother’s point of view. 3.12 A manufacturing unit in a facility is thinking of purchasing automatic lathes that could save $67,000 per year on labor and scrap. This lathe has an expected life of 5 years and no market value. If the company tells the manufacturing unit that it is expecting to see a 15% ROI per year, how much could be justified now for the purchase of this lathe? Explain it from the manufacturing unit’s perspective. 3.13 An ambitious student wants to start a restaurant, so he wants to buy a nice kitchen set consisting of two state-of-the-art grills, three stoves, two dishwashers, three refrigerators, two ovens, and three microwave ovens, along with some other stuff. So he plans on spending

7477_book.fm Page 60 Tuesday, March 13, 2007 3:34 PM

60

Computational Economic Analysis for Engineering and Industry

$112,000 on the equipment alone. He feels that these all would produce him a net income of $25,000 per year. If he does not change his mind and keeps this equipment for 4 years considering he doesn’t lose much in this business, what would be the resale value of all this equipment at the EOY 4 to justify his investment? A 15% annual return on investment is desired. 3.14 In the process of saving some money for my kid’s college, I plan to make six annual deposits of $4000 into a secret savings account that pays an interest of 4% compounded annually. Two years after making the last deposit, the interest rate increases to 7% compounded annually. Twelve years after the last deposit, the accumulated money is taken out for the first time to pay for his tuition. How much is withdrawn? 3.15 A newly employed engineer wants to find out how much he should invest at 12% nominal interest, compounded monthly, to provide an annuity of $25,000 (per year) for 6 years starting 12 years from now.

7477_book.fm Page 61 Tuesday, March 13, 2007 3:34 PM

chapter four

Economic methods for comparing investment alternatives The objective of performing an economic analysis is to make it easier to decide between or among potential investments that are competing for limited financial resources. Investment opportunities are either mutually exclusive or independent. For mutually exclusive investment opportunities, only one viable project can be selected; therefore, each project competes with the others. On the other hand, more than one project may be selected if the investment opportunities are independent; in such a case, the alternatives do not compete with one another in the evaluation. There are several methods for comparing investment alternatives: the present value analysis, the annual value analysis, the rate of return analysis, and the benefit/cost ratio analysis. In using these methods, the do-nothing (DN) option is a viable alternative that must be considered except when an investment alternative must in any case be selected. The selection of the DN alternative as the accepted project means, of course, that no new investment will be initiated. Three different analysis periods are usually used for evaluating alternatives: equal service lives for all alternatives, different service lives for all alternatives, and infinite service lives for all alternatives. The type of analysis period used may influence the method of evaluation chosen. In Chapter 3, we described each of the fundamental equations used in economic analysis; in this chapter, we present their applications in evaluating single and multiple investments.

4.1 Net present value analysis The net present value (NPV) analysis is the application of some of the engineering economic analysis factors in which the present amount is unknown. It is usually used for projects with equal service lives and can be used for evaluating one alternative, two or more mutually exclusive opportunities, or 61

7477_book.fm Page 62 Tuesday, March 13, 2007 3:34 PM

62

Computational Economic Analysis for Engineering and Industry

independent alternatives. The NPV analysis evaluates projects by converting all future cash flows into their present equivalent. The guidelines for using the present-value analysis for evaluating investment alternatives follow: • For one alternative: Calculate NPV at the minimum attractive rate of return (MARR). If NPV ≥ 0, the requested MARR is met or exceeded, and the alternative is economically viable. • For two or more alternatives: Calculate the NPV of each alternative at MARR. Select the alternative with the numerically largest NPV value. The numerically largest value indicates a lower NPV of cost cash flows (less negative) or a larger NPV of net cash flows (more positive). • For independent projects: Calculate the NPV of each alternative. Select all projects with a NPV ≥ 0 at the given MARR. Let P = cash flow value at the present time period. This usually occurs at time 0. F = cash flow value at some time in the future. A = a series of equal, consecutive, and end-of-period cash flow. This is also called annuity. G = a uniform arithmetic gradient increase in period-by-period cash flow. n = the total number of time periods, which can be in days, weeks, months, or years. i = interest rate per time period expressed as a percentage. Z = face, or par, value of a bond. C = redemption or disposal price (usually equal to Z). r = bond rate (nominal interest rate) per interest period. NCFt = estimated net cash flow for each year t. NCFA = estimated equal amount net cash flow for each year. np = discounted payback period. The general equation for the present value analysis is

(

) ( ) +  A ( P/A , i , n ) + G ( P/G, i , n )  + A ( P/A , g , i , n )

NPV = A0 + A P/A , i , n + F P/F , i , n

(4.1)

1

1

This equation reduces to a manageable size depending on the cash-flow profiles of the alternatives. For example, for bonds, the NPV analysis equation becomes:

(

) (

)

(

) (

NPV = A P/A , i, n + F P/F , i, n = rZ P/A , i, n + C P/F , i, n

)

(4.2)

Two extensions of NPV analysis are capitalized cost and discounted payback period.

7477_book.fm Page 63 Tuesday, March 13, 2007 3:34 PM

Chapter four:

Economic methods for comparing investment alternatives

63

Example 4.1 A $10,000 bond has a nominal interest rate of 10%, paid monthly, and matures 10 years after it is issued. After the original purchaser has had the bond for 5 years, she needs to sell it. The nominal rate for similar bonds is now 11.4%. What is the present worth measure of the bond to potential buyers? Solution From the question, C = Z = $10, 000 r=

10% = 0.83% per month 12

i=

11.4% = 0.95% per month 12

Therefore, using Equation 4.2:

(

)

(

NPV = 83.33 P/A, 0.95%, 60 + 10, 000 P/F , 0.95%, 60

)

= $9468.68 That is, the current selling price of the bond is $9,468.68; therefore, a potential buyer must not pay more than this amount for the bond. The ENGINEA software described in Chapter 12 can be used to obtain values for interest rates not tabulated in Appendix E. Example 4.2 A new manufacturing technology can be implemented using either of two alternative sources of energy: solar cells or a gas power plant. Solar cells will cost $12,600 to install and will have a useful life of 4 years with no salvage value. Annual costs for maintenance are expected to be $1,400. A gas power plant will cost $11,000 to install, with gas costs expected to be $800 per year. Because the technology will be replaced after 4 years, the salvage value of the gas plant is considered to be zero. At an interest rate of 17% per year, which alternative should be selected using the present value approach? Solution In order to solve this problem, Equation 4.1 reduces to NPV = A0 + A(P/A, i, n). Because one of these sources of energy must be selected, DN is not an alternative.

7477_book.fm Page 64 Tuesday, March 13, 2007 3:34 PM

64

Computational Economic Analysis for Engineering and Industry The NPV for solar cells is:

(

)

(

)

NPVSC = −12 , 600 − 1400 P/A, 17%, 4

= −$16, 440.50 The NPV for a gas power plant is: NPVGPP = −11, 000 − 800 P/A, 17%, 4

= −$13, 194.60 Based on the preceding computation, we must select the gas power plant option because it requires a smaller cost in the present time. Example 4.3 A local waste disposal company is considering two alternatives for a new truck. The most likely cash flows for the two alternatives are as follows:

Model

First Cost

Annual Operating Cost

Annual Income

Salvage Value

Useful Life

A B

$60,000 $80,000

$1,000 $2,000

$13,000 $15,000

$10,000 $12,000

10 years 10 years

Using the net present value approach and interest rate of 8% per year, which truck should the company buy? Solution The cash-flow diagram using the ENGINEA for each of the alternatives is shown below: Cash-flow diagram for Model A: 22000 12000 12000 12000 12000 12000 12000 12000 12000 12000

0

60000

1

2

3

4

5

6

7

8

9

10

7477_book.fm Page 65 Tuesday, March 13, 2007 3:34 PM

Chapter four:

Economic methods for comparing investment alternatives

65

Cash-flow diagram for Model B: 25000 13000 13000 13000 13000 13000 13000 13000 13000 13000

0

1

2

3

4

5

6

7

8

9

10

80000

The computation of the NPV follows:

( ) ( ) = −80, 000 + 13, 000 ( P/A, 8%, 9 ) + 25, 000 ( P/F , 8%, 10 ) = $12 , 789.38

NPVA = −60, 000 + 12 , 000 P/A, 8%, 9 + 22 , 000 P/F , 8%, 10 = $25, 152.91 NPVB

Therefore, Model A should be selected because it has a higher NPV. Example 4.4 RAB General Hospital is evaluating three contractors to manage its emergency ambulance service for 5 years. Using the present value approach, which of these contractors should be selected if the interest rate is 10% per year? Contractor A

Contractor B

Contractor C

$15,000 1,600 8,000 3,000

$20,000 1,000 10,000 4,500

$25,000 900 13,000 6,000

Initial cost Maintenance and operating costs Annual benefits Salvage value

The cash-flow diagrams for Contractors A, B, and C, respectively, are as follows: 9400

0

15000

1

2

3

4

18100

13500

6400 6400 6400 6400

12100 12100 12100 12100

9000 9000 9000 9000

5

0

20000

1

2

3

4

5

0

25000

1

2

3

4

5

7477_book.fm Page 66 Tuesday, March 13, 2007 3:34 PM

66

Computational Economic Analysis for Engineering and Industry

The computation of the NPV follows:

( ) ( ) = −20, 000 + 9, 000 ( P/A, 10%, 4 ) + 13, 500 ( P/F , 10%, 5 ) = $16, 911.23 = −25, 000 + 12 , 100 ( P/A, 10%, 4 ) + 18, 100 ( P/F , 10%, 5 ) = $24, 594.05

NPVA = −15, 000 + 6, 400 P/A, 10%, 4 + 9, 400 P/F , 10%, 5 = $11, 123.80 NPVB NPVC

Therefore, Contractor C is the best option for RAB General Hospital.

4.2 Annual value analysis The net annual value (NAV) method of evaluating investment opportunities is the most readily used of all the measures because people easily understand what it means. This method is mostly used for projects with unequal service life because it requires the computation of the equivalent amount of the initial investment and the future amounts for only one project life cycle. NAV analysis converts all future and present cash flows into equal end-of-period amounts. For mutually exclusive alternatives, calculate NAV at MARR and select the viable alternatives based on the following guidelines: • One alternative: Select an alternative with NAV ≥ 0 because MARR is met or exceeded. • Two or more alternatives: Choose alternative with the lowest-cost or the highest-revenue NAV value. Let CR = capital recovery component A = annual amount component of other cash flows P = initial investment (first cost) of all assets S = estimated salvage value of the assets at the end of their useful life i = investment interest rate The annual value amount for an alternative consists of two components: capital recovery for the initial investment P at a stated interest rate (usually at MARR) and the equivalent annual amount A. Therefore, the general equation for the annual value analysis is: NAV = −CR − A

(

) (

)

= −  P A/P , i, n − S A/F , i, n  − A

(4.3)

NAV is especially useful in areas such as asset replacement and retention, break-even studies and make-or-buy decisions, as well as all studies relating to profit measure. It should be noted that expenditures of money increase

7477_book.fm Page 67 Tuesday, March 13, 2007 3:34 PM

Chapter four:

Economic methods for comparing investment alternatives

67

NAV, whereas receipts of money such as selling an asset for its salvage value decrease it. The NAV method assumes the following: • The service provided will be needed forever because it computes the annual value per cycle. • The alternatives will be repeated exactly the same in succeeding life cycles. This is especially important when the service life extends several years into the future. • All cash flows will change by the same amount as the inflation or deflation rate. The validity of these assumptions is based on the accuracy of the cash-flow estimates. If the cash-flow estimates are very accurate, then the assumptions based on this method will be valid and should minimize the degree of uncertainty surrounding the final decision. Example 4.5 Repeat Example 4.2 using the annual value approach. Solution In order to solve this problem, we use Equation 4.3:

(

) (

)

NAV = −  P A/P , i , n − S A/F , i , n  − A



The NAV for solar cells:

(

) (

)

) (

)

NAVSC = − 12 , 600 A/P , 17%, 4 − 0 A/F , 17%, 4  − 1400



= −5992.70 The NAV for the gas power plant:

(

NAVGPP = − 11, 000 A/P , 17%, 4 − 0 A/F , 17%, 4  − 800



= −4809.50 Again, we should select the gas power plant option because it requires less cost on an annual basis. Example 4.6 An oil-and-gas company now finds it necessary to stop gas flaring. Its E&P (exploration and production) department estimates that the gas processing

7477_book.fm Page 68 Tuesday, March 13, 2007 3:34 PM

68

Computational Economic Analysis for Engineering and Industry

will cost $30,000 in the first year and will decline by $3,000 each year for the next 10 years. As an alternative, a specialized gas processing company has offered to process the gas for a fixed price of $15,000 per year for the next 10 years, payable at the end of each year. If the oil and gas company uses a 7% interest rate, which alternative must be selected using the annual value analysis? Solution The cash-flow diagrams using the ENGINEA software are shown here: Cash-flow diagram based on the E&P department estimates: 0

1

2

3

4

5

6

7

8 6000

24000

21000

10

9000

12000

27000

9 3000

15000

18000

30000

Cash-flow diagram based on the contractor estimates: 0

1

2

3

4

5

6

7

8

9

10

15000 15000 15000 15000 15000 15000 15000 15000 15000 15000 15000

2

3

4

5

6

7

8

9

27000

27000

27000

27000

27000

10

27000

1

27000

0

27000

+

27000

30000

0

27000

+

0

1

2

3

4

5

6

7

8

9

27000

24000

21000

18000

15000

12000

9000

6000

3000

The E&P cash-flow diagram is equivalent to the summation of three cash-flow diagrams that are shown below:

10

7477_book.fm Page 69 Tuesday, March 13, 2007 3:34 PM

Chapter four:

Economic methods for comparing investment alternatives

69

Hence, the NAV computation follows:

(

(

)

(

NAVE& P = −30, 000 − 27 , 000 P/A, 7%, 10 + 3, 000 P/G, 7%, 10

))( A/P, 7%, 10)

= −$19, 433.11

(

)

NAVContractor = −15, 000 A/P, 7%, 10 − 15, 000 = −$17 , 135.66 Therefore, the alternative of using the contractor has the smallest net annual cost and should be selected. Example 4.7 The project engineer of a food-processing company is considering a new labeling style that can be produced by two alternative machines. The respective costs and benefits of the machines follow: Cash flow Initial cost Maintenance costs Annual benefit Salvage value Useful life

Machine A

Machine B

$25,000 400 13,000 6,000 10 years

$15,000 1,600 8,000 3,000 7 years

If the annual interest rate is 12%, which machine should be selected according to the annual cash-flow analysis? Solution The cash-flow diagram for Machine A: 18600 12600 12600 12600 12600 12600 12600 12600 12600 12600

0

25000

1

2

3

4

5

6

7

8

9

10

7477_book.fm Page 70 Tuesday, March 13, 2007 3:34 PM

70

Computational Economic Analysis for Engineering and Industry

Therefore,

(

)

(

)

NPVA = −25, 000 A/P, 12%, 10 + 12 , 600 + 18, 600 A/F , 12%, 10 = $8517.30 The cash-flow diagram for Machine B: 9400

0

6400

6400

6400

6400

6400

6400

1

2

3

4

5

6

7

15000

Therefore,

(

)

(

)

NPVB = −15, 000 A/P, 12%, 7 + 6400 + 9400 A/F , 12%, 7 = $3410.59 The annual cash-flow analysis shows that Machine A should be selected because its net annual value is greater than that of Machine B.

4.3 Internal rate of return analysis The internal rate of return (IRR) is the third and most widely used method of measurement in the industry. It is also referred to as simply rate of return (ROR) or return on investment (ROI). It is defined as the interest rate that equates the equivalent value of investment cash inflows (receipts and savings) to the equivalent value of cash outflows (expenditures) — that is, the interest rate at which the benefits are equivalent to the costs. Let NPV = present value EUAB = equivalent uniform annual benefits EUAC = equivalent uniform annual costs If i* denotes the internal rate of return, then the unknown interest rate can be solved for either using the following expressions:

7477_book.fm Page 71 Tuesday, March 13, 2007 3:34 PM

Chapter four:

Economic methods for comparing investment alternatives

71

Table 4.1 Application of Descartes’ Rule of Signs to Cash Flows Number of Sign Changes 0 1 2 3 4 5

Number of Positive Roots No positive roots One positive root Two positive roots or no positive roots Three positive roots or one positive root Four positive roots or two positive roots or no positive roots Five positive roots or three positive roots or one positive root

(

)

(

)

PW Benefits − PW Costs = 0

(4.4)

EUAB − EUAC = 0 The procedure for selecting the viable alternatives is: • If i* ≥ MARR, accept the alternative as an economically viable project. • If i* < MARR, the alternative is not economically viable. When applied correctly, the internal rate of return analysis will always result in the same decision as with NPV or NAV analysis. However, there are some difficulties with IRR analysis: multiple i*, reinvestment at i*, and computational difficulty. Multiple i* usually occurs whenever there is more than one sign change in the cash-flow profile; hence, there is no unique i* value. This accords with Descartes’ rule of signs, which states that “an n-degree polynomial will have at most as many real positive roots as there are number of sign changes in the coefficients of the polynomial.” Table 4.1 gives a summary of the number of possible positive roots with respect to the number of sign changes in a cash-flow diagram. In addition, there may be no real value of i* that will solve Equation 4.4 even though only real values of i* are valid in economic analysis. Moreover, IRR analysis usually assumes that the selected project can be reinvested at the calculated i*, but this assumption is not valid in economic analysis. These difficulties have given rise to an alternative form of IRR analysis called External Rate of Return (ERR) analysis. Example 4.8 If an investor can receive three payments of $1050 each — at the end of 2, 4, and 6 years, respectively, for an investment of $1500 today, what is the resulting rate of return?

7477_book.fm Page 72 Tuesday, March 13, 2007 3:34 PM

72

Computational Economic Analysis for Engineering and Industry Solution

The cash-flow diagram is shown below: 1050

0

1

2

1050

1050

3

4

5

6

1500

The computation follows: At what i value is 1500 = 1050(P/F,i%,2) + 1050(P/F,i%,4) + 1050(P/F,i%,6)? Try i = 20%:

( ) ( ) ( 1500 = 1050 ( 0.6944 ) + 1050 ( 0.4823 ) + 1050 ( 0.3349 )

1500 = 1050 P/F , 20%, 2 + 1050 P/F , 20%, 4 + 1050 P/F , 20%, 6

)

1500 ≠ 1587.15 Try i = 22%:

( ) ( ) ( 1500 = 1050 ( 0.6719 ) + 1050 ( 0.4514 ) + 1050 ( 0.3033 )

1500 = 1050 P/F , 22%, 2 + 1050 P/F , 22%, 4 + 1050 P/F , 22%, 6

)

1500 ≠ 1497.93 The tried i values show that the resulting rate of return lies between 20 and 22%. 20 − X 1587.15 − 1500 = 20 − 22 1587.15 − 1497.93 Therefore, X ≈ 21.95%. The resulting rate of return is 21.95%.

7477_book.fm Page 73 Tuesday, March 13, 2007 3:34 PM

Chapter four:

Economic methods for comparing investment alternatives

73

Example 4.9 A manufacturing plant is for sale for $240,000. According to a feasibility study, the plant will produce a profit of $65,000 the first year, but the profit will decline at a constant rate of $5,000 until it eventually reaches zero. Should an investor looking for a rate of return of at least 20% per year accept this investment? Solution

60000

55000

50000

45000

40000

35000

30000

25000

20000

15000

10000

5000

0

65000

The cash-flow diagram of this proposed investment is:

1

2

3

4

5

6

7

8

9

10

11

12

13

14

240000

(

)

(

)

At what i value is 240, 000 = 65, 000 P/A, i%, 14 − 5000 P/G, i%, 14 ? Try i = 20%:

( ) ( = 65, 000 ( 4.6106 ) − 5000 (17.6008 )

240, 000 = 65, 000 P/A, 20%, 14 − 5000 P/G, 20%, 14

)

≠ 211, 685 Because the NPV for this investment at 20% per year is less than the initial cost of investment, an investor looking for at least a 20% rate of return should not accept this investment. The actual rate of return for this investment is as follows: Try i = 15%:

( ) ( = 65, 000 ( 5.7245 ) − 5000 ( 24.9725 )

240, 000 = 65, 000 P/A, 15%, 14 − 5000 P/G, 15%, 14

≠ 247 , 220

)

7477_book.fm Page 74 Tuesday, March 13, 2007 3:34 PM

74

Computational Economic Analysis for Engineering and Industry

Therefore, the actual rate of return should lie between 15 and 20%: 15 − X 247 , 200 − 240, 000 = 15 − 20 247 , 200 − 211, 685 X ≈ 16% Hence, the actual rate of return is 16%, which is less than the desired 20%. This further supports our initial conclusion that this is not a feasible investment for an investor looking for a rate of return of at least 20%.

4.3.1

External rate of return analysis

The difference between ERR and IRR is that ERR takes into account the interest rate external to the project at which the net cash flow generated or required by the project over its useful life can be reinvested or borrowed. Therefore, this method requires that one know the external MARR for a similar project under evaluation. The expression for calculating ERR is given by

(

F = P 1 + i′

)

n

(4.5)

Using Equation 4.5 requires several steps: (1) the net present value (P) of all net cash outflows is computed at the given external MARR (ε); (2) the net future value (F) of all net cash inflows is computed at the given ε; and (3) these values (P and F) are substituted into Equation 4.5 in order to determine the ERR (i') for the investment. Using this method, a project is acceptable when the calculated i' is greater than ε. However, if i' is equal to ε (a break-even situation), noneconomic factors may be used to justify the final decision. The ERR method has two advantages over the IRR method: it does not resort to simple trial-and-error in determining the unknown rate of return, and it is not subject to the possibility of multiple rates of return even when there are several sign changes in the cash-flow profile. Example 4.10 A new technology can be purchased today for $25,000, but it will lose $1,000 each year for the first 4 years. An additional $10,000 can be invested in the technology during the fourth year for a profit of $6,000 each year, from the fifth year through the fifteenth year. The salvage value of the technology after 15 years is $31,000. What is the ERR for this investment if ε is 10%? Solution For easy computation, we draw the cash-flow diagram for the investment using the ENGINEA software:

7477_book.fm Page 75 Tuesday, March 13, 2007 3:34 PM

Chapter four:

Economic methods for comparing investment alternatives

75 37000

6000 6000 6000 6000 6000 6000 6000 6000 6000 6000 0

1 2 3 1000 1000 1000

4

5

6

7

8

9

10

11

12

13

14

15

11000 25000

The ERR steps result in the following computation: 1. P = 25,000 + 1000(P/A,10%,3) + 11,000(P/F,10%,4) = $34,999.90. 2. F = 6000(F/A,10%,11) + 31,000 = $142.187.20. 3. F = P(1 + i′)n = $142,187.20 = $34,999.90(1 + i′)15. Therefore,

(

4.06 = 1 + i′

( 4.06)

1

15

)

15

= 1 + i′

1.098 = 1 + i′ i′ = 0.098 That is, ERR = 9.8%. This ERR is less than the external MARR; hence, this is not a feasible investment.

4.4 Incremental analysis Under some circumstances, IRR analysis does not provide the same ranking of alternatives as do NPV and NAV analyses for multiple alternatives. Hence, there is a need for a better approach for analyzing multiple alternatives using the IRR method. Incremental analysis can be defined as the evaluation of the differences between alternatives. The procedure essentially decides whether or not differential costs are justified by differential benefits. Incremental analysis is mandatory for economic analysis involving the use of IRR and benefit/cost (B/C) ratio analyses and when evaluating two or more mutually exclusive alternatives. It is not used for independent projects because more than one project can be selected. The steps involved in using incremental analysis follow.

7477_book.fm Page 76 Tuesday, March 13, 2007 3:34 PM

76

Computational Economic Analysis for Engineering and Industry 1. If the IRR (B/C ratio) for each alternative is given, reject all alternatives with an IRR < MARR (B/C < 1.0). 2. Arrange other alternatives in increasing order of initial cost (total costs). 3. Compute incremental cash flow pairwise, starting with the first two alternatives. 4. Compute incremental measures of value based on the appropriate equations. 5. Use the following criteria for selecting the alternatives that will advance to the next stage of comparisons: a. If ∆IRR ≥ MARR, select the higher-cost alternative. b. If ∆B/C ≥ 1.0, select the higher-cost alternative. 6. Eliminate the defeated alternative and repeat Steps 3 to 5 for the remaining alternatives. 7. Continue until only one alternative remains. This last alternative is the most economically viable one.

4.5 Guidelines for comparison of alternatives • Total Cash-Flow Approach (Ranking Approach): a. Use the individual cash flow for each alternative to calculate the following measures of worth: PW, AW, IRR, ERR, Payback Period, or B/C ratio. b. Rank the alternatives on the basis of the measures of worth. c. Select the highest-ranking alternative as the preferred alternative. • Incremental Cash-Flow Approach: Find the incremental cash flow needed to go from a “lower-cost” alternative to a “higher-cost” alternative. Then calculate the measures of worth for the incremental cash flow. The incremental measures of worth are denoted as: ∆PW, ∆AW, ∆IRR, ∆ERR, ∆PB, ∆B/C Step 1. Arrange alternatives in increasing order of initial cost. Step 2. Compute incremental cash flow pairwise, starting with the first two alternatives. Step 3. Compute incremental measures of worth as explained previously. Step 4. Use the following criteria for selecting the alternatives that will advance to the next stage of comparisons: • If ∆PW ≥ 0, select higher-cost alternative. • If ∆AW ≥ 0, select higher-cost alternative. • If ∆FW ≥ 0, select higher-cost alternative. • If ∆IRR ≥ MARR, select higher-cost alternative. • If ∆ERR ≥ MARR, select higher-cost alternative. • If ∆B/C ratio ≥ 1.0, select higher-cost alternative.

7477_book.fm Page 77 Tuesday, March 13, 2007 3:34 PM

Chapter four:

Economic methods for comparing investment alternatives

77

Step 5. Eliminate the defeated alternative and repeat Steps 2, 3, and 4 for the remaining alternatives. Continue until only one alternative remains. This last alternative is the preferred alternative. Example 4.11 The estimated cash flows for three design alternatives for a laboratory steam-generating plant at a research institute are shown here.

Capital investment Expected annual savings Useful life

A

Alternatives B

C

$85,600 $10,200 10

$93,200 $15,100 8

$71,800 $12,650 9

If MARR is 4% per year, compounded yearly, which alternative, if any, should be selected, using the ROR method if the proposed project is expected to last for only 8 years? Solution For this problem, DN is a valid alternative because the question says “if any.” If the question had said “must” instead of “if any,” then, DN would not be a valid alternative. In order to use the ROR method, it must be the incremental ROR method. Therefore, we use the “incremental ROR” method. Step 1: Rank the alternatives in order of increasing capital investment (initial cost). For this problem, the ranking is as follows: (1) DN, (2) C, (3) A, and (4) B. Steps 2 to 5: We now compute the incremental ROR and make a selection: Changes in investment capital Changes in annual savings Incremental IRR Is incremental cost justified? Therefore, select

C – DN –$71,800 $12,650 8.34% Yes C

A–C –$13,800 –$2,450 $35,000, we replace the defender now. Using the ENGINEA software, we click on the “Result” button for the

7477_book.fm Page 94 Friday, March 30, 2007 2:27 PM

94

Computational Economic Analysis for Engineering and Industry

decision. A snapshot of the decision note using the ENGINEA software is shown here:

Example 5.3 An existing machine has the TMC per year shown as follows. What is the proper replacement decision for this machine when it is compared to the machine described in Example 5.1 at a 10% interest rate? EOY

TMC

1 2 3 4 5

$25,000 $29,000 $32,000 $36,000 $39,000

Solution To answer this question we have to determine whether the given TMC is increasing or not. Based on the data given, the TMC is increasing; therefore, we should compare the TMC of the defender with the minimum EUAC of the challenger in order to determine when the defender should be replaced (option 1). We do not need to compute the EUAC for the defender. Recall that the minimum EUAC for the challenger is $35,000. Comparing this value to the TMC, we see that the defender should be kept for 3 years, after which it should be replaced with the challenger because the TMC for the defender in each of the first 3 years is less than the minimum EUAC for the challenger. However, an extra cost of only $1,000 ($36,000 – $35,000) would be incurred if the defender were to be kept for 4 more years instead of 3 more years. This decision is based on the assumption that the data given will remain valid for the next 3 years and that the challenger would still be available to replace the defender. However, a more appropriate approach is to keep the defender for another year and revisit the study in the following year. This approach would allow for new and possibly additional information for the analysis. The problems described in the preceding text are based on the assumption that the TMC of the defender is available or can be computed. However,

7477_book.fm Page 95 Friday, March 30, 2007 2:27 PM

Chapter five:

Asset replacement and retention analysis

95

in some cases, this may not be possible. In these cases, the replacement analysis can be done from either the perspective of opportunity cost or the perspective of cash flow. Both perspectives will give the same results when the useful life of the defender and the challenger is the same; however, they will give different and inconsistent results when the useful lives of the defender and challenger are different. The opportunity cost perspective, however, gives consistent results in both cases and is usually used. We will use this perspective to analyze a replacement problem when the marginal cost is not available in the following example. Example 5.4 For a replacement analysis, the MV of an asset (defender) bought 6 years ago for $900 is $300, and the first cost of a potential challenger is $1200. The remaining life for the defender is 4 years, and the useful life for the challenger is also 4 years. What is the replacement decision if MARR is 10% per year? Solution The first cost for the defender is the current MV, which is $300, and the first cost for the challenger is $1200. We determine the AV of the assets over their remaining and useful lives, respectively:

(

)

AVdefender = $300 A/P, 10%, 4 = $94.64

(

)

AVchallenger = $1, 200 A/P, 10%, 4 = $378.56 The difference in AV is $283.92, which indicates that the additional cost of the challenger over the next 4 years is justified; therefore, we replace the defender now. Example 5.5 Repeat Example 5.4, except that, here, the remaining life of the defender is 4 years but the useful life of the challenger is 8 years. Solution

(

)

AVdefender = $300 A/P, 10%, 4 = $94.64

(

)

AVchallenger = $1, 200 A/P, 10%, 8 = $224.93 The difference in AV now is $130.29, which still indicates that the additional cost of the challenger over the next 8 years is justified; therefore, we replace the defender now.

7477_book.fm Page 96 Friday, March 30, 2007 2:27 PM

96

Computational Economic Analysis for Engineering and Industry

The problems described in the preceding text deal with replacement analysis; however, in some cases it may be desirable to abandon the asset without a replacement. In such cases, we need to determine the most economic period in which to abandon the asset. This is similar to determining the economic life of an asset in that we are determining the span of economic life that maximizes profits or minimizes costs. The following example illustrates the procedures in abandonment problems. Example 5.6 A small commercial pump is being considered for abandonment. The pump has the following net cash flows and estimated MV (abandonment value of pump) over its useful life. Determine the optimum period for the pump to be abandoned if it was acquired for $5500 and its useful life should not be more than 4 years. MARR for the firm is 10% per year.

1 Annual benefits Estimated MV

$500 $4000

End of Year 2 3 $600 $3500

$750 $3000

4 $900 $2500

Solution In order to solve this problem, we compute the present value of keeping the pump for 1, 2, 3, or 4 years. The year that has the maximum present value is the optimum year of abandonment. If we keep the pump for 1 year:

(

)(

)

PV1 = −$5500 + $500 + $4000 P/F , 10%, 1 = −$1409.05 If we keep it for 2 years:

(

) (

)(

PV2 = −$5500 + $500 P/F , 10%, 1 + $600 + $3500 P/F , 10%, 2 = −$1657.21 If we keep it for 3 years:

(

)

(

PV3 = −$5500 + $500 P/F , 10%, 1 + $600 P/F , 10%, 2

(

)(

+ $750 + $3, 000 P/F , 10%, 3 = −$1732.24

)

)

)

7477_book.fm Page 97 Friday, March 30, 2007 2:27 PM

Chapter five:

Asset replacement and retention analysis

97

If we keep it for 4 years:

(

)

(

PV1 = −$5500 + $500 P/F , 10%, 1 + $600 P/F , 10%, 2

(

) (

)(

)

+$750 P/F , 10%, 3 + $900 + $2500 P/F , 10%, 4

)

= −$1663.94 Based on the computations, keeping the pump for 1 year minimizes costs; therefore, the optimum period to abandon the pump would be after 1 year.

Practice problems for replacement and retention analysis 5.1 A new machine is expected to have the following MV and annual expenses. If the cost of capital is 12% per year, what is the expected life and minimum EUAC of this machine? EOY

MV

Annual Expenses

0 1 2 3 4 5

$150,000 $85,000 $73,000 $55,000 $30,000 $20,000

$5,000 $10,000 $18,000 $29,000 $40,000 $55,000

5.2 An existing machine has the following TMC per year. When do you think it is most appropriate to replace this machine with the new machine described in Problem 5.1? EOY

TMC

1 2 3 4 5

$55,000 $40,000 $55,000 $48,000 $60,000

5.3 Repeat Problem 5.2 if the TMC per year is as follows: EOY

TMC

1 2 3 4 5

$45,000 $50,000 $65,000 $78,000 $80,000

7477_book.fm Page 98 Friday, March 30, 2007 2:27 PM

98

Computational Economic Analysis for Engineering and Industry 5.4 An old machine was installed 3 years ago at a cost of $100,000, and you still owe $40,000 before it is completely paid off. It has a present realizable market value of $40,000. If kept, it can be expected to last 5 more years with annual expenses of $10,000 and an MV of $20,000 at the end of 5 years. This machine can be replaced with an improved version costing $125,000 with an expected life of 10 years. The challenger will have estimated annual expenses of $4,200 and a salvage value of $25,000 at the end of 10 years. The machine is expected to be needed indefinitely, and the effect of taxes should be ignored. Using MARR of 20% per year, determine whether to keep or replace the old machine. 5.5 Determine the economic life of a new piece of equipment if it initially cost $10,000. The first-year maintenance cost is $1,500. Maintenance cost is projected to increase $500 per year for each year after the first. Complete the table below if the interest rate is 15% per year. EOY

Maintenance Cost

1 2 3 4 5 6 7 8 9 10

$1,500 2,000 2,500 3,000 3,500 4,000 4,500 5,000 5,500 6,000

EUAC of Capital Recovery

EUAC of Maintenance

Total EUAC

7477_book.fm Page 99 Tuesday, March 13, 2007 3:34 PM

chapter six

Depreciation methods The concept of depreciation is important in economic analysis primarily because it is useful for income-tax computation purposes; however, it has an important role to play in economic analysis for other purposes as well. Depreciation is used in relation to tangible assets such as equipment, computers, machinery, buildings, and vehicles. Almost everything (except land, which is considered a nondepreciable asset) depreciates as time proceeds. Depreciation can be defined as: • A decline in the market value (MV) of an asset (deterioration). • A decline in the value of an asset to its owner (obsolescence). • The allocation of the cost of an asset over its depreciable or useful life. Accountants usually use this definition, and it is adopted in economic analysis for income-tax computation purposes. Therefore, depreciation is a way to claim, over time, an already paid expense for a depreciable asset. For an asset to be depreciated, it must satisfy these three requirements: 1. The asset must be used for business purposes to generate income. 2. The asset must have a useful life that can be determined, and that is longer than 1 year. 3. The asset must be one that decays, gets used up, wears out, becomes obsolete, or loses value to the owner over time as a result of natural causes.

6.1 Terminology of depreciation • Depreciation: The annual depreciation amount, Dt, is the decreasing value of the asset to the owner. It does not represent an actual cash flow or actual usage pattern.

99

7477_book.fm Page 100 Tuesday, March 13, 2007 3:34 PM

100

Computational Economic Analysis for Engineering and Industry • Book depreciation: This is an internal description of depreciation. It is the reduction in the asset investment due to its usage pattern and expected useful life. • Tax depreciation: This is used for after-tax economic analysis. In the U.S. and many other countries, the annual tax depreciation is tax deductible using the approved method of computation. • First cost or unadjusted basis: This is the cost of preparing the asset for economic use. This is also called the basis. This term is used when an asset is new. Adjusted basis is used after some depreciation has been charged. • Book value: This represents the difference between the basis and the accumulated depreciation charges at a particular period. It is also called the undepreciated capital investment and is usually calculated at the end of each year. • Salvage value: Estimated trade-in or market value at the end of the asset’s useful life. It may be positive, negative, or zero. It can be expressed as a dollar amount or as a percentage of the first cost. • Market value: This is the estimated amount realizable if the asset were sold in an open market. This amount may be different from the book value. • Recovery period: This is the depreciable life of an asset in years. Often there are different n values for book and tax depreciations. Both values may be different from the asset’s estimated productive life. • Depreciation or recovery rate: This is the fraction of the first cost removed by depreciation each year. Depending on the method of depreciation, this rate may be different for each recovery period. • Half-year convention: This is used with the Modified Accelerated Cost Recovery System (MACRS) depreciation method, which will be discussed later. It assumes that assets are placed in service or disposed of in midyear, regardless of when during the year these placements or disposal actually occur. There are also midquarter and midmonth conventions.

6.2 Depreciation methods There are five principal methods of depreciation: • Classical (historical) depreciation methods: • Straight-Line (SL) • Declining balance (DB) • Sum-of-years digits (SYD) • Unit-of-production • Modified accelerated cost recovery system (MACRS)

7477_book.fm Page 101 Tuesday, March 13, 2007 3:34 PM

Chapter six:

Depreciation methods

101

In describing each of these methods, let n B S Dt MV BVt d t

6.2.1

= = = = = = = =

recovery period in years first cost, unadjusted basis, or basis estimated salvage value annual depreciable charge market value book value after period t depreciation rate = 1/n year (t = 1, 2, 3, …, n)

Straight-line (SL) method

This is the simplest and best-known method of depreciation. It assumes that a constant amount is depreciated each year over the depreciable (useful) life of the asset; hence, the book value decreases linearly with time. The SL method is considered the standard against which other depreciation models are compared. It offers an excellent representation of an asset used regularly over an estimated period, especially for book depreciation purposes. The annual depreciation charge is Dt =

B−S = B−S d n

(

)

(6.1)

The BV after t year is BVt = B −

(

)

t B − S = B − tDt n

(6.2)

Example 6.1 A piece of new petroleum-drilling equipment has a cost basis of $5000 and a 6-year depreciable life. The estimated salvage value of the machine is $500 at the end of 6 years. What are the annual depreciation amounts and the book value at the end of each year using the SL method? Solution For this problem, the basis (B) = $5000, S = $500, and n = 6. To compute the annual depreciation amounts and the book value, we use Equation 6.1 and Equation 6.2, respectively. Depreciation analysis is another module in ENGINEA. Using ENGINEA, we obtain the following results:

7477_book.fm Page 102 Tuesday, March 13, 2007 3:34 PM

102

Computational Economic Analysis for Engineering and Industry

To use this module, we select the depreciation method, input the initial cost (basis), the salvage value, and the life (depreciable life). We click on “Calculate” to view the depreciation table. As expected, the annual depreciation amount is constant at $750. In addition, the BV at the end of year 6 is $500, which is equal to the salvage value at the end of the depreciable life.

6.2.2

Declining balance (DB) method

This method is commonly applied as the book depreciation method in the industry because it accelerates the write-off of an asset value. It is also called the Fixed (Uniform) Percentage method; therefore, a constant depreciation rate is applied to the book value of the asset. According to the Tax Reform Act of 1986, two rates are applied to the straight-line rate: 150% and 200%. If 150% is used, it is called the DB method, and if 200% is used, it is called the Double Declining Balance (DDB) method. The DB annual depreciation charge is 1.5B  1.5B  Dt = 1−  n  n 

t −1

(6.3)

7477_book.fm Page 103 Tuesday, March 13, 2007 3:34 PM

Chapter six:

Depreciation methods

103

The total DB depreciation at the end of t years is t   t 1.5    = B 1 − 1 − d  B 1 −  1 −     n     

(

)

(6.4)

The BV at the end of t years is t

 1.5  B1 − = B 1− d n  

(

)

t

(6.5)

For the DDB (200% depreciation) method, substitute 2.0 for 1.5 in Equation 6.3, Equation 6.4, and Equation 6.5. It should be noted that salvage value is not used in the equations for the DB or DDB methods; therefore, these methods are independent of the salvage value of the asset. The implication of this is that the depreciation schedule may go below an implied salvage value, above an implied salvage value, or just at the implied salvage value. Any of these three situations is possible in the real world. However, the U.S. Internal Revenue Service (IRS) does not permit a deduction for depreciation charges below the salvage value, whereas companies generally do not like to deduct depreciation charges that would keep the book value above the salvage value. The solution to this problem is to use a composite depreciation method. The IRS provides that a taxpayer may switch from DB or DDB to SL at any time during the life of an asset. However, the question is when it is a better time to switch. The criterion used to determine when to switch is based on the maximization of the present value of the total depreciation. Figure 6.1 shows a graphical depiction of the three possible profiles of salvage values (S) in relation to ending book value (BV). Part (a) of the figure shows ending book value being below the salvage value, part (b) shows ending book value being

P

P

P

BV

BV

BV

S

S

S

N

N

N

Years

Years

Years

(a)

(b)

(c)

Figure 6.1 Effect of salvage value on DDB depreciation.

7477_book.fm Page 104 Tuesday, March 13, 2007 3:34 PM

104

Computational Economic Analysis for Engineering and Industry

equal to the salvage value, and part (c) shows the ending book value being above the salvage value. Part (b) is the desirable scenario. If Part (a) occurs, less depreciation should be charged in the later years, or depreciation should be terminated to bring the BV up to the estimated salvage value at the end of N years. If Part (c) occurs, more depreciation should be charged in the later years to bring the BV down to the estimated salvage value at the end of N years. So, we have the following options: Resolution of (a): Termination of depreciation as soon as value is reached. Resolution of (c): Switch from a declining balance to straight-line depreciation in any year when the latter yields a larger depreciation charge. Example 6.2 Repeat Example 6.1 using the DB and DDB methods. Solution Again, we use ENGINEA to solve this problem. The annual depreciation and BV at the end of each year are obtained by substituting the given values into Equation 6.3 and Equation 6.5, respectively. For DB using 150%:

7477_book.fm Page 105 Tuesday, March 13, 2007 3:34 PM

Chapter six:

Depreciation methods

105

For DDB:

6.2.3

Sums-of-years’ digits (SYD) method

This method results in larger depreciation charges during the early years of an asset (than SL) and smaller charges during the latter part of the estimated useful life; however, write-off is not as rapid as for DDB or MACRS (modified accelerated cost recovery system). As in the case of the SL method, this method uses the salvage value in computing the annual depreciation charge. The annual depreciation charge is as follows: Dt =

n−t+1 B − S = dt B − S SUM

SUM =

(

(

)

(

) (6.6)

)

n n+1 2

The BV at the end of t years is   t t  n − + 0.5 2   BVt = B − B−S SUM

(

)

(6.7)

7477_book.fm Page 106 Tuesday, March 13, 2007 3:34 PM

106

Computational Economic Analysis for Engineering and Industry Example 6.3

Repeat Example 6.1 using the SYD method. Solution Equation 6.6 and Equation 6.7 can be used to obtain the annual depreciation and BV at the end of each year. The result using ENGINEA is displayed here:

6.2.4

Modified accelerated cost recovery system (MACRS) method

This is the only approved tax depreciation method in the U.S. It is a composite method that automatically switches from DB or DDB to SL depreciation. The switch usually takes place whenever the SL depreciation results in larger depreciation charges, that is, a more rapid reduction in the BV of the asset. One advantage of the MACRS method is that it assumes that the salvage value is zero; therefore, it always depreciates to zero. Another outstanding advantage of this method is that it uses property classes that specify the recovery periods, n. The method adopts the half-year convention, which makes the actual recovery period 1 year longer than the specified period. The half-year convention means that the IRS assumes that the assets are placed in service halfway through the year, no matter when the assets were actually placed in service. This convention is also applicable when the asset is disposed of before the end of the depreciation period. The MACRS method

7477_book.fm Page 107 Tuesday, March 13, 2007 3:34 PM

Chapter six:

Depreciation methods

107

consists of two systems for computing depreciation deductions: the General Depreciation System (GDS) and the Alternative Depreciation System (ADS). The ADS system is used for properties placed in any tax-exempt use, as well as properties used predominantly outside the U.S. The system provides a longer recovery period and uses only the SL method of depreciation. Therefore, this system is generally not considered an option for economic analysis. However, any property that qualifies for GDS can be depreciated under ADS, if preferred. The following information is required to depreciate an asset using the MACRS method: • • • • •

The cost basis The date the property was placed in service The property class and recovery period The MACRS depreciation system to be used (GDS or ADS) The time convention that applies (for example, the half-year or quarteryear convention)

The steps involved in using the MACRS depreciation method follow: 1. Determine the property class of the asset being depreciated using published tables (see Table 6.1 and Table 6.2). Any asset not in any of the stated classes is automatically assigned a 7-year recovery period under the GDS system. 2. After the property class is known, find the appropriate published depreciation schedule (see Table 6.3). For nonresidential real property, see Table 6.4. 3. The last step is to multiply the asset’s cost basis by the depreciation schedule for each year to get the annual depreciation charge as stated in Equation 6.8. The MACRS annual depreciation amount is

(

) (

Dt = First cost × Tabulated depreciation sched dule

)

= dt B

(6.8)

The annual book value is BVt = First cost − Sum of accumulated depreciatiion t

= B−

∑D

j

j =1

(6.9)

7477_book.fm Page 108 Tuesday, March 13, 2007 3:34 PM

108

Computational Economic Analysis for Engineering and Industry

Table 6.1 Property Classes Based on Asset Description Asset Class 00.11 00.12 00.22 00.241 00.25 00.40 01.11 01.21 13.00 13.30 15.00 20.1 20.10 20.20 24.10 32.20 48.10 48.2 49.12 49.13 49.23 50.00 80.00 a

Asset Description Office furniture, fixtures, and equipment Information systems: computers/ peripherals Automobiles, taxis Light general-purpose trucks Railroad cars and locomotives Industrial steam and electric distribution Cotton gin assets Cattle, breeding or dairy Offshore drilling assets Petroleum refining assets Construction assets Manufacture of motor vehicles Manufacture of grain and grain mill products Manufacture of yarn, thread, and woven fabric Cutting of timber Manufacture of cement Telephone distribution plant Radio and television broadcasting equipment Electric utility nuclear production plant Electric utility steam production plant Natural gas production plant Municipal wastewater treatment plant Theme and amusement assets

Class Life (Years) ADRa 10 6 3 4 15 22 10 7 7.5 16 6 12 17

MACRS Property Class (Years) GDS ADS 7 5 5 5 7 15 7 5 5 10 5 7 10

10 6 6 6 15 22 10 7 7.5 16 6 12 17

11

7

11

6 20 24 6

5 15 15 5

6 20 24 6

20 28 14 24 12.5

15 20 7 15 7

20 28 14 24 12.5

ADR = Asset Depreciation Range.

Example 6.4 Repeat Example 6.1 using MACRS. Solution Because this is petroleum-drilling equipment, the property class is 5 years, according to Table 6.2. Therefore, the applicable percentage is as tabulated in the third column of Table 6.3. These tabulated values are substituted into Equation 6.8 to compute the annual depreciation amount. The sum of the accumulated depreciation is substituted into Equation 6.9 to compute the BV at the end of each year. The depreciation schedule using ENGINEA is as follows:

7477_book.fm Page 109 Tuesday, March 13, 2007 3:34 PM

Chapter six:

Depreciation methods

109

Because of the assumption of the half-life convention, the equipment is depreciated for 6 years even though the property class is 5 years. The BV at the end of the 6th year is zero because MACRS assumes that the salvage value is always zero.

Practice problems: Depreciation methods 6.1 A construction company purchased a piece of equipment for $220,000 five years ago. The current market value of the equipment is $70,000, and its book value is $50,000. A new piece of similar equipment is currently advertised for $350,000. What are the annual depreciation amounts at end of each year using the straight-line method? 6.2 A manufacturing company has bought some machinery for the manufacture of electronic devices with a cost basis of $2 million. Its market value at the end of 6 years is estimated to be $200,000. The before-tax MARR is 20% per year. Develop a GDS depreciation schedule for this machinery. 6.3 Some front-office furniture cost $85,000 and has an estimated salvage value of $10,000 at the end of 7 years of useful life. Compute the depreciation schedules and BV to the end of the useful life of the furniture by the following methods: a. SL b. SYD c. DDB d. DB e. MACRS

7477_book.fm Page 110 Tuesday, March 13, 2007 3:34 PM

110

Computational Economic Analysis for Engineering and Industry

Table 6.2 MACRS GDS Property Classesa Property Class Personal Property (All Property Except Real Estate) Special handling devices for food processing and beverage manufacture Special tools for the manufacture of finished plastic products, fabricated metal products, and motor vehicles Property with ADR class life of 4 years or less 5-Year property Automobilesa and trucks Aircraft (of non-air-transport companies) Equipment used in research and experimentation Computers Petroleum drilling equipment Property with ADR class life of more than 4 years and less than 10 years 7-Year property All other property not assigned to another class Office furniture, fixtures, and equipment Property with ADR class life of 10 years or more and less than 16 years 10-Year property Assets used in petroleum refining and certain food products Vessels and water transportation equipment Property with ADR class life of 16 years or more and less than 20 years 15-Year property Telephone distribution plants Municipal sewage treatment plants Property with ADR class life of 20 years or more and less than 25 years 20-Year property Municipal sewers Property with ADR class life of 20 years or more and less than 25 years 3-Year property

27.5 Years 39 Years

Real Property (Real Estate) Residential rental property (does not include hotels and motels) Nonresidential real property

a

The depreciation deduction for automobiles is limited to $7660 (maximum) the first tax year, $4900 the second year, $2950 the third year, and $1775 per year in subsequent years. Source: U.S. Department of the Treasury (2007), Internal Revenue Service Publication 946, How to Depreciate Property. Washington, D.C.

6.4 Make a plot of the depreciation schedules computed in Problem 6.3 and discuss the depreciation implication of each schedule for small-size, medium-size, and large-size companies. 6.5 The MACRS method is a hybrid depreciation method of either DB or DDB with a switch to the SL method. Using the tabulated percentage in Table 6.3, determine which property classes use DB with a switch to SL, and which classes use DDB with a switch to SL.

7477_book.fm Page 111 Tuesday, March 13, 2007 3:34 PM

Chapter six:

Depreciation methods

111

Table 6.3 MACRS Depreciation for Personal Property Based on the Half-Year Convention Applicable Percentage for Property Class 5-Year 7-Year 10-Year 15-Year Property Property Property Property

Recovery Year

3-Year Property

1 2 3

33.33 44.45 14.81

20.00 32.00 19.20

14.29 24.49 17.49

10.00 18.00 14.40

5.00 9.50 8.55

3.750 7.219 6.677

4 5 6

7.41

11.52 11.52 5.76

12.49 8.93 8.92

11.52 9.22 7.37

7.70 6.93 6.23

6.177 5.713 5.285

8.93 4.46

6.55 6.55 6.56

5.90 5.90 5.91

4.888 4.522 4.462

6.55 3.28

5.90 5.91 5.90

4.461 4.462 4.461

13 14 15

5.91 5.90 5.91

4.462 4.461 4.462

16 17 18

2.95

4.461 4.462 4.461

7 8 9 10 11 12

19 20 21

20-Year Property

4.462 4.461 2.231

Table 6.4 MACRS Depreciation for Real Property (Real Estate) Recovery Year 1 2–39 40

1

2

3

Recovery Percentage for Nonresidential Real Property (Month Placed in Service) 4 5 6 7 8 9

10

11

12

2.461 2.247 2.033 1.819 1.605 1.391 1.177 0.963 0.749 0.535 0.321 0.107 2.564 2.564 2.564 2.564 2.564 2.564 2.564 2.564 2.564 2.564 2.564 2.564 0.107 0.321 0.535 0.749 0.963 1.177 1.391 1.605 1.819 2.033 2.247 2.461

7477_book.fm Page 112 Tuesday, March 13, 2007 3:34 PM

7477_book.fm Page 113 Friday, March 30, 2007 2:27 PM

chapter seven

Break-even analysis The term break-even analysis refers to an analysis to determine the point at which there exists a balanced performance level when a project’s income is equal to its expenditure. The total cost of an operation is expressed as the sum of the fixed and variable costs with respect to output quantity. That is, TC(x) = FC + VC(x) where x is the number of units produced, TC(x) is the total cost of producing x units, FC is the total fixed cost, and VC(x) is the total variable cost associated with producing x units. The total revenue (TR) resulting from the sale of x units is defined as follows: TR(x) = px where p is the price per unit. The profit (P) due to the production and sale of x units of the product is calculated as P(x) = TR(x) – TC(x) The break-even point of an operation is defined as the value of a given parameter that will result in neither profit nor loss. The parameter of interest may be the number of units produced, the number of hours of operation, the number of units of a resource type allocated, or any other measure of interest. At the break-even point, we have the following relationship: TR(x) = TC(x) or P(x) = 0

7.1 Illustrative examples In some cases, there may be a known mathematical relationship between cost and the parameter of interest. For example, there may be a linear cost relationship between the total cost of a project and the number of units

113

7477_book.fm Page 114 Friday, March 30, 2007 2:27 PM

114

Computational Economic Analysis for Engineering and Industry $

Total revenue Total cost

Variable cost FC

0

Fixed cost

Break even point

x (Units)

Figure 7.1 Break-even point for a single project.

produced. Such cost expressions facilitate straightforward break-even analysis. Figure 7.1 shows an example of a break-even point for a single project. Figure 7.3 shows examples of multiple break-even points that exist when multiple projects are compared. When two project alternatives are compared, the break-even point refers to the point of indifference between the two alternatives, or the point at which it does not matter which alternative is chosen. In Figure 7.3, x1 represents the point where projects A and B are equally desirable, x2 represents the point at which A and C are equally desirable, and x3 represents the point at which B and C are equally desirable. The figure shows that if we are operating below a production level of x2 $

y1 y2

0

x1

x2

Figure 7.2 Incremental production mapped to incremental cost.

x (Units)

7477_book.fm Page 115 Friday, March 30, 2007 2:27 PM

Chapter seven:

Break-even analysis $

115

A B

Total profit

C

0

x1 x2

x3

x (Units)

Figure 7.3 Break-even points for multiple projects.

units, then project C is the preferred project among the three. If we are operating at a level more than x2 units, then project A is the best choice. For x2 – x1 incremental production, there is y2 – y1 incremental cost. Example 7.1 Three project alternatives are being considered for producing a new product. The required analysis will determine which alternative should be selected on the basis of how many units of the product are produced per year. Based on past records, there is a known relationship between the number of units produced per year, x, and the net annual profit, P(x), from each alternative. The level of production is expected to be between 0 and 250 units per year. The net annual profits (in thousands of dollars) are as follows for each alternative: Project A: P(x) = 3x – 200 Project B: P(x) = x Project C: P(x) = (1/50)x2 – 300 This problem can be solved mathematically by finding the intersection points of the profit functions and evaluating the respective profits over the given range of product units. However, it can also be solved by a graphical approach. Figure 7.4 shows the break-even chart, which is simply a plot of the profit functions. The plot shows that Project B should be selected if between 0 and 100 units are to be produced. Project A should be selected if between 100 and 178.1 units (178 physical units) are to be produced. Project C should be selected if more than 178 units are to be produced. It should be noted that if less than 66.7 units (66 physical units) are produced, Project A will generate a net loss rather than a net profit. Similarly, Project C will generate losses if less than 122.5 units (122 physical units) are produced.

7477_book.fm Page 116 Friday, March 30, 2007 2:27 PM

116

Computational Economic Analysis for Engineering and Industry $ Project C

950

Project A

Project B

0

66.7

100 122.5 150

178.1

250

Units (x)

–200 –300

Figure 7.4 Plot of profit functions.

Example 7.2 RAB General Hospital must decide whether it should perform some medical tests for its patients in-house or whether it should pay a specialized laboratory to undertake the procedures. To perform the procedures in-house, the hospital will have to purchase computers, printers, and other peripherals at a cost of $15,000. The equipment will have a useful life of 3 years, after which it will be sold for $3,000. The employee who manages the tests will be paid $48,000 per year. In addition, each test will have an average cost of $4. Alternatively, the company can outsource the procedure at a flat-rate fee of $20 per test. At an interest rate of 10% per year, how many tests must the hospital perform each year in order for the alternatives to break even? Solution Let X = number of tests per year. The required equation is

(

)

(

)

I A / P, i%, n − Annual Salary + SV A / F , i%, n − 4X = −200X where I is the cost of purchasing the equipment, and SV is its salvage value after 3 years. The left-hand side of the equation shows the cost when the procedures are performed in-house, and the right-hand side of the equation represents the cost when the procedures are performed by the specialized laboratory.

7477_book.fm Page 117 Friday, March 30, 2007 2:27 PM

Chapter seven:

Break-even analysis

117

Substituting values, the equation becomes:

( ) ( ) −15, 000 ( 0.40211) − 48,000 + 3, 000 ( 0.30211) = −20X + 4X

−15, 000 A / P, 10%, 3 − 48,000 + 3, 000 A / F , 10%, 3 − 4X = −20X

−53, 125.32 = −16X ∴ X ≈ 3, 320 The preceding equation shows the break-even number of tests to be 3,320. If the hospital plans to perform less than 3,320 tests in a year (that is, X < 3,320), it will be better to outsource the procedures; but if X > 3,320, it will be better to perform the procedures in-house. When X = 3,320, it does not matter whether the procedure is performed in-house or outsourced.

7.2 Profit ratio analysis Break-even charts offer the opportunity for several different types of analysis. In addition to the break-even points, other measures of worth, or criterion measures, may be derived from the charts. For example, a measure called the profit ratio permits one to obtain a further comparative basis for competing projects. The profit ratio is defined as the ratio of the profit area to the sum of the profit-and-loss areas in a break-even chart. That is, Profit ratio =

Area of profit region Area of profit region + Area of loss region

Example 7.3 Suppose that the expected revenue and the expected total costs associated with a project are given, respectively, by the following expressions: R(x) = 100 + 10x TC(x) = 2.5x + 250 where x is the number of units produced and sold from the project. In Figure 7.5, we can see that the break-even chart shows the break-even point to be 20 units. Net profits will be realized from the project if more than 20 units are produced, and net losses will result if fewer than 20 units are produced. It should be noted that the revenue function in Figure 7.5 represents an unusual case in which a revenue of $100 is realized even when zero units are produced. Suppose it is desired to calculate the profit ratio for this project if the number of units that can be produced is limited to between 0 and 100 units.

7477_book.fm Page 118 Friday, March 30, 2007 2:27 PM

118

Computational Economic Analysis for Engineering and Industry $ 1100 R(x)

TC(x) Profit area

300 250

100 0

20

100

Units

Figure 7.5 Area of profit vs. area of loss.

From Figure 7.5, the surface areas of the profit region and the loss region can both be calculated by using the standard formula for finding the area of a triangle, Area = (1/2)(Base)(Height). Using this formula, we have the following:

(

=

)(

1 Base Height 2

Area of profit region =

(

)

)(

1 1100 − 500 100 − 20 2

= 24, 000 square units

(

)(

Area of loss region =

1 Base Height 2

=

1 250 − 10 00 20 2

(

)

)( )

= 1500 square units Thus, the profit ratio is computed as Profit Ratio =

24000 24000 + 1500

= 0.9411 = 94.11%

)

7477_book.fm Page 119 Friday, March 30, 2007 2:27 PM

Chapter seven:

Break-even analysis

119

$ TC(x) 12000 10500 10000 8000 Area 3 (loss)

6390 6000 4000

R(x) Area 1 (loss)

Area 2 (profit)

2000 511 500 0

0 3.3

20

40

60

76.8 80

100

x (Units)

Figure 7.6 Break-even chart for revenue and cost functions.

The profit ratio may be used as a criterion for selecting among project alternatives. Note that the profit ratios for all the alternatives must be calculated over the same values of the independent variable. The project with the highest profit ratio will be selected as the desired project. For example, Figure 7.6 presents the break-even chart for an alternate project, say Project II. It is seen that both the revenue and cost functions for the project are nonlinear. The revenue and cost are defined as follows: R(x) = 160x – x2 TC(x) = 500 + x2 If the cost and/or revenue functions for a project are not linear, the areas bounded by the functions may not be easy to determine. For those cases, it may be necessary to use techniques such as definite integrals to find the areas. Figure 7.6 indicates that the project generates a loss if less than 3.3 units (3 actual units) or more than 76.8 units (76 actual units) are produced. The respective profit and loss areas on the chart are calculated as follows:

∫ ( 500 + x ) − (160x − x ) dx 3.3

Area 1 (loss) =

2

0

= 802.8 unit-dollars

2

7477_book.fm Page 120 Friday, March 30, 2007 2:27 PM

120

Computational Economic Analysis for Engineering and Industry

∫ (160x − x ) − ( 500 + x ) dx 76.8

Area 2 (profit) =

2

2

3.3

= 132 , 272.08 unit-dollars

∫ ( 500 + x ) − (160x − x ) dx 100

Area 3 (loss) =

2

2

76.8

= 48, 135.98 unit-dollars Consequently, the profit ratio for Project II is computed as

Profit ratio = =

Total area of profit region To otal area of profit region + Total area of loss region 132 , 272.08 802.76 + 132 , 272.08 + 48, 135.98

= 72.99% The profit ratio approach evaluates the performance of each alternative over a specified range of operating levels. Most of the existing evaluation methods use single-point analysis with the assumption that the operating condition is fixed at a given production level. The profit ratio measure allows an analyst to evaluate the net yield of an alternative, given that the production level may shift from one level to another. It is possible, for example, for an alternative to operate at a loss for most of its early life but for it to generate large incomes to offset the earlier losses in its later stages. Conventional methods cannot easily capture this type of transition from one performance level to another. In addition to being used to compare alternate projects, the profit ratio may also be used for evaluating the economic feasibility of a single project. In such a case, a decision rule may be developed. An example of such a decision rule follows: If a profit ratio is greater than 75%, accept the project. If a profit ratio is less than or equal to 75%, reject the project.

Practice problems: Break-even analysis 7.1 Considering Example 7.2, if the RAB General Hospital anticipates that the number of tests to be conducted in a year varies between 500 and 5000, which option should be selected (in-house or outsource) based on the profit ratio analysis? 7.2 The capacity of a gear-producing plant is 4,600 gears per month. The fixed cost is $704,000 per month. The variable cost is $130 per gear, and the selling price is $158 per gear. Assuming that all products

7477_book.fm Page 121 Friday, March 30, 2007 2:27 PM

Chapter seven:

Break-even analysis

121

produced are sold, what is the break-even point in number of gears per month? What percent increase in the break-even point will occur if the fixed costs are increased by 20% and unit variable costs are increased by 4%? 7.3 A manufacturing firm produces and sells three different types of tires — car tires, van tires, and bus tires. Due to warehouse space constraints, the firm’s production is limited to 20,000 car tires, 16,000 van tires, and 24,000 bus tires. The variable and fixed costs associated with each type of tire are as follows: Tire Type Car Van Bus

Cost Type Variable Cost Fixed Cost $17.00 $25.00 $32.00

$800,000/month $1,200,000/month $1,600,000/month

If the selling price of a car, van, and bus tire is $100, $150, and $200 each, respectively, determine the break-even point for each type of tire. 7.4 A company is considering establishing a warehouse in either the southern or eastern part of the city where there is a potential market for their products. The management of the company needs to decide where to set up the warehouse by comparing the expenses that will be incurred if the warehouse is set up in either of the locations. Given the following data, compare the two sites in terms of their fixed, variable, and total costs. Which is the better location? Cost Factor Cost of land Estimated cost of building materials Number of laborers required Hourly payment of laborers Estimated miscellaneous expenses

South

East

$1.2 million $600,000 80 $12/h $50,000

$1.4 million $450,000 60 $10/h $62,000

7.5 A company produces a brand of sugar that is used for baking by several retail confectioneries. The fixed cost is $13,000 per month, and the variable cost is $4 per unit. The selling price per unit p = $45 – 0.01D. For this scenario, a. Determine the optimal volume of this brand of sugar and confirm that a profit occurs instead of a loss at this demand. b. Find the volume at which break-even occurs, that is, what is the domain of profitable demand?

7477_book.fm Page 122 Friday, March 30, 2007 2:27 PM

7477_book.fm Page 123 Friday, March 30, 2007 2:27 PM

chapter eight

Effects of inflation and taxes The effect of inflation is an important consideration in financial and economic analysis of projects. Inflation can be defined as a decline in the purchasing power of money, and multiyear projects are particularly subject to its effects. Some of the most common causes of inflation include the following: • • • •

An increase in the amount of currency in circulation A shortage of consumer goods An escalation of the cost of production An arbitrary increase of prices by resellers

The general effects of inflation are felt as an increase in the price of goods and a decrease in the worth of currency. In cash-flow analysis, return on investment (ROI) for a project will be affected by the time value of money as well as by inflation. The real interest rate (d) is defined as the desired rate of return in the absence of inflation. When we talk of “today’s dollars” or “constant dollars,” we are referring to the use of the real interest rate. The combined interest rate (i) is the rate of return combining the real interest rate and the rate of inflation. If we denote the inflation rate as j, then the relationship between the different rates can be expressed as follows: 1 + i = (1 + d)(1 + j) Thus, the combined interest rate can be expressed as i = d + j + dj Note that if j = 0 (i.e., there is no inflation), then i = d. We can also define the commodity escalation rate (g) as the rate at which individual commodity prices escalate. This may be greater or less than the overall inflation rate. In practice, several measures are used to convey inflationary effects. Some of these are the Consumer Price Index, the Producer Price Index, and the Wholesale Price Index. A “market basket” rate is defined as the estimate of inflation

123

7477_book.fm Page 124 Friday, March 30, 2007 2:27 PM

124

Computational Economic Analysis for Engineering and Industry Constant–worth cash flow

Then–current cash flow Tk

Ck = T0

T0

0

1

k

T0

n

0

1

k

n

Figure 8.1 Cash flows for effects of inflation.

based on a weighted average of the annual rates of change in the costs of a wide range of representative commodities. A “then-current” cash flow is a cash flow that explicitly incorporates the impact of inflation. A “constant-worth” cash flow is a cash flow that does not incorporate the effect of inflation. The real interest rate, d, is used for analyzing constant-worth cash flows. Figure 8.1 shows constant-worth and then-current cash flows. The then-current cash flow in the figure is the equivalent cash flow considering the effect of inflation. Ck is what it would take to buy a certain “basket” of goods after k time periods if there was no inflation. Tk is what it would take to buy the same “basket” in k time period if inflation were taken into account. For the constant worth cash flow, we have Ck = T0, k = 1, 2, …, n and for the then-current cash flow, we have Tk = T0 (1 + j )k, k = 1, 2, …, n where j is the inflation rate. If Ck = T0 = $100 under the constant worth cash flow, then we mean $100 worth of buying power. If we are using the commodity escalation rate, g, then we obtain Tk = T0 (1 + g)k, k = 1, 2, …, n Thus, a then-current cash flow may increase based on both a regular inflation rate ( j ) and a commodity escalation rate (g). We can convert a then-current cash flow to a constant-worth cash flow by using the following relationship: Ck = Tk(1 + j )– k, k = 1, 2, …, n If we substitute Tk from the commodity escalation cash flow into the preceding expression for Ck, we get the following:

7477_book.fm Page 125 Friday, March 30, 2007 2:27 PM

Chapter eight:

Effects of inflation and taxes

(

)

(

) (1 + j )

Ck = Tk 1 + j

= T 1+ g

(

125

−k

k

= T0  1 + g

−k

) (1 + j )

k

,

k = 1, 2 , … , n

Note that if g = 0 and j = 0, then Ck = T0. That is, there is no inflationary effect. We now define effective commodity escalation rate (v) as v = [(1 + g)/(1 + j)] – 1 and we can express the commodity escalation rate (g) as g = v + j + vj Inflation can have a significant impact on the financial and economic aspects of a project. In economic terms, inflation can be seen as the increase in the amount of currency in circulation such that there results a relatively high and sudden fall in its value. To a producer, inflation means a sudden increase in the cost of items that serve as inputs for the production process (equipment, labor, materials, etc). To the retailer, inflation implies an imposed higher cost of finished products. To an ordinary citizen, inflation portends an unbearable escalation of prices of consumer goods. All these aspects of inflation are interrelated in a project-management environment. The amount of money supply, as a measure of a country’s wealth, is controlled by that country’s government. For various reasons, governments often feel impelled to create more money or credit to take care of old debts and pay for social programs. When money is generated at a faster rate than the growth of goods and services, it becomes a surplus commodity, and its value (purchasing power) will fall. This means that there will be too much money available to buy only a few goods and services. When the purchasing power of a currency falls, each individual in a product’s life cycle has to dispense more of the currency in order to obtain the product. Some of the classic concepts of inflation are discussed here. 1. Increases in producers’ costs are passed on to consumers. At each stage of a product’s journey from producer to consumer, prices are escalated disproportionately in order to make a good profit. The overall increase in the product’s price is directly proportional to the number of intermediaries it encounters on its way to the consumer. This type of inflation is called cost-driven (or cost-push) inflation. 2. Excessive spending power of consumers forces an upward trend in prices. This high spending power is usually achieved at the expense

7477_book.fm Page 126 Friday, March 30, 2007 2:27 PM

126

Computational Economic Analysis for Engineering and Industry

3.

4.

5.

6.

of savings. The law of supply and demand dictates that the higher the demand, the higher the price. This type of inflation is known as demand-driven (or demand-pull) inflation. The impact of international economic forces can induce inflation in a local economy. Trade imbalances and fluctuations in currency values are notable examples of international inflationary factors. Increasing base wages of workers generate more disposable income and, hence, higher demands for goods and services. The high demand, consequently, creates a pull on prices. Coupled with this, employers pass on the additional wage cost to consumers through higher prices. This type of inflation is, perhaps, the most difficult to solve because union-set wages and producer-set prices almost never fall, at least not permanently. This type of inflation may be referred to as wage-driven (or wage-push) inflation. Easy availability of credit leads consumers to “buy now and pay later” and, thereby, creates another loophole for inflation. This is a dangerous type of inflation because the credit not only pushes prices up but also leaves consumers with less money later on to pay for the credit. Eventually, many credits become uncollectible debts, which may then drive the economy into recession. Deficit spending results in an increase in money supply and, thereby, creates less room for each dollar to get around. The popular saying, “a dollar doesn’t go as far as it used to,” simply refers to inflation in layman’s terms. The different levels of inflation may be categorized in the following way.

8.1 Mild inflation When inflation is mild (2 to 4%), the economy actually prospers. Producers strive to produce at full capacity in order to take advantage of the high prices to the consumer. Private investments tend to be brisk, and more jobs become available. However, the good fortune may only be temporary. Prompted by the prevailing success, employers become tempted to seek larger profits, and workers begin to ask for higher wages. They cite their employers’ prosperous business as a reason to bargain for bigger shares of the business profit. So, we end up with a vicious cycle in which the producer asks for higher prices, the unions ask for higher wages, higher wages necessitate higher prices, and inflation starts an upward trend.

8.2 Moderate inflation Moderate inflation occurs when prices increase at 5 to 9%. Consumers start purchasing more as an edge against inflation. They would rather spend their money now than watch it decline further in purchasing power. The increased market activity serves to fuel further inflation.

7477_book.fm Page 127 Friday, March 30, 2007 2:27 PM

Chapter eight:

Effects of inflation and taxes

127

8.3 Severe inflation Severe inflation is indicated by price escalations of 10% or more. Double-digit inflation implies that prices rise much faster than wages do. Debtors tend to be the ones who benefit from this level of inflation because they repay debts with money that is less valuable than the money that was borrowed.

8.4 Hyperinflation When each price increase signals another increase in wages and costs, which again sends prices further up, the economy has reached a stage of malignant galloping inflation or hyperinflation. Rapid and uncontrollable inflation will destroy an economy. Currency becomes economically useless as the government prints it excessively to pay for obligations. Inflation can affect any project in terms of raw materials procurement, salaries and wages, and/or difficulties with cost tracking. Some effects are immediate and easily observable, others subtle and pervasive. Whatever form it takes, inflation must be taken into account in long-term project planning and control. Large projects may be adversely affected by the effects of inflation in terms of cost overruns and poor resource utilization. The level of inflation will determine the severity of the impact on projects. Example 8.1 An Internet company is considering investing in a new technology that would place its business above its competitors at the end of 3 years, when the project would be completed. Two of the company’s top suppliers have been contacted to develop the technology. The yearly project costs of the new technology for each of the suppliers are as follows. If the Internet company uses an MARR of 15%, and if the general price inflation is assumed to be 2.5% per year over the next 3 years, which supplier should be given the contract? Year

Supplier A (All values in Future Dollars)

Supplier B (All Values in Today’s Dollars)

1 2 3

120,000 132,000 145,200

120,000 120,000 120,000

Solution It must be noted that Supplier A presents all its costs in terms of future dollars, and Supplier B presents its in terms of today’s dollars. To answer this question, we can either convert future dollars to today’s dollars or convert today’s dollars to future dollars. If the calculation is done properly, we should get the same results. From the question, j = 0.025 and i = 0.15.

7477_book.fm Page 128 Friday, March 30, 2007 2:27 PM

128

Computational Economic Analysis for Engineering and Industry

Approach I: Convert today’s dollars to future dollars and find the present value (PV) using i (the combined interest rate or MARR). Using the equation Future dollar = Today’s dollar (1 + j)k ≡ Tk = T0(1 + j)k, the equivalent future dollars for Supplier B are

(

)

(

)

(

)

1

Year 1: 120 , 000 × 1 + 0.025 = 123 , 000 Year 2: 120 , 000 × 1 + 0.025 Year 3: 120 , 000 × 1 + 0.025

2

3

= 126 , 075 = 129 , 227

Finding the present value,

(

)

(

)

(

)

)

(

)

(

)

PVA = 120, 000 P/F , 15%, 1 + 132 , 000 P/F , 15%, 2 + 145, 200 P/F , 15%, 3 = $299, 615

(

PVB = 123, 000 P/F , 15%, 1 + 126, 075 P/F , 15%, 2 + 129, 227 P/F , 15%, 3 = $287 , 246

Therefore, Supplier B should be selected because its PV is the lesser of the two. Approach II: Convert future dollars to today’s dollars and find the PV using d (the real interest rate): d=

i − j 0.15 − 0.025 = = 12.2% 1+ j 1 + 0.025

Using the equation Today’s dollar = Future dollar (1 + j)–k ≡ T0 = Tk (1 + j) , the equivalent today’s dollars for Supplier A are –k

(

)

−1

(

)

−2

(

)

Year 1: 120 , 000 × 1 + 0.025 Year 2: 132 , 000 × 1 + 0.025 Year 3: 145 , 200 × 1 + 0.025

−3

= 117 , 073 = 125 , 640 = 134 , 833

7477_book.fm Page 129 Friday, March 30, 2007 2:27 PM

Chapter eight:

Effects of inflation and taxes

129

Finding the present value,

(

)

(

)

(

PVA = 117 , 073 P/F , 12.2%, 1 + 125, 640 P/F , 12.2%, 2 + 134, 833 P/F , 12.2%, 3 = $299, 605

(

PVB = 120, 000 P/A, 12.2%, 3

)

= $287 , 232 Again, Supplier B should be selected because its PV is the lesser. The difference in PV is due to round-off errors.

8.5 Foreign-exchange rates The idea of accounting for the effects of inflation on local investments can be extended to account for the effects of devalued currency on foreign investments. When local businesses invest in a foreign country, several factors come into consideration, such as when the initial investment is made and when the benefits are returned to the local business. As a result of changes in international businesses, exchange rates between currencies fluctuate, and countries continually devalue their currencies to satisfy international trade agreements. Using the U.S. as the base country where the local business is situated, let ius = rate of return in terms of a market interest rate relative to U.S. dollars. ifc = rate of return in terms of a market interest rate relative to the currency of a foreign country. fe = annual devaluation rate between the currency of a foreign country and the U.S. dollar. A positive value means that the foreign currency is being devalued relative to the U.S. dollar. A negative value means that the U.S. dollar is being devalued relative to the foreign currency. Therefore, the rate of return of the local business with respect to a foreign country can be given by the following: ius =

i fc − fe 1 + fe

( )

(8.1)

i fc = ius + fe + fe ius Example 8.2

A European company is considering investing in a technology in the U.S. The company has a MARR of 20% on investments in Europe. Additionally,

)

7477_book.fm Page 130 Friday, March 30, 2007 2:27 PM

130

Computational Economic Analysis for Engineering and Industry

its currency, the RAW, is very strong, with the U.S. dollars being devalued an average of 4% annually with respect to it. a. Based on a before-tax analysis, would the European company want to invest in a technology proposal in the U.S. for $10 million that would repay $2.5 million each year for 10 years with no salvage value? b. If a U.S. company with a MARR of 20% wanted to invest in this European country, what MARR should it expect on cash flows in RAWs? Solution a. From the question, for the European company,

(

fe = −4% because the U.S. dollar is being devallued

)

i fc = 20% n = 10 iUS =

(

)

i fc − fe 0.2 − −0.04 = = 0.25 1 + fe 1 + −0.04

(

)

(

)

PViUS = 2 , 500, 000 P/A, 25%, 10 = $8, 926, 258 Because the PV of the benefits of the investment is less than the amount invested, the European company would not want to invest in this proposal. b. For the U.S. company,

( ) = 0.20 + ( −0.04 ) + ( −0.04 ) ( 0.20 )

i fc = iUS + fe + fe iUS

= 15.2% Therefore, the U.S. company should not expect a MARR of more than 15.2% in RAWs.

8.6 After-tax economic analysis For complete and accurate economic analysis results, both the effects of inflation and taxes must be taken into consideration, especially when alternatives are being evaluated. Taxes are an inevitable burden such that their

7477_book.fm Page 131 Friday, March 30, 2007 2:27 PM

Chapter eight:

Effects of inflation and taxes

131

effects must be accounted for in economic analysis. There are several types of taxes: • Income taxes: These are taxes assessed as a function of gross revenue less allowable deductions, and are levied by the federal government, and most state and municipal governments. • Property taxes: They are assessed as a function of the value of property owned, such as land, buildings, and equipment. They are mostly levied by municipal, county, or state governments. • Sales taxes: These are assessed on purchases of goods and services; hence, sales taxes are independent of gross income or profits. They are normally levied by state, municipal, or county governments. Sales taxes are relevant in economic analysis only to the extent that they add to the cost of items purchased. • Excise taxes: These are federal taxes assessed as a function of the sale of certain goods or services often considered nonnecessities. They are usually charged to the manufacturer of the goods and services, but a portion of the cost is passed on to the purchaser. Income taxes are the most significant type of tax encountered in economic analysis; therefore, the effects of income taxes can be accounted for using these relations. Let TI = Taxable income (amount upon which taxes are based). T = Tax rate (percentage of taxable income owed in taxes). NPAT = Net Profit after Taxes (taxable income less income taxes each year. This amount is returned to the company). Therefore, TI = gross income − expenses − depreciation (depletion) deductions T = TI × Applicable tax rate

(

NPAT = TI 1 − T

(8.1)

)

The tax rate used in economic analysis is usually the effective tax rate, and it is computed using this relationship: Effective tax rates (Te) = state rate + (1 − state rate)(federal rate)

(8.2)

Therefore,

(

)( )

T = Taxable Income Te

(8.3)

7477_book.fm Page 132 Friday, March 30, 2007 2:27 PM

132

Computational Economic Analysis for Engineering and Industry

8.7 Before-tax and after-tax cash flow The only difference between a before-tax cash flow (BTCF) and an after-tax cash flow (ATCF) is that ATCF includes expenses (or savings) due to income taxes and uses an after-tax MARR to calculate equivalent worth. Hence, after-tax cash flow is the before-tax cash flow less taxes. The after-tax MARR is usually smaller than the before-tax MARR, and they are related by the following equation:

(

)(

After-tax MARR ≅ Before-tax MARR 1 − Te

)

(8.4)

8.8 Effects of taxes on capital gain Capital gain is the amount incurred when the selling price of a property exceeds its first cost. Because future capital gains are difficult to estimate, they are not detailed in after-tax study. However in actual tax law, there is no difference between a short-term or long-term gain. Capital loss is the loss incurred when a depreciable asset is disposed of for less than its current book value. An economic analysis does not usually account for capital loss because it is not easily estimated for alternatives. However, after-tax replacement analysis should account for any capital loss. For economic analysis, this loss provides a tax saving in the year of replacement. Depreciation recapture occurs when a depreciable asset is sold for more than its current book value. Therefore, depreciation recapture is the selling price less the book value. This is often present in after-tax analysis. When the MACRS system of depreciation is used, the estimated salvage value of an asset can be anticipated as the depreciation recapture because MACRS assumes zero salvage value. Therefore, the taxable income equation can be rewritten as TI = gross income − expenses − depreciation (depletion) deductions + depreciation recapture + capital gain − capital loss

(8.5)

8.9 After-tax computations The ATCF estimates are used to compute the net present value (NPV) or net annual value (NAV) or net future value (NFV) at the after-tax MARR. The same logic applies as for the before-tax evaluation methods discussed in Chapter 5; however, the calculations required for after-tax computations are certainly more involved than those for before-tax analysis. The major elements in an after-tax economic analysis are these: • Before-tax cash flow • Depreciation • Taxable income

7477_book.fm Page 133 Friday, March 30, 2007 2:27 PM

Chapter eight:

Effects of inflation and taxes

133

• Income taxes • After-tax cash flow The incorporation of tax and depreciation aspects into the before-tax cash flow produces the after-tax cash flow, upon which all the computational procedures presented earlier can be applied to obtain measures of economic worth. The measures emanating from the after-tax cash flow are referred to as after-tax measures of economic worth.

Practice problems: Effects of inflation and taxes 8.1 You are considering buying some equipment to recover materials from an effluent at your chemical plant. These chemicals will have a market value of $157,000 next year and should increase in value at an inflation rate of 7.5% each year. Your company is using a planning period of 7 years for this type of project and a MARR of 15%. How much can you afford to pay for the recovery equipment? Ignore the effect of taxes. 8.2 On January 1, 1995, the National Price Index was 208.5, and on January 1, 2005, it was 516.71. What was the inflation rate compounded annually over that 10-year period? 8.3 A manufacturing machine cost $65,000 in 1990, and an equivalent model 5 years later cost $98,600. If inflation is considered the cause of the increase, what was the average annual rate of inflation? 8.4 A project has the following estimates in future dollars. If the inflation rate is 7%, what are the equivalent estimates in today’s dollars? EOY

Future Dollars

0 1 2 3 4 5 6 7 8 9 10

100,000 22,300 24,851 27,680 30,815 34,290 38,140 42,404 47,126 52,354 78,141

Today’s Dollars

8.5 (a) What is the rate of return of the project described in Problem 8.4 in terms of future dollars and in terms of today’s dollars? Discuss the difference between these values. (b) What is the discounted payback period of the project described in Problem 8.4 in terms of future dollars and in terms of today’s dollars if the MARR is as computed in Problem 8.5 (a)?

7477_book.fm Page 134 Friday, March 30, 2007 2:27 PM

7477_book.fm Page 135 Tuesday, March 13, 2007 3:34 PM

chapter nine

Advanced cash-flow analysis techniques In some cases, an analyst must resort to more advanced or intricate economic analysis in order to take into account the unique characteristics of a project. This chapter presents a collection of such techniques, which are based on an application of the computational processes presented in earlier chapters. Computation of amortization of capitals, tent cash-flow analysis, and equity break-even point calculation are presented in this chapter as additional techniques of economic analysis.

9.1 Amortization of capitals Many capital investment projects are financed with external funds repaid according to an amortization schedule. A careful analysis must be conducted to ensure that the company involved can financially handle the amortization schedule, and here, a computer program such as GAMPS (Graphic Evaluation of Amortization Payments) might be useful for this purpose. Such a program analyzes installment payments, the unpaid balance, principal amount paid per period, total installment payment, and current cumulative equity. It also calculates the “equity break-even point” for the debt being analyzed. The equity break-even point indicates the time when the unpaid balance on a loan is equal to the cumulative equity on the loan. This is discussed in a later section of this chapter. With this or a similar program, the basic cost of servicing the project debt can be evaluated quickly. A part of the output of the program presents the percentage of the installment payment going into the equity and the interest charge, respectively. The computational procedure for analyzing project debt follows the steps outlined here: 1. Given a principal amount, P, a periodic interest rate, i (in decimals), and a discrete time span of n periods, the uniform series of equal end-of-period payments needed to amortize P is computed as 135

7477_book.fm Page 136 Tuesday, March 13, 2007 3:34 PM

136

Computational Economic Analysis for Engineering and Industry

(

)

n P i 1 + i     A= 1+ i −1

(

)

It is assumed that the loan is to be repaid in equal monthly payments. Thus, A(t) = A for each period t throughout the life of the loan. 2. The unpaid balance after making t installment payments is given by

(

A 1 − 1 + i U t =  i

()

)(

t− n

) 

3. The amount of equity or principal amount paid with installment payment number t is given by

()

(

E t = A 1+ i

)

t − n− 1

4. The amount of interest charge contained in installment payment number t is derived to be

()

(

I t = A 1 − 1 + i 

)

t − n− 1

 

where A = E(t) + I(t). 5. The cumulative total payment made after t periods is denoted by t

()

C t =

∑ A( k ) k =1 t

=

∑A k =1

( )( )

= A t

6. The cumulative interest payment after t periods is given by

()

Q t =

t

∑ I ( x) x =1

7477_book.fm Page 137 Tuesday, March 13, 2007 3:34 PM

Chapter nine:

Advanced cash-flow analysis techniques

137

7. The cumulative principal payment after t periods is computed as t

()

S t =

∑ E(k) k =1

t

∑ (1 + i ) (

)

− n− k + 1

=A

k =1

(

)

 1+ i t − 1  = A  i 1+ i n   

(

)

where t

∑x

n

n= 1

=

x x +1 − x x−1

8. The percentage of interest charge contained in installment payment number t is I t ( ) A( ) (100%)

f t =

9. The percentage of cumulative interest charge contained in the cumulative total payment up to and including payment number t is

()

F t =

( ) (100%) C (t )

Q t

10. The percentage of cumulative principal payment contained in the cumulative total payment up to and including payment number t is

()

() () C (t ) − Q (t ) = C (t ) Q (t ) = 1− C (t ) = 1 − F (t )

H t =

S t

C t

7477_book.fm Page 138 Tuesday, March 13, 2007 3:34 PM

138

Computational Economic Analysis for Engineering and Industry Table 9.1 Amortization Schedule for Financed Project

Example 9.1 Suppose a manufacturing productivity improvement project is to be financed by borrowing $500,000 from an industrial development bank. The annual nominal interest rate for the loan is 10%. The loan is to be repaid in equal monthly installments over a period of 15 years. The first payment on the loan is to be made exactly one month after financing is approved. A detailed analysis of the loan schedule is desired. Table 9.1 presents a partial listing of the loan repayment schedule. These figures are computed using the ENGINEA software, which is described in Chapter 12. The tabulated result shows a monthly payment of $5,373.03 on the loan. If time t = 10 months, one can see the following results: U(10) = $487,473.83 (unpaid balance) A(10) = $5,373.03 (monthly payment) E(10) = $1,299.91 (equity portion of the tenth payment) I(10) = $4,073.11 (interest charge contained in the tenth payment) C(10) = $53,730.26 (total payment to date)

7477_book.fm Page 139 Tuesday, March 13, 2007 3:34 PM

Chapter nine:

Advanced cash-flow analysis techniques

139

Plot of unpaid balance and cumulative equity

$500000

U(t)

$400000

ebp

$300000 $200000 $100000 $0

S(t) 0

120 Months

180

Figure 9.1 Plot of unpaid balance and cumulative equity.

S(10) = $12,526.17 (total equity to date) f (10) = 75.81% (percentage of the tenth payment going into interest charge) F(10) = 76.69% (percentage of the total payment going into interest charge) Thus, over 76% of the sum of the first 10 installment payments goes into interest charges. The analysis shows that by time t = 180, the unpaid balance has been reduced to zero. That is, U(180) = 0.0. The total payment made on the loan is $967,144.61 and the total interest charge is $967,144.61 – $500,000 = $467,144.61. So, 48.30% of the total payment goes into interest charges. The information about interest charges might be very useful for tax purposes. The tabulated output shows that equity builds up slowly, whereas the unpaid balance decreases slowly. Note that very little equity is accumulated during the first 3 years of the loan schedule. This is shown graphically in Figure 9.1 (also constructed using the ENGINEA software). The effects of inflation, depreciation, property appreciation, and other economic factors are not included in the analysis just presented, but a project analysis should include such factors whenever they are relevant to the loan situation.

9.1.1

Equity break-even point

The point at which the curves intersect is referred to as the equity break-even point. It indicates when the unpaid balance is exactly equal to the accumulated equity or the cumulative principal payment. For the preceding example, the equity break-even point is approximately 120 months (10 years). The

7477_book.fm Page 140 Tuesday, March 13, 2007 3:34 PM

140

Computational Economic Analysis for Engineering and Industry

importance of the equity break-even point is that any equity accumulated after that point represents the amount of ownership or equity that the debtor is entitled to after the unpaid balance on the loan is settled with project collateral. The implication of this is very important, particularly in the case of mortgage loans. “Mortgage” is a word of French origin, meaning “death pledge,” which, perhaps, is an ironic reference to the burden of mortgage loans. The equity break-even point can be calculated directly from the formula derived in the following text: Let the equity break-even point, x, be defined as the point where U(x) = S(x). That is,  1 − 1 + i − ( n− x )   1+ i x − 1    A = A    i 1+ i n  i    

(

)

(

)

(

)

Multiplying both the numerator and denominator of the left-hand side of this expression by (1+i)n and then simplifying yields

(1 + i ) − (1 + i ) i (1 + i ) n

x

n

on the left-hand side. Consequently, we have

(1 + i ) − (1 + i ) = (1 + i ) n

x

1+ i (1 + i ) = ( 2)

n

x

x

−1

+1

which yields the equity break-even expression

(

)

n ln  0.5 1 + i + 0.5     x= ln 1 + i

(

)

where ln = the natural log function n = the number of periods in the life of the loan i = the interest rate per period Figure 9.2 presents a plot of the total loan payment and the cumulative equity with respect to time. The total payment starts from $0.0 at time 0 and

7477_book.fm Page 141 Tuesday, March 13, 2007 3:34 PM

Chapter nine:

Advanced cash-flow analysis techniques

141

$1,000,000 900,000 800,000 700,000

C(t)

600,000 500,000 400,000 300,000 200,000

S(t)

100,000 0

0

108 Months

Figure 9.2 Plot of total loan payment and total equity.

goes up to $967,144.61 by the end of the last month of the installment payments. Because only $500,000 was borrowed, the total interest payment on the loan is $967,144.61 – $500,000 = $467,144.61. The cumulative principal payment starts at $0.0 at time 0 and slowly builds up to $500,000, which is the original loan amount. Figure 9.3 presents a plot of the percentage of interest charge in the monthly payments and the percentage of interest charge in the total payment. The percentage of interest charge in the monthly payments starts at 77.55% for the first month and decreases to 0.83% for the last month. By comparison, the percentage of interest in the total payment starts also at 77.55% for the first month and slowly decreases to 48.30% by the time the last payment is made at time 180. Table 9.1 and Figure 9.3 show that an increasing proportion 100 90 80 F(t)

Percentage

70 60 50 40 30

F(t)

20 10 0

0

Months

Figure 9.3 Plot of percentage of interest charge.

180

7477_book.fm Page 142 Tuesday, March 13, 2007 3:34 PM

142

Computational Economic Analysis for Engineering and Industry

of the monthly payment goes into the principal payment as time goes on. If the interest charges are tax deductible, the decreasing values of f (t) mean that there would be decreasing tax benefits from the interest charges in the later months of the loan.

9.2 Introduction to tent cash-flow analysis Analysis of arithmetic gradient series (AGS) cash flows is one of the more convoluted problems in engineering economic analysis. This chapter presents an interesting graphical representation of AGS cash flows and the approach is in the familiar shape of tents. Several designs as well as a closed-form analysis of AGS cash-flow profiles are presented for a better understanding of this and other cash-flow profiles and their analysis. A general tent equation (GTE) that can be used to solve various AGS cash flows is developed. The general equation can be used for various tent structures with the appropriate manipulations. GTE eliminates the problem of using the wrong number of periods and is amenable to software implementation.

9.3 Special application of AGS AGS cash flows feature prominently in many contract payments, but misinterpretation of them can seriously distort the financial reality of a situation. Good examples can be found in the contracts of sports professionals. The pervasiveness of, and extensive publicity attending, such contracts make the analysis of arithmetic gradient series both appealing and economically necessary. A good example is the 1984 contract of Steve Young, a quarterback for the LA Express team in the former USFL (United States Football League). The contract was widely reported as being worth $40 million at that time. The cash-flow profile of the contract revealed an intricate use of various segments of arithmetic gradient series cash flows. When everything was taken into account, the $40 million touted in the press amounted only to a present worth of the contract of about $5 million at that time. The trick was that the club included some deferred payments stretching over 37 years (1990–2027) at a 1984 present cost of only $2.9 million. The deferred payments were reported as being worth $34 million, which was the raw sum of the amounts in the deferred cash-flow profile. Thus, it turns out that clever manipulation of an AGS cash flow can create unfounded perceptions of the worth of a professional sports contract. This may explain why some sports professionals end up almost bankrupt even after receiving what they assume to be multimillion-dollar contracts. Similar examples have been found in reviewing the contracts of other sports professionals. Other real-life examples of gradient cash-flow constructions can be found in general investment cash flows and maintenance operations. In a particular economic analysis of maintenance operations, an escalation factor was applied to the annual cost of maintaining a major piece of equipment.

7477_book.fm Page 143 Tuesday, March 13, 2007 3:34 PM

Chapter nine:

Advanced cash-flow analysis techniques

143

The escalation is justified because of inflation, increased labor costs, and other forecasted needs.

9.4 Design and analysis of tent cash-flow profiles AGS cash flows usually start with some base amount at the end of the first period and then increase or decrease by a constant amount thereafter. The nonzero base amount is denoted as AT starting at period T. The analysis of the present worth for such cash flows requires breaking the cash flow into a uniform series cash flow of amount AT starting at period T and an AGS cash flow with a zero base amount. The uniform series present worth formula is used to calculate the present worth of the uniform series portion, whereas the basic AGS formula is used to calculate the arithmetic gradient series part of the cash-flow profile. The overall present worth is then calculated as follows: P = Puniform series ± Parithmetic gradient series Figure 9.4 presents a conventional AGS cash flow. Each cash-flow amount at time t is defined as At = (t–1). The standard formula for this basic AGS profile is derived as follows: n

P=

∑ A (1 + i)

−t

t

t =1 n

=

∑ (t − 1) G (1 + i)

−t

t =1

n

= G

∑ (t − 1) (1 + i)

−t

t =1

= 

(

) ( )  ( )  = G ( P/G, i, n) , in tabulated form  1 + i n − 1 + ni = G n  i2 1 + i 

The computational process of deriving and using the P/G formula is where new engineering economy students often stumble. Recognizing the tent-like structure of the cash flow, and the fact that several applications of AGS are beyond its conventional format, leads to the idea of tent cash flows. Figure 9.5 presents the basic tent (BT) cash-flow profile. It is composed of

7477_book.fm Page 144 Tuesday, March 13, 2007 3:34 PM

144

Computational Economic Analysis for Engineering and Industry (n–1)G 2G G 0

0

…….

1

2

t

3

n

P

Figure 9.4 Conventional Arithmetic Gradient Series (AGS) cash flow.

A … 1

… T

t

N

P

Figure 9.5 Basic Tent (BT) cash-flow profile.

an up-slope gradient and a down-slope portion, both on a uniform series base of A = A1. Computational formula analysis of BT cash flow is shown here. For the first half of the cash flow (from t = 1 to t = T), T

T

Pa =



A1 (1 + i)− t +

t =1

∑ A (1 + i)

−t

t

t =1

where At = (t – 1)G. T

T

Pa =



∑ (t − 1)(1 + i)

A1 (1 + i)− t + G

t =1

−t

t =1

 (1 + i)T − (1 + Ti)   (1 + i)T − 1  Pa = A1  +G  T  i 2 (1 + i)T    i(1 + i)  For the second half of the cash flow (from t = T +1 to t = N), N

N

Pb =



t =T +1

where At = (t – 1)G.

AT +1 (1 + i)− t −

∑ A (1 + i) t

t =T +1

−t

(9.1)

7477_book.fm Page 145 Tuesday, March 13, 2007 3:34 PM

Chapter nine:

Advanced cash-flow analysis techniques N

N

Pb = AT +1

∑ (1 + i) − ∑ (t − 1)G(1 + i) −t

t =T +1

145

−t

t =T +1

: :  (1 + i)N −T − 1   (1 + i)N −T − [1 + ( N − T )i]  Pb = AT +1  −G  N −T  i 2 (1 + i)N −T  i(1 + i)    Let x = (N – T),  (1 + i)x − (1 + xi)   (1 + i)x − 1  ∴ Pb = AT +1  G −    x i 2 (1 + i)x    i(1 + i) 

(9.2)

P = Pa + Pb (1 + i)− T

(9.3)

Hence,

The basic approach to solving this type of cash-flow profile is to partition it into simpler forms. By partitioning, BT can be solved directly using standard cash-flow conversion factors. That is, the solution can be obtained as the sum of a uniform series cash flow (base amount), an increasing AGS, and a decreasing AGS. That is, P = G( P/A, i, N ) + G( P/G, i, T ) + G(T - 2)( P/A, i, N -T ) − G( P/G, i, N -T )  ( P/F , i, T ) The preceding two approaches should yield the same result. Obviously, the partitioning approach using existing standard factors is a more ingenious method. One common student error when using the existing AGS factor is the use of an incorrect number of periods, n. It should be recognized that the standard AGS factor was derived for a situation where P is located one period before the “nose” of the increasing series. Students often tend to locate P right at the same point on the time line as the “nose” of the series, which means that n will be off by one unit. One way to avoid this error is to redraw the time line and renumber it from a reference point of zero; that is, relocate time zero (t = 0) to one period before the AGS begins. Figure 9.6 shows a profile of the ET cash-flow profile. It has a constant amount of increasing and decreasing AGS. The magnitudes of the cash-flow amounts at times Tj (j =1, 2, …) are equal. For Part A: PA = P1 + PT1 +1 (1 + i)− T1

(9.4)

7477_book.fm Page 146 Tuesday, March 13, 2007 3:34 PM

146

Computational Economic Analysis for Engineering and Industry Part A

Part C

Part B

… 1

… T1

… N1

… T2

… N2

… T3

N3

t

P

Figure 9.6 Executive tent (ET) cash-flow profile.

where  (1 + i)T1 − (1 + T1i)   (1 + i)T1 − 1  P1 = A1  +G  T1  i 2 (1 + i)T1    i(1 + i) 

(9.5)

 (1 + i)x1 − 1   (1 + i)x1 − (1 + x1i)  PT1 +1 = AT1 +1  G −    x1 i 2 (1 + i)x1    i(1 + i) 

(9.6)

and

while x1 = (N1 – T1). For Part B: PB = PN1 +1 (1 + i)− N1 + PT2+1 (1 + i)− T2

(9.7)

 (1 + i)T2 − 1   (1 + i)T2 − (1 + T2i)  PN1 +1 = AN1 +1  +G  T2  i 2 (1 + i)T2    i(1 + i) 

(9.8)

 (1 + i)x2 − 1   (1 + i)x2 − (1 + x2i)  PT2+1 = AT2 +1  G −    x2 i 2 (1 + i)x2  i(1 + i)   

(9.9)

where

and

while x2 = (N2 – T2). For Part C: PC = PN2 +1 (1 + i)− N2 + PT3+1 (1 + i)− T3

(9.10)

7477_book.fm Page 147 Tuesday, March 13, 2007 3:34 PM

Chapter nine:

Advanced cash-flow analysis techniques

147

where  (1 + i)T3 − (1 + T3i)   (1 + i)T3 − 1  + PN2 +1 = AN2 +1  G    T3 i 2 (1 + i)T3    i(1 + i) 

(9.11)

 (1 + i)x3 − 1   (1 + i)x3 − (1 + x3i)  −G PT3+1 = AT3 +1   x3  i 2 (1 + i)x3   i(1 + i)  

(9.12)

and

while x3 = (N3 – T3). ∴ P = PA + PB + PC

(9.13)

Figure 9.7 presents a Saw-Tooth Tent (STT) cash-flow profile. The present value analysis of the cash flow is computed as follows: P = P1 + PT1 +1 (1 + i)− T1 + PT2 +1 (1 + i)− T2

(9.14)

 (1 + i)T1 − (1 + T1i)   (1 + i)T1 − 1  P1 = A1  G +    T1 i 2 (1 + i)T1    i(1 + i) 

(9.15)

  T2  (1 + i)T2 − 1   + G  (1 + i) − (1 + T2i)  PT1 +1 = AT1 +1  T    i(1 + i)T2  i 2 (1 + i) 2    

(9.16)

 (1 + i)N − 1   (1 + i)N − (1 + Ni)  PT2 +1 = AT2 +1  G +    N i 2 (1 + i)N  i(1 + i)   

(9.17)

where

….. 1

….. T1

….. T2

P

Figure 9.7 Saw-Tooth Tent (STT) cash-flow profile.

N

t

7477_book.fm Page 148 Tuesday, March 13, 2007 3:34 PM

148

Computational Economic Analysis for Engineering and Industry

….. 1

….. N1 T1

….. N2 T2

N

P

Figure 9.8 Reversed Saw-Tooth Tent (R-STT) cash-flow profile.

In Figure 9.8, a Reversed Saw-Tooth Tent (R-STT) is constructed. Its present value computation is handled as shown here. P = P1 + PT1 (1 + i)− T1 + PT2 (1 + i)− T2

(9.18)

 (1 + i)N1 − (1 + N1i)   (1 + i)N1 − 1  P1 = A1  −G  N1  i 2 (1 + i)N1    i(1 + i) 

(9.19)

 (1 + i)x1 − 1   (1 + i)x1 − (1 + x1i)  PT1 = AT1  G −    x1 i 2 (1 + i)x1  i(1 + i)   

(9.20)

where

while x1 = (N2 – N1).  (1 + i)x2 − (1 + x i)   (1 + i)x2 − 1  2   PT2 = AT2  G − x2  2 x2   1 1 + i ( + i ) i i ( )    

(9.21)

while x2 = (N – N2). Increasingly complicated profiles can be designed depending on the level of complexity desired to test different levels of student understanding. Note that the ET, STT, and R-STT cash flows can also be solved by the partitioning approach shown earlier for the BT cash flow.

9.5 Derivation of general tent equation To facilitate a less convoluted use of the tent cash-flow computations, this section presents a General Tent Equation (GTE) for AGS cash flows. This is suitable for adoption by engineering economy instructors or students. We combine all the tent cash-flow equations into a general tent cash-flow equation that is amenable to software implementation. The general equation is as follows:

7477_book.fm Page 149 Tuesday, March 13, 2007 3:34 PM

Chapter nine:

Advanced cash-flow analysis techniques

(

)

(

) ( )  ( ) 

 1+ i T − 1  1 + i T − 1 + Ti  + G1  P0 = A0 + A1  T  i 1+ i T   i2 1 + i   

(

(

)

)

(

) ( )  ( ) 

 1+ i x − 1  1 + i x − 1 + xi  + G2  PT = AT1  x  i 1+ i x   i2 1 + i   

(

)

149

(9.22)

(9.23)

when G1 ≥ 0 or G2 ≤ 0,

(

)

−T

(

)

−T

TPV0 = P0 + PT 1 + i

(9.24)

when G1 < 0 or G2 > 0, TPV0 = P0 − PT 1 + i

(9.25)

where TPV0 P0 PT i N T x A0 A1 AT1 G1

= = = = = = = = = = =

Total Present Value at time t = 0 Present Value for the first half of the tent at time t = 0 Present Value for the second half of the tent at time t = T interest rate in fractions number of periods the center time value of the tent (N – T) amount at time t = 0 amount at time t = 1 amount at time = T + 1 gradient series of the first half of the tent ⇒ increasing G1 (up-slope) is a positive value and decreasing G1 (down-slope) is a negative value G2 = gradient series of the first half of the tent ⇒ increasing G2 (up-slope) is a positive value and decreasing G2 (down-slope) is a negative value

This GTE is based on the BT but can be used for either one-sided (see Figure 9.1) or two-sided (see Figure 9.2) tents. It can also be used for Executive Tents (ET) with more than two cycles. The process involved in this case is to divide the tent into smaller sections of two cycles, such as Part A, Part B, and Part C (see Figure 9.3), and then to use the equation to solve for the PV of each part at time t = 0, t = N 1, and t = N 2. The PV of the future values at t = N 1 and t = N 2 are added to the PV at t = 0 to determine the

7477_book.fm Page 150 Tuesday, March 13, 2007 3:34 PM

150

Computational Economic Analysis for Engineering and Industry

overall PV of the ET at t = 0. This GTE eliminates the problem associated with having an incorrect number of periods and makes AGS analysis interesting. This equation can also be used for situations where the uniform series base of A1 is zero by finding the PV at time t = 1 and taking it back one step to determine the total present value at time t = 0. Other scenarios can also be considered by appropriate manipulations of the equation. Example 9.2 Using the GTE, find the PV of the multiple BTs (Figure 9.9) at time t = 0, where i = 10% per period. Solving this tent cash flow usually poses a lot of problems for students; however, the use of the GTE reduces such problems considerably. Because this tent consists of two BTs, we will use the general equation to determine the present value at time t = 0 for the first BT and the PV at time t = 11, we will then find the PV at time t = 0 for the second tent and sum it to the PV for the first tent and the PV at time t = 1. Using the GTE for the first basic tent, we obtained P0 = $101.17 for the first half of the first basic tent, and P5 = $82.24 for the second half of the tent: PV0 = $152.23. To solve the second BT, we renumbered the tent so that t = 11 becomes t = 0, t = 12 becomes t = 1, and so on. Therefore, t = T is at t = 7, and we obtained P11 = $88.16 for the first half of the second BT and P18 = $82.24 for the second half of the second BT: PV11 = $130.36. The total PV for the dual BT at time t = 0 is

(

TNPV0 = P−1 ( F/P, i%, N ) + P0 + P11 P/F , i%, N

)

(

)

TNPV0 = 5( F/P, 10%, 1) + 152.23 + 130.36 P/F , 10%, 11 TNPV0 = $203.42

Therefore, the overall PV of this dual BT at time t = 0 is $203.42.

15 5

20

25

30

35

30

25

20

15 10

10

–1 0

15 5 5

1

2

3

4

5

6

7

8

10

20

25

30

35

30

25

20

15 10

5

9 10 11 12 13 14 15 16 17 18 19 20 21 22 23 24

Figure 9.9 Dual BT cash-flows computation.

7477_book.fm Page 151 Tuesday, March 13, 2007 3:34 PM

chapter ten

Multiattribute investment analysis and selection This chapter presents useful techniques for assessing and comparing investments in order to improve the selection process. The techniques presented include utility models, the project value model, polar plots, benchmarking techniques, and the analytic hierarchy process (AHP).

10.1 The problem of investment selection Investment selection is an important aspect of investment planning. The right investment must be undertaken at the right time to satisfy the constraints of time and resources. A combination of criteria can be used to help in investment selection, including technical merit, management desire, schedule efficiency, benefit/cost ratio, resource availability, criticality of need, availability of sponsors, and user acceptance. Many aspects of investment selection cannot be expressed in quantitative terms. For this reason, investment analysis and selection must be addressed by techniques that permit the incorporation of both quantitative and qualitative factors. Some techniques for investment analysis and selection are presented in the sections that follow. These techniques facilitate the coupling of quantitative and qualitative considerations in the investment decision process. Such techniques as net present value, profit ratio, and equity break-even point, which have been presented in the preceding chapters, are also useful for investment selection strategies.

10.2 Utility models The term utility refers to the rational behavior of a decision maker faced with making a choice in an uncertain situation. The overall utility of an investment can be measured in terms of both quantitative and qualitative factors. This section presents an approach to investment assessment based on utility models that have been developed within an extensive body of literature. The 151

7477_book.fm Page 152 Tuesday, March 13, 2007 3:34 PM

152

Computational Economic Analysis for Engineering and Industry

approach fits an empirical utility function to each factor that is to be included in a multiattribute selection model. The specific utility values (weights) that are obtained from the utility functions are used as the basis for selecting an investment. Utility theory is a branch of decision analysis that involves the building of mathematical models to describe the behavior of a decision maker faced with making a choice among alternatives in the presence of risk. Several utility models are available in the management science literature. The utility of a composite set of outcomes of n decision factors is expressed in the following general form:

()

(

U x = U x1 , x2 , ... , xn

)

where xi = specific outcome of attribute Xi , i = 1, 2, …, n and U(x) is the utility of the set of outcomes to the decision maker. The basic assumption of utility theory is that people make decisions with the objective of maximizing those decisions’ expected utility. Drawing on an example presented by Park and Sharp-Bette (1990), we may consider a decision maker whose utility function with respect to investment selection is represented by the following expression:

()

u x = 1 − e −0.0001x where x represents a measure of the benefit derived from an investment. Benefit, in this sense, may be a combination of several factors (e.g., quality improvement, cost reduction, or productivity improvement) that can be represented in dollar terms. Suppose this decision maker is faced with a choice between two investment alternatives, each of which has benefits specified as follows: Investment I: Probabilistic levels of investment benefits Benefit, x

$10,000

$0

$10,000

$20,000

$30,000

Probability, P(x)

0.2

0.2

0.2

0.2

0.2

Investment II: A definite benefit of $5,000 Assuming an initial benefit of zero and identical levels of required investment, the decision maker must choose between the two investments. For Investment I, the expected utility is computed as follows:

()

E  u x  =

∑ u( x){P ( x )}

7477_book.fm Page 153 Tuesday, March 13, 2007 3:34 PM

Chapter ten:

Multiattribute investment analysis and selection Benefit, x $10,000 $0 $10,000 $20,000 $30,000

Utility, u(x)

P(x)

u(x) P(x)

1.7183 0 0.6321 0.8647 0.9502

0.2 0.2 0.2 0.2 0.2

0.3437 0 0.1264 0.1729 0.1900

Sum

0.1456

153

Thus, E[u(x)1] = 0.1456. For Investment II, we have u(x)2 = u($5,000) = 0.3935. Consequently, the investment providing the certain amount of $5000 is preferred to the riskier Investment I, even though Investment I has a higher expected benefit of ΣxP(x) = $10,000. A plot of the utility function used in the preceding example is presented in Figure 10.1. If the expected utility of 0.1456 is set equal to the decision-maker’s utility function, we obtain the following: 0.1456 = 1 – e-0.0001x* which yields x* = $1,574, referred to as the certainty equivalent (CE) of Investment I (CE1 = 1,574). The certainty equivalent of an alternative with variable outcomes is a certain amount (CA), which a decision maker will consider to be desirable to the same degree as the variable outcomes of the alternative. In general, if CA represents the certain amount of benefit that can be obtained from Investment II, then the criteria for making a choice between the two investments can be summarized as follows: If CA < $1,574, select Investment I. If CA = $1,574, select either investment. If CA > $1,574, select Investment II. u(x)

1.0 0.8 0.6 0.4 0.2 0.0 –0.2 –0.4 –0.6 –0.8 –1.0 –1.2 –1.4 –1.6

(1574, 0.1456)

10

40 20 30 x(Benefit in $1,000s)

Figure 10.1 Utility function and certainty equivalent.

50

7477_book.fm Page 154 Tuesday, March 13, 2007 3:34 PM

154

Computational Economic Analysis for Engineering and Industry

The key in using utility theory for investment selection is choosing the proper utility model. The sections that follow describe two simple but widely used utility models: the additive utility model and the multiplicative utility model.

10.2.1

Additive utility model

The additive utility of a combination of outcomes of n factors (X1, X2, …, Xn) is expressed as follows: n

()

U x =

∑U ( x , x ) 0 i

i

i =1 n

=

∑ k U (x ) i

i

i =1

where = measured or observed outcome of attribute i = number of factors to be compared = combination of the outcomes of n factors = utility of the outcome for attribute i, xi = combined utility of the set of outcomes, x = weight or scaling factor for attribute i(0 < ki < 1) = variable notation for attribute i = worst outcome of attribute i = best outcome of attribute i = set of worst outcomes for the complement of xi = utility of the outcome of attribute I and the set of worst outcomes for the complement of attribute i ki = U ( xi* , xi0 )

xi n x U(xi) U(x) ki Xi xi0 xi* xi0 U ( xi , xi0 ) n

∑ k = 1.0 (required for the additive model). i

i =1

Example 10.1 Let A be a collection of four investment attributes defined as A = {Profit, Flexibility, Quality, Productivity}. Now define X = {Profit, Flexibility} as a subset of A. Then, X is the complement of X defined as X = {Quality, Productivity}. An example of the comparison of two investments under the additive utility model is summarized in Table 10.1 and yields the following results:

7477_book.fm Page 155 Tuesday, March 13, 2007 3:34 PM

Chapter ten:

Multiattribute investment analysis and selection

155

Table 10.1 Example of Additive Utility Model Attribute (i)

Weight (ki)

Investment A Ui(xi)

Investment B Ui(xi)

0.4 0.2 0.3 0.1 1.00

0.95 0.45 0.35 0.75

0.90 0.98 0.20 0.10

Profitability Flexibility Quality Throughput

()

U x

()

U x

n

A

B

=

∑ k U ( x ) = .4 (.95) + .2 (.45) + .3 (.35) + .1(.75) = 0.650 i

i

i

i =1 n

=

∑ k U ( x ) = .4 (.90) + .2 (.98) + .3 (.20) + .1(.10) = 0.626 i

i

i

i =1

Because U(x)A > U(x)B, Investment A is selected.

10.2.2

Multiplicative utility model

Under the multiplicative utility model, the utility of a combination of outcomes of n factors (X1, X2, …, Xn1) is expressed as

()

U x =

1  C 



n

∏ (Ck U ( x ) + 1) − 1 i

i

i

i =1



where C and ki are scaling constants satisfying the following conditions: n

∏ (1 + Ck ) − C = 1.0 i

i=1

1.0 < C < 0.0 0 < ki < 1 The other variables are as defined previously for the additive model. Using the multiplicative model for the data in Table 10.1 yields U(x)A = 0.682 and U(x)B = 0.676. Thus, Investment A is the best option.

10.2.3

Fitting a utility function

An approach presented in this section for multiattribute investment selection is to fit an empirical utility function to each factor to be considered in the

7477_book.fm Page 156 Tuesday, March 13, 2007 3:34 PM

156

Computational Economic Analysis for Engineering and Industry

selection process. The specific utility values (weights) that are obtained from the utility functions may then be used in any of the standard investment justification methodologies. One way to develop empirical utility function for an investment attribute is to plot the “best” and “worst” outcomes expected from the attribute and then to fit a reasonable approximation of the utility function using concave, convex, linear, S-shaped, or any other logical functional form. Alternately, if an appropriate probability density function can be assumed for the outcomes of the attribute, then the associated cumulative distribution function may yield a reasonable approximation of the utility values between 0 and 1 for corresponding outcomes of the attribute. In that case, the cumulative distribution function gives an estimate of the cumulative utility associated with increasing levels of attribute outcome. Simulation experiments, histogram plotting, and goodness-of-fit tests may be used to determine the most appropriate density function for the outcomes of a given attribute. For example, the following five attributes are used to illustrate how utility values may be developed for a set of investment attributes. The attributes are return on investment (ROI), productivity improvement, quality improvement, idle-time reduction, and safety improvement. Example 10.2 Suppose we have historical data on the ROI for investing in a particular investment. Assume that the recorded ROI values range from 0 to 40%. Thus, the worst outcome is 0%, and the best outcome is 40%. A frequency distribution of the observed ROI values is developed and an appropriate probability density function (pdf) is fitted to the data. For our example, suppose the ROI is found to be exceptionally distributed with a mean of 12.1%. That is,  1 − x/β , if x ≥ 0  e f x = β  0, otherw wise 

()

 1 − e − x/β , if x ≥ 0 F x = otherwise 0,

()

()

≈U x

where β = 12.1, F(x) approximates U(x). The probability density function and cumulative distribution function are shown graphically in Figure 10.2. The utility of any observed ROI within the applicable range may be read directly from the cumulative distribution function. For the productivity improvement attribute, suppose it is found (based on historical data analysis) that the level of improvement is normally distributed with a mean of 10% and a standard deviation of 5%. That is,

7477_book.fm Page 157 Tuesday, March 13, 2007 3:34 PM

Chapter ten:

Multiattribute investment analysis and selection Exponential distribution (mean = 12.1)

157

1

U (x)

f (x)

0.8 0.6 0.4 0.2 0

20

40 x(% ROI)

60

0

80

0

20

40 60 x(% ROI)

80

Figure 10.2 Estimated utility function for investment ROI.

1

U(x)

f(x)

0.8 0.6 0.4 0.2 –15

–5

5

15

25

0 –15 –5

35

5

x(%)

15

25

35

x(%)

Figure 10.3 Utility function for productivity improvement.

()

f x =

1 2 πσ

e

1  x −µ  −  2  σ 

2

−∞ 0

β −1

7477_book.fm Page 158 Tuesday, March 13, 2007 3:34 PM

158

Computational Economic Analysis for Engineering and Industry

where a = lower limit for the distribution b = upper limit for the distribution α, β = the shape parameters for the distribution As with the normal distribution, there is no closed-form expression for F(x) for the beta distribution. However, if either of the shape parameters is a positive integer, then a binomial expansion can be used to obtain F(x). Figure 10.4 shows a plot of f(x) and the estimated U(x) for quality improvement due to the proposed investment. Based on work analysis observations, suppose idle-time reduction is found to be best described by a log normal distribution with a mean of 10% and standard deviation of 5%. This is represented as follows:

()

f x =

1 2 πσ

e

1  x−µ  −  2  σ 

2

−∞ 3. Therefore, use 0.90 factor. Thus, 125,333 ∞ 79.45 ∞ 0.90 = 8,961,936. A + B + C = 40,320,252.

7477_book.fm Page 216 Tuesday, March 13, 2007 3:34 PM

216

Computational Economic Analysis for Engineering and Industry A+B+C/376,000 = 40,320,252/376,000 = $107.23/sq. ft., which is the expected estimated average cost per square foot for the commercial buildings. For Building C: Assume A = 16,000 sq. ft. for restaurant. B = 37,000 sq. ft. for college student union. A: Means Median = $119/sq. ft. Typical sq. ft. = 4,400 Proposed sq. ft/typical sq. ft. = 16,000/4,400 = 3.63 > 3. Therefore, use 0.90 factor. 16,000 sq. ft. ∞ 119 ∞ 0.90 = 1,713,600. B: Means Median = $129/sq. ft. Typical sq. ft. = 33,400. Proposed sq. ft./typical sq. ft. = 37,000/33,400 = 1.1 < 3. Therefore, use 0.98 factor. Thus, 37,000 ∞ 129 ∞ 0.98 = 4,773,000. A+B = 6,486,600. A+B/53,000 = 6,486,600/53,000 = 122.4.

Now, (122.40 – 107.23)/107.23 = 14.15%, which implies a 14.15% cost increase of C over Building A. Thus, we have F3 Impact = 14%($63) = $8.82 where 14.15% has been rounded down to 14%.

13.6.4

F4: Impact of excessive contract terms and conditions

Government contract terms and conditions often have excessive requirements that have cost implications. Based on experiential data obtained from construction supervisors, we allocate a 14% impact to this factor. Thus, we have F4 Impact = 14.00%($63) = $8.82

13.6.5

F5: Impact of procurement process

One nuance of government-funded projects is the cumbersome procurement process, which leads to less cost-effective implementations. A suggested impact of 18% was assigned to this factor. Thus, we have F5 Impact = 18.00%($63) = $11.34

7477_book.fm Page 217 Tuesday, March 13, 2007 3:34 PM

Chapter thirteen:

Cost benchmarking case study

217

Table 13.5 Summary of Cost Difference Distribution Factor

Percentage (%)

Dollar Amount

1 2 3 4 5 Total

7.00 47.00 14.00 14.00 18.00 100.00

4.41 29.61 8.82 8.82 11.34 $63.00

A summary of the cost distribution over the factors is shown in Table 13.5. A graphical representation of the cost difference contribution is shown in Figure 13.9. Figure 13.10 presents the graphical information in a bar-chart format. 70 $ Contributions

60 50 40 30 20 10 0 F1

F2

F3

F4

Factors

Figure 13.9 Line chart of factor contributions by Dollar amounts. 100.00 18.00

90.00 80.00

14.00

Percentage (%)

70.00

14.00

60.00 50.00 40.00 30.00

47.00

20.00 10.00 7.00

0.00

Factor F1

F2

F3

F4

F5

Figure 13.10 Bar chart of factor contributions by percentage.

F5

7477_book.fm Page 218 Tuesday, March 13, 2007 3:34 PM

218

Computational Economic Analysis for Engineering and Industry

13.7 Conclusions and recommendations The results in this study are validated based on a combination of anecdotal information, quantitative computations, a review of construction data, and reference to published literature on government construction projects. The conclusion is that government construction projects, by virtue of their bureaucratic nature, create an incidence of cost escalations and productivity loss. It is recommended that government projects be benchmarked against commercial projects. In this respect, government projects can borrow the “best practices” of commercially executed projects in lowering construction costs and improving work productivity. Most contractors will carry out their projects successfully if given operating guidelines and left with the latitude to develop the means and methods to reduce cost and improve quality and productivity. In spite of the prevailing tight government control procedures, management oversight, and implementation audits, many government projects still end up with quality problems and cost overruns. Project analysts should review the marginal benefit of government procedures vs. the incremental cost brought on by those procedures. The government process itself has fueled the stereotype that government projects have infinite loopholes and unlimited funding potentials. This has the consequence that contractors, who would have normally executed projects under their own efficient methods, resort to unproductive practices in order to take advantage of the “government project” stereotypes and extra opportunities for profit. Due to the difference in project cost accounting and cost collection methods used on these projects, the soft costs could not be compared in a meaningful way. However, design costs as a percentage of construction costs were compared for the projects involved, and they all fall into a range of 6 to 7%. Although the reasons for the differences in the soft costs were not evaluated in this study, anecdotal evidence suggests they may be caused by overly prescriptive requirements and lengthy design review cycles required for DOE projects. These soft costs need to be evaluated in a separate focused study. Table 13.2 depicts the hard costs of the RSC (DOE-funded project) as approximately $66 per sq. ft higher than the other two construction projects. To further understand the differences in hard costs, the RSC project (DOE) and the JICS/ORCAS project (State of Tennessee/commercial) were compared in more detail. This was straightforward because of their similar square footage, similar structures, close proximity to each other, and similar contracting (design–bid–build). In comparing the two, the following five factors were found to be the major contributors to the hard cost differences: • Impact of Davis-Bacon wages • Impact of PLA • Overly prescriptive requirements of the UT–Battelle Construction Specification Division 1, General Requirements, including ESH&Q oversight, and site training

7477_book.fm Page 219 Tuesday, March 13, 2007 3:34 PM

Chapter thirteen:

Cost benchmarking case study

219

• Apparently excessive requirements of procurement contract terms and conditions • Impact of lengthy procurement process on project schedules The available data confirm that the DOE approach to new commercial or light industrial (nonnuclear) buildings is process driven. This emphasis forces contractors to focus on adhering to the requirements of DOE documents (Processes for Project Management [Required by DOE O 413.3] and Construction Project Acquisition), rather than focusing on the successful construction of a building. Thus, a project is considered more successful if the DOE process has been followed without any mistakes, rather than focusing on assuring that the maximum building (bricks and mortar) is constructed for the allocated budget. The contractor is concerned more about satisfying the process instead of pursuing practices that will achieve building performance that meets or exceeds expectations.

13.7.1

Recommendations for reducing future project costs

• Perform further benchmarking to determine commercial practices and implement changes as appropriate to further reduce construction costs. • Recommend to DOE the elimination of prescriptive requirements that are not required by regulations and statutes, and the identification of ways to change facility construction progress from a processbased project management system to a performance-based project management system. • Evaluate the need for a PLA for building federal and/or DOE “commercial-like” construction projects. • Evaluate the need for a standard DOE line-item project management process and review cycle according to DOE O 413.3, Program and Project Management for the Acquisition of Capital Assets, for commercial-type projects under $50 million. The benefit of using the standard project management process for research or science projects is questionable. • Recommend that DOE use a more commercial, less process-driven approach for new construction projects (e.g., eliminate DOE O 413.3 for “nonnuclear commercial-like” projects). For example, the research team recommends fewer reviews, focusing on performance, and focusing on the use of commercial contracting methods (e.g., American Institute of Architects [AIA] model contracts). These approaches result in more facility for the dollar and better “other” performance indicators (e.g., safety performance). The research team also recommends the use of DOE project personnel to support and provide oversight of the management and operating (M&O) contractor in adopting this revised approach. • The team recommends further benchmarking studies at other locations to confirm the data and to provide the DOE with additional rationale for adapting its processes for commercial-type construction projects.

7477_book.fm Page 220 Tuesday, March 13, 2007 3:34 PM

220

Computational Economic Analysis for Engineering and Industry

13.7.1.1

Overall project recommendation

For regular construction not involving safety-critical process plants, best commercial construction practices should be used. The practice of using the same rigid review process for all projects is wasteful and ineffective. Nonprocess-plant projects do not need tight review and control of every project step. It is recommended that other government construction projects be cost-benchmarked by using a methodology similar to the one presented in this chapter.

Reference 1. Badiru, Adedeji B., Delgado, Vincent, Ehresma-Gunter, Jamie, Omitaomu, Olufemi A., Nsofor, Godswill, and Saripali, Sirisha (2004). Graphical and Analytical Methodology for Cost Benchmarking for New Construction Projects. Proceedings of 13th International Conference on Management of Technology (IAMOT 2004), Washington, DC. April 3–7.

7477_book.fm Page 221 Tuesday, March 13, 2007 3:34 PM

appendix A

Definitions and terms Accounting Life: The period of time over which the amount of the asset cost to be depreciated, or recovered, will be allocated to expenses by accountants. Actual Dollars: Cash flow at the time of the transaction. Alternative, Contingent: An alternative that is feasible only if some other alternative is accepted. The opposite of a mutually exclusive alternative. Alternative, Economic: A plan, project, or course of action intended to accomplish some objective that has or will be valued in monetary terms. Alternative, Independent: An alternative such that its acceptance has no influence on the acceptance of other alternatives under consideration. Alternative, Mutually Exclusive: An alternative such that its selection rules out the selection of any other alternatives under consideration. Amortization: (1) (a) As applied to a capitalized asset, the distribution of the initial cost by periodic charges to expenses as in depreciation. Most amortizable assets have no fixed life; (b) The reduction of a debt by either periodic or irregular payments. (2) A plan to pay off a financial obligation according to some prearranged program. Annual Cost: The negative of Annual Worth. (See Equivalent Uniform Annual Cost.) Annual Equivalent: In the time value of money, one of a sequence of equal end-of-year payments that would have the same financial effect when interest is considered as another payment or sequence of payments that are not necessarily equal in amount or equally spaced in time. Annual Worth: A uniform amount of money at the end of each and every period over the planning horizon, equivalent to all cash flows discounted over the planning horizon when interest is considered. Annuity: (1) An amount of money payable to a beneficiary at regular intervals for a prescribed period of time out of a fund reserved for that purpose. (2) A series of equal payments occurring at equally spaced periods of time.

221

7477_book.fm Page 222 Tuesday, March 13, 2007 3:34 PM

222

Computational Economic Analysis for Engineering and Industry

Annuity Factor: The function of interest rate and time that determines the amount of periodic annuity that may be paid out of a given fund. (See Capital Recovery Factor.) Annuity Fund: A fund that is reserved for payment of annuities. The present worth of funds required to support future annuity payments. Annuity Fund Factor: The function of the interest rate and time that determines the present worth of funds required to support a specified schedule of annuity payments. (See Present Worth Factors, Uniform Series.) Apportion: In accounting or budgeting, the process by which a cash receipt or disbursement is divided among and assigned to specific time periods, individuals, organization units, products, projects, services, or orders. Bayesian Statistics: (1) Classical — the use of probabilistic prior information and evidence about a process to predict probabilities of future events. (2) Subjective — the use of subjective forecasts to predict probabilities of future events. Benefit/Cost (Cost-Benefit) Analysis: An analysis technique in which the consequences on an investment evaluated in monetary terms are divided into separate categories of costs and benefits. Each category is then converted into an annual equivalent or present worth for analysis purposes. Benefit/Cost Ratio: A measure of project worth in which the equivalent benefits are divided by the equivalent costs. Benefit/Cost Ratio Method: (See Benefit-Cost Analysis.) Book Value: The original cost of an asset or group of assets less the accumulated book depreciation. Break-Even Chart: (1) A graphic representation of the relation between total income and total costs (sum of fixed and variable costs) for various levels of production and sales indicating areas of profit and loss. (2) Graphic representation of a figure of merit as a function of a specified relevant parameter. Break-Even Point: (1) The rates of operations, output, or sales at which income will just cover costs. Discounting may or may not be used in making these calculations. (2) The value of a parameter such that two courses of action result in an equal value for the figure of merit. Capacity Factor: (1) The ratio of current output to maximum capacity of the production unit. (2) In electric utility operations, it is the ratio of the average load carried during a period of time divided by the installed rating of the equipment carrying the load. (See Demand Factor and Load Factor). Capital: (1) The financial resources involved in establishing and sustaining an enterprise or project. (2) A term describing wealth that may be utilized to economic advantage. The form that this wealth takes may be as cash, land, equipment, patents, raw materials, finished products, etc. (See Investment and Working Capital.)

7477_book.fm Page 223 Tuesday, March 13, 2007 3:34 PM

Appendix A:

Definitions and terms

223

Capital Budgeting: The process by which organizations periodically allocate investment funds to proposed plans, programs, or projects. Capital Recovery: (1) Charging periodically to operations amounts that will ultimately equal the amount of capital expended. (2) The replacement of the original cost of an asset plus interest. (3) The process of regaining the new investment in a project by means of setting revenues in excess of the economic investment costs. (See Amortization, Depletion, and Depreciation.) Capital Recovery Factor: A number that is a function of time and the interest rate, used to convert a present sum to an equivalent uniform annual series of end-of-period cash flows. (See Annuity Factor.) Capital Recovery with Return: The recovery of an original investment with interest. In the public utility industry, this is frequently referred to as the revenue requirements approach. Capitalized Asset: Any asset capitalized on the books of account of an enterprise. Capitalized Cost: (1) The present worth of a uniform series of periodic costs that continue indefinitely (hypothetically infinite). Not to be confused with capitalized expenditure. (2) The present sum of capital which, if invested in a fund earning a stipulated interest rate, will be sufficient to provide for all payments required to replace and/ or maintain an asset in perpetual service. Cash Flow: The actual monetary units (e.g., dollars) passing into and out of a financial venture or project being analyzed. Cash-Flow Diagram: The illustration of cash flows (usually vertical arrows) on a horizontal line where the scale along the line is divided into time-period units. Cash-Flow Table: A listing of cash flows, positive and negative, in a table in the order of the time period in which the cash flow occurs. Challenger: In replacement analysis, a proposed property or equipment that is being considered as a replacement for the presently owned property or equipment (the defender). In the analysis of multiple alternatives, an alternative under consideration that is to be compared with the last acceptable alternative (the defender). (See MAPI Method.) Common Costs: In accounting, costs that cannot be identified with a given output of products, operations, or services. Expenditures that are common to all alternatives. Compound Amount: (1) The equivalent value, including interest, at some stipulated time in the future of a series of cash flows occurring prior to that time. (2) The monetary sum that is equivalent to a single (or a series of) prior sums when interest is compounded at a given rate. Compound Amount Factors: Functions of interest and time that when multiplied by a single cash flow (single payment compound amount factor), or a uniform series of cash flows (uniform series compound

7477_book.fm Page 224 Tuesday, March 13, 2007 3:34 PM

224

Computational Economic Analysis for Engineering and Industry

amount factor) will give the future worth at compound interest of such single cash flow or series. Compound Interest: (1) The type of interest that is periodically added to the amount of investment (or loan) so that subsequent interest is based on the cumulative amount. (2) The interest charges under the condition that interest is charged on any previous interest earned in any time period, as well as on the principal. Compounding, Continuous: A compound-interest assumption in which the compounding period is of infinitesimal length and the number of periods is infinitely great. A mathematical concept that is conceptually attractive and mathematically convenient for dealing with frequent (e.g., daily) compounding periods within a year. Compounding, Discrete: A compound interest assumption in which the compounding period is of a specified length such as a day, week, month, quarter year, half year, or year. Compounding Period: The time interval between dates (or discrete times) at which interest is paid and added to the amount of an investment or loan. Usually designates the frequency of compounding during a year. Constant Dollars: An amount of money at some point in time, usually the beginning of the planning horizon, equivalent in purchasing power to the active dollars necessary to buy the good or service. Actual dollars adjusted to a relative price change. Cost/Benefit Analysis: (See Benefit/Cost Analysis.) Cost-Effectiveness Analysis: An analysis in which the major benefits may not be expressed in monetary terms. One or more effectiveness measures are substituted for monetary values, resulting in a trade-off between marginal increases in effectiveness and marginal increases in costs. Cost of Capital: A term, usually used in capital budgeting, to express as an interest rate percentage the overall estimated cost of investment capital at a given point in time, including both equity and borrowed funds. Current Dollars: (See Actual Dollars). Cutoff Rate of Return (Hurdle Rate): The rate of return after taxes that will be used as a criterion for approving projects or investments. It is determined by management based on the supply and demand for funds. It may or may not be equal to the minimum attractive rate of return (MARR) but is at least equal to the estimated cost of capital. Decision Theory: With reference to engineering economy, it is a branch of economic analysis devoted to the study of decision processes involving multiple possible outcomes, defined either discretely or on a continuum, and deriving from the theory of games and economic behavior and probabilistic modeling.

7477_book.fm Page 225 Tuesday, March 13, 2007 3:34 PM

Appendix A:

Definitions and terms

225

Decision Tree: In decision analysis, a graphical representation of the anatomy of decisions showing the interplay between a present decision, the probability of chance events, possible outcomes and future decisions, and their results or payoffs. Decisions Under Certainty: From the literature of decision theory, that class of problems wherein single estimates with respect to cash flows and economic life (complete information) are used in arriving at a decision among alternatives. Decisions Under Risk: From the literature of decision theory, that class of problems in which multiple outcomes are considered explicitly for each alternative, and the probabilities of the outcomes are assumed to be known. Decisions Under Uncertainty: From the literature of decision theory, that class of problems in which multiple outcomes are considered explicitly for each alternative but the probabilities of the outcomes are assumed to be unknown. Defender: In replacement analysis, the presently owned property or equipment being considered for replacement by the most economical challenger. In the analysis of multiple alternatives, the previously judged acceptable alternative against which the next alternative to be evaluated (the challenger) is to be compared. Deflating (By a Price Index): Adjusting some nominal magnitude, e.g., an actual dollar estimate, by a price index. It may be required in order to express that magnitude in units of constant purchasing power. (See Inflating, Constant Dollars, and Current Dollars.) Deflation: A decrease in the relative price level of a factor of production, an output, or the general price level of all goods and services. A deflationary period is one in which there is (or is expected to be) a sustained decrease in price levels. Demand Factor: (1) The ratio of the current production rate of the system divided by the maximum instantaneous production rate. (2) The ratio of the average production rate, as determined over a specified period of time, divided by the maximum production rate. (3) In electric utility operations, it is the ratio of the maximum kilowatt load demanded during a given period divided by the connected load. (Also see Capacity Factor and Load Factor.) Depletion: (1) An estimate of the lessening of the value of an asset due to a decrease in the quantity available for exploitation. It is similar to depreciation except that it refers to natural resources such as coal, oil, and timber. (2) A form of capital recovery applicable to properties such as listed previously. Its determination for income-tax purposes may be on a unit of production basis, related to original cost or appraised value of the resource (known as cost depletion), or based on a percentage of the income received from extracting or harvesting (known as percentage depletion.)

7477_book.fm Page 226 Tuesday, March 13, 2007 3:34 PM

226

Computational Economic Analysis for Engineering and Industry

Depletion Allowance: An annual tax deduction based upon resource extraction. (See Depletion.) Depreciation: (1) (a) Decline in value of a capitalized asset; (b) A form of capital recovery, usually without interest, applicable to property with two or more years’ life span in which an appropriate portion of the asset’s value periodically is charged to current operations. (2) A loss of value due to physical or economic reasons. (3) In accounting, depreciation is the allocation of the book value of this loss to current operations according to some systematic plan. Depending on then existing income-tax laws, the amount and timing of the charge to current operations for tax purposes may differ from that used to report annual profit and loss. Depreciation, Accelerated: Depreciation methods accepted by the taxing authority that write off the value (cost) of an asset usually over a shorter period of time (i.e., at a faster rate) than the expected economic life of the asset. An example is the Accelerated Cost Recovery System (ACRS) introduced in the U.S. in 1981 and modified in later years. Depreciation Allowance: An annual income-tax deduction, and/or charge to current operations, of the original cost of a fixed asset. The income-tax deduction may not equal the charge to current operations. (See Depreciation.) Depreciation Basis: In tax accounting, the cost of the otherwise-determined value of a group of fixed assets, including installation costs and certain other expenditures, and excluding certain allowances. The depreciation basis is the amount that, by law, and/or acceptance by the taxing authority, may be written off for tax purposes over a period of years. Depreciation, Declining Balance: A method of computing depreciation in which the annual charge is a fixed percentage of the depreciated book value at the beginning of the year to which the depreciation charge applies. Depreciation, Multiple Straight-Line: A method of depreciation accounting in which two or more straight-line rates are used. This method permits a predetermined portion of the asset to be written off in a fixed number of years. One common practice is to employ a straight-line rate that will write off Ω of the cost in the first half of the anticipated service life with a second straight-line rate used to write off the remaining π in the remaining half life. (See Depreciation, Straight-Line.) Depreciation, Sinking Fund: (1) A method of computing depreciation in which the periodic charge is assumed to be deposited in a sinking fund that earns interest at a specified rate. The sinking fund may be real but usually is hypothetical. (2) A method of depreciation where a fixed sum of money regularly is deposited at compound interest in a real or hypothetical fund in order to accumulate an

7477_book.fm Page 227 Tuesday, March 13, 2007 3:34 PM

Appendix A:

Definitions and terms

227

amount equal to the total depreciation of an asset at the end of the asset’s estimated life. The depreciation charge to operations for each period equals the sinking fund deposit amount plus interest on the beginning of the period sinking fund balance. Depreciation, Straight-Line: A method of computing depreciation wherein the amount charged to current operations is spread uniformly over the estimated life of an asset. The allocation may be performed on a unit-of-time basis or a unit-of-production basis, or some combination of the two. Depreciation, Sum-of-Years Digits: A method of computing depreciation wherein the amount charged to current operations for any year is based on the ratio: (years of remaining life)/(1 + 2 + 3 + … + n), n being the estimated life. Deterioration: A reduction in value of a fixed asset due to wear and tear and action of the elements. It is a term used frequently in replacement analysis. Development Cost: (1) The sum of all the costs incurred by an inventor or sponsor of a project up to the time the project is accepted by those who will promote it. (2) In international literature, activities in developing nations intended to improve their infrastructure. Direct Cost: A traceable cost that can be segregated and charged against specific products, operations, or services. Discount Rate: (See Interest Rate and Discounted Cash Flow.) Discounted Cash Flow: (1) Any method of handling cash flows over time, either receipts or disbursements, in which compound interest and compound interest formulae are employed in their analytical treatment. (2) An investment analysis that compares the present worth of projected receipts and disbursements occurring at designated times in order to estimate the rate of return from the investment or project. In this sense, also see Rate of Return and Profitability Index. Dollars, Constant (Real Dollars): Dollars (or some other monetary unit) of constant purchasing power independent of the passage of time. In situations where inflationary or deflationary effects have been assumed when cash flows were estimated, those estimates are converted to constant dollars (base-year dollars) by adjustment by some readily accepted general inflation/deflation index. Sometimes termed actual dollars, although this term also is used to describe current dollar values. (See Current Dollars, Inflating, and Deflating.) Dollars, Current (Then-Current Dollars): Estimates of future cash flows that include any anticipated changes in amount due to inflationary or deflationary effects. Usually, these amounts are determined by applying an index to base-year dollar (or other monetary unit) estimates. Sometimes termed actual dollars, although this term also is used to describe constant dollar values. (See Constant Dollars, Inflating, and Deflating.)

7477_book.fm Page 228 Tuesday, March 13, 2007 3:34 PM

228

Computational Economic Analysis for Engineering and Industry

Earning Value (Earning Power of Money): The present worth of an income producer’s estimated future net earnings as predicted on the basis of recent and present expenses and earnings and the business outlook. Economic Life: The period of time extending from the date of installation to the date of retirement from the intended service, over which a prudent owner expects to retain an equipment or property so as to minimize cost or maximize net return. (See Life.) Economy: (1) The cost or net return situation regarding a practical enterprise or project, as in economy study, engineering economy, or project economy. (2) A system for the management of resources. (3) The avoidance of (or freedom from) waste in the management of resources. Effective Interest: (See Interest Rate, Effective.) Effectiveness: In the engineering economy, the measurable consequences of an investment not reduced to monetary terms; e.g., reliability, maintainability, safety. Endowment: A fund established for the support of some project for succession of donations or financial obligations. Endowment Method: As applied to an economy study, a comparison of alternatives based on the present worth or capitalized cost of the anticipated financial events. Engineering Economy: (1) The application of economic or mathematical analysis and synthesis to engineering decisions. (2) A body of knowledge and techniques concerned with the evaluation of the worth of commodities and services relative to their costs and with methods of estimating inputs. Equivalent Uniform Annual Cost (EUAC): (See Annual Cost.) Estimate: A magnitude determined as closely as it can be by the use of past history and the exercise of sound judgment based upon approximate computations, not to be confused with offhand approximations that are little better than outright guesses. Exchange Rate: The rate at a given point in time at which the currency of one nation exchanges for that of another. Expected Yield: In finance, the ratio of the expected return from an investment, usually on an after-tax basis, divided by the investment. External Rate of Return: A rate of return calculation that takes into account the cash receipts and disbursements of a project and assumes that all net receipts (cash throw offs) are reinvested elsewhere in the enterprise at some stipulated interest rate. (Also see Rate of Return and Internal Rate of Return.) Fair Rate of Return: The maximum rate of return that an investor-owned public utility is entitled to earn on its rate base in order to pay interest and dividends and attract new capital. The rate, or percentage, usually is determined by state or federal regulatory bodies.

7477_book.fm Page 229 Tuesday, March 13, 2007 3:34 PM

Appendix A:

Definitions and terms

229

First Cost: The initial investment in a project or the initial cost of capitalized property, including transportation, installation, preparation for service, and other related initial expenditures. Fixed Assets: The tangible portion of an investment in an enterprise or project that is comprised of land, buildings, furniture, fixtures, and equipment with an expected life greater than 1 year. Fixed Cost: Those costs that tend to be unaffected by changes in the number of units produced or the volume of service given. Future Worth: (1) The equivalent value at a designated future date based on the time value of money. (2) The monetary sum, at a given future time, which is equivalent to one or more sums at given earlier times when interest is compounded at a given rate. Going-Concern Value: The difference between the value of a property as it stands possessed of its going elements and the value of the property alone as it would stand at completion of construction as a bare or inert assembly of physical parts. Goodwill Value: That element of value that inheres in the fixed and favorable consideration of customers arising from an established well-known and well-conducted business. This is determined as the difference between what a prudent business person is willing to pay for the property and its going-concern value. Gradient Factors: A group of compound-interest factors used for equivalence conversions of arithmetic or geometric gradients in cash flow. In general use are the arithmetic gradient to uniform series (gradient conversion) factor, the arithmetic gradient to present worth (gradient present worth) factor, and the geometric gradient to present worth factor. Increment Cost (Incremental Cost): The additional (or direct) cost that will be incurred as the result of increasing output by one unit more. Conversely, it may be defined as the cost that will not be incurred if the output is reduced by one unit. (2) The variation in output resulting from a unit change in input. (3) The difference in costs between a pair of mutually exclusive alternatives. Indirect Cost: Nontraceable or common costs that are not charged against specific products, operations, or services but rather are allocated against all (or some group of) products, operations, and/or services by a predetermined formula. Inflating (By a Price Index): The adjustment of a present- or base-year price by a price index in order to obtain an estimate of the current (or then current) price at future points in time. (See Deflating, Constant Dollars, and Current Dollars.) Inflation: A persistent rise in price levels, generally not justified by increased productivity, and usually resulting in a decline in purchasing power. Sometimes the term is used interchangeably with escalation. However, this latter term more often is restricted to the differential

7477_book.fm Page 230 Tuesday, March 13, 2007 3:34 PM

230

Computational Economic Analysis for Engineering and Industry

increase in a price relative to some specific index of general changes in price levels. (See Deflation.) Intangibles: (1) In economy studies, those elements, conditions, or economic factors that cannot be evaluated readily or accurately in monetary terms. (2) In accounting, the assets of an enterprise that cannot reliably be values in monetary terms (e.g., goodwill). (See Irreducibles.) Interest: (1) The monetary return or other expectation that is necessary to divert money away from consumption and into long-term investment. (2) The cost of the use of capital. It is synonymous with the term time value of money. (3) In accounting and finance, (a) a financial share in a project or enterprise; (b) periodic compensation for the lending of money. Interest Rate: The ratio of the interest accrued in a given period of time to the amount owed or invested at the start of that period. Interest Rate, Effective: The actual interest rate for one specified period of time. Frequently, the term is used to differentiate between nominal annual interest rates and actual annual interest rates when there is more than one compounding period in a year. Interest Rate, Market: The rate of interest quoted in the market place that includes the combined effects of the earning value of capital, the availability of funds, and anticipated inflation or deflation. Interest Rate, Nominal: (1) The interest rate for some period of time that ignores the compounding effect of interest calculations during subperiods within that period. (2) The annual interest rate, or annual percentage rate (APR), frequently quoted in the media. Interest Rate, Real: An estimate of the true earning rate of money when other factors, especially inflation, affecting the market rate have been removed. Internal Rate of Return: A rate of return calculation which takes into account only the cash receipts and disbursements generated by an investment, their timing, and the time value of money. (See Rate of Return, Discounted Cash Flow, and External Rate of Return.) Investment: (1) As applied to an enterprise as a whole, the cost (or present value) of all the properties and funds necessary to establish and maintain the enterprise as a going concern. The capital tied up in an enterprise or project. (2) Any expenditure which has substantial and enduring value (generally more than one year) and which is therefore capitalized. (See First Cost.) Investor’s Method: A term most often used in the valuation of bonds. (See Rate of Return, Internal Rate of Return, and Discounted Cash Flow.) Irreducibles: Those intangible conditions or economic factors which cannot readily be reduced to monetary terms; e.g., ethical considerations, esthetic values, or nonquantifiable potential environment concerns. Leaseback: A business arrangement wherein the owner of land, buildings, and/or equipment sells such assets and simultaneously leases them back under a long-term lease.

7477_book.fm Page 231 Tuesday, March 13, 2007 3:34 PM

Appendix A:

Definitions and terms

231

Life: (1) Economic: that period of time after which a machine or facility should be retired from primary service and/or replaced as determined by an engineering economy study. The economic impairment may be absolute or relative. (2) Physical: that period of time after which a machine or facility can no longer be repaired or refurbished to a level such that it can perform a useful function. (3) Service: that period of time after which a machine or facility cannot perform satisfactorily its intended function without major overhaul. Life Cycle Cost: The present worth or equivalent uniform annual cost of equipment or a project which takes into account all associated cash flows throughout its life including the cost of removal and disposal. Load Factor: (1) Applied to a physical plant or equipment, it is the ratio of the average production rate for some period of time to the maximum rate. Frequently, it is expressed as a percentage. (2) In electric utility operations, it is the average electric usage for some period of time divided by the maximum possible usage. (See Capacity Factor and Demand Factor.) MAPI Method: A procedure for equipment replacement analysis developed by George Terborgh for the Machinery and Allied Products Institute. It uses a fixed format and provides charts and graphs to facilitate calculations. A prominent feature of this method is that it includes explicitly an allowance for obsolescence. Marginal Cost: (1) The rate of change of cost as a function of production or output. (2) The cost of one additional unit of production, activity, or service. (See Increment Cost and Direct Cost.) Matheson Formula: A title for the formula used for declining balance depreciation. (See Declining Balance Depreciation.) Maximax Criterion: In decision theory, probabilities unknown, is a rule that says choose the alternative with the maximum of the maximum returns identified for each alternative. Maximin Criterion: In decision theory, probabilities unknown, is a rule that says choose the alternative with the maximum of the minimum returns identified for each alternative. Also called a maximum security level strategy or Wald’s strategy. Minimax Criterion: In decision theory, probabilities unknown, is a rule that says choose the alternative with the minimum of the maximum costs identified for each alternative. Also called a maximum security level strategy. Minimax Regret Criterion: In decision making under uncertainty, a rule that says choose the alternative with the least potential net return or cost regret. Minimin Criterion: In decision theory, probabilities unknown, is a rule that says choose the alternative with the minimum of the minimum costs identified for each alternative.

7477_book.fm Page 232 Tuesday, March 13, 2007 3:34 PM

232

Computational Economic Analysis for Engineering and Industry

Minimum Attractive Rate of Return: The effective annual rate of return on investment, either before or after taxes, which just meets the investor’s threshold of acceptability. It takes into account the availability and demand for funds as well as the cost of capital. Sometimes termed the minimum acceptable return. (See Cost of Capital and Cutoff Rate of Return.) Minimum Cost Life: (See Economic Life.) Multiple Rates of Return (Multiple Roots): A situation in which the structure of a cash-flow time series is such that it contains more than one solving internal rate of return. Nominal Dollars: (See Actual Dollars.) Nominal Interest: (See Interest Rate, Nominal.) Obsolescence: (1) The condition of being out of date. A loss of value occasioned by new developments that place the older property at a competitive disadvantage. A factor in depreciation. (2) A decrease in the value of an asset brought about by the development of new and more economical methods, processes, and/or machinery. (3) The loss of usefulness or worth of a product or facility as the result of the appearance of better and/or more economical products, methods, or facilities. Opportunity Cost: The cost of not being able to use monetary funds otherwise due to that limited resource being applied to an “approved” investment alternative and thus not being available for investment in other income-producing alternatives. Sometimes expressed as a rate. Payback Period: (1) Regarding an investment, the number of years (or months) required for the related profit or savings in operating cost to equal the amount of said investment. (2) The period of time at which a machine, facility, or other investment has produced sufficient net revenue to recover its investment costs. Payback Period, Discounted: Same as Payback Period except the period includes a return on investment at the interest rate used in the discounting. Payoff Period: (See Payback Period.) Payoff Table: A tabular presentation of the payoff results of complex decision questions involving many alternatives, events, and possible future states. Payout Period: (See Payback Period.) Perpetual Endowment: An endowment with hypothetically infinite life. (See Capitalized Cost and Endowment.) Planning Horizon: (1) A stipulated period of time over which proposed projects are to be evaluated. (2) That point of time in the future at which subsequent courses of action are independent of decisions made prior to that time. (3) In utility theory, the largest single dollar amount that a decision maker would recommend to be spent. (See Utility.)

7477_book.fm Page 233 Tuesday, March 13, 2007 3:34 PM

Appendix A:

Definitions and terms

233

Present Worth (Present Value): (1) The monetary sum that is equivalent to a future sum or sums when interest is compounded at a given rate. (2) The discounted value of future sums. Present Worth Factors: (1) Mathematical formulae involving compound interest used to calculate present worths of various cash-flow streams. In table form, these formulae may include factors to calculate the present worth of a single payment, of a uniform annual series, of an arithmetic gradient, and of a geometric gradient. (2) A mathematical expression also known as the present value of an annuity of one. (The present worth factor, uniform series, also is known as the Annuity Fund Factor.). Principal: Property or capital, as opposed to interest or income. Profitability Index: An economic measure of project performance. There are a number of such indexes described in the literature. One of the most widely quoted is one originally developed and so named (the PI) by Ray I. Reul, which essentially is based upon the internal rate of return. (Also see Discounted Cash Flow, Investor’s Method and Rate of Return). Promotion Cost: The sum of all expenses found to be necessary to arrange for the financing and organizing of the business unit that will build and operate a project. Rate of Return (Internal Rate of Return): (1) The interest rate earned by an investment. (2) The interest rate at which the present worth equation (or the equivalent annual worth or future worth equations) for the cash flows of a project or project increment equals zero. (3) As used in accounting, often it is the ratio of annual profit, or average annual profit, to the initial investment or the average book value. Rate of Return, External: A rate of return calculation that employs one or more supplemental interest rates to produce equivalence transformations on a portion or all of the cash flows and then solves for rate of return on that equivalent cash-flow series. Real Dollars: (See Constant Dollars.) Replacement Policy: A set of decision rules for the replacement of facilities that wear out, deteriorate, become obsolete, or fail over a period of time. Replacement models generally are concerned with comparing the increasing operating costs (and possibly decreasing revenues) associated with aging equipment against the net proceeds from alternative equipment. Replacement Study: An economic analysis involving the comparison of an existing facility and one or more facilities with equal or improved characteristics proposed to supplant or displace the existing facility. Required Return: The minimum return or profit necessary to justify an investment. Often it is termed interest, expected return or profit, or charge for the use of capital. It is the minimum acceptable percentage, no more and no less. (See Cost of Capital, Cutoff Rate of Return, and Minimum Attractive Rate of Return.)

7477_book.fm Page 234 Tuesday, March 13, 2007 3:34 PM

234

Computational Economic Analysis for Engineering and Industry

Retirement of Debt: The termination of a debt obligation by appropriate settlement with the lender. The repayment is understood to be in the full amount unless partial settlement is specified. Risk: (1) Exposure to a chance of loss or injury. (2) Exposure to undesired economic consequences. Risk Analysis: Any analysis performed to assess economic risk. Often this term is associated with the use of decision trees. (See Decision Under Risk and Decision Tree.) Salvage Value: (1) The cost recovered or that could be recovered from a used property when removed from service, sold, or scrapped. A factor used in appraisal of property value and in computing depreciation. (2) Normally, it is an estimate of an asset’s net market value at the end of its estimated life. In some cases, the cost of removal may exceed any sale or scrap value; thus, net salvage value is negative. (3) The market value that a machine or facility has at any point in time. Sensitivity: The relative magnitude of decision criterion change with changes in one or more elements of an economy study. If the relative magnitude of the criterion exhibits large change, the criterion is said to be sensitive; otherwise it is insensitive. Sensitivity Analysis: A study in which the elements of an engineering economy study are changed in order to test for sensitivity of the decision criterion. Typically, it is used to assess needed measurement or estimation precision, and often it is used as a substitute for more formal or sophisticated methods such as risk analysis. Service Life: (See Life.) Simple Interest: (1) Interest that is not compounded, i.e., is not added to the income-producing investment or loan. (2) Interest charges under the condition that interest in any time period is only charged on the principal. Frequently, interest is charged on the original principal amount disregarding the fact that the principal still owing may be declining through time. (See Interest Rate, Nominal.) Sinking Fund: (1) A fund accumulated by periodic deposits and reserved exclusively for a specific purpose, such as retirement of a debt or replacement of a property. (2) A fund created by making periodic deposits (usually equal) at compound interest in order to accumulate a given sum at a given future time usually for some specific purpose. Sinking Fund Deposit Factor: (See Sinking Fund Factor.) Sinking Fund Factor: The function of interest rate and time that determines the periodic deposit required to accumulate a specified future amount. Study Period: The length of time that is presumed to be covered in the schedule of events and appraisal of results. Often, it is the anticipated life of the project under consideration, but may be either longer or (more likely) shorter. (See Life and Planning Horizon.)

7477_book.fm Page 235 Tuesday, March 13, 2007 3:34 PM

Appendix A:

Definitions and terms

235

Sunk Cost: A cost that, because it occurred in the past, has no relevance with respect to estimates of future receipts or disbursements. This concept implies that, because a past outlay is the same regardless of the alternative selected, it should not influence a new choice among alternatives. Time Value of Money: (1) The cumulative effect of elapsed time and the money value of an event, based on the earning power of equivalent invested funds and on changes in purchasing power. (2) The expected compound interest rate that capital should or will earn. (See Interest.) Traceable Costs: Cost elements that can be identified with a given product, operation, or service. (See Direct Cost and Marginal Cost.) Uncertainty: (1) That which is indeterminate, indefinite, or problematical. (2) An attribute of the precision of an individual’s or group’s precision of knowledge that is about some fact, event, consequence, or measurement. Uniform Gradient Series: A uniform or arithmetic pattern of receipts or disbursements that is increasing or decreasing by a constant amount in each time period. (See Gradient Factors.) Utility: (1) In economics, a process of evaluating factor inputs and outputs in quantitative units (i.e., utiles) in order to arrive at a single measure of performance to assist in decision making. (2) In economic analysis, a measured preference among various choices available in risk situations based on the decision-making environment, the alternatives being considered, and the decision maker’s personal attitudes. Utility Function: A mathematically derived relationship between utility, measured in utiles, and quantities of money and/or commodities or attributes based on a decision maker’s attitudes and preferences. Valuation or Appraisal: The art and science of estimating the fair-exchange monetary value of specific properties. Variable Cost: A cost that tends to fluctuate according to changes in the number of units produced. (Also see Marginal Cost.) Working Capital: (1) The portion of investment that is represented by current assets (assets that are not capitalized) less the current liabilities. The capital that is necessary to sustain operations as opposed to that invested in fixed assets. (2) Those funds, other than investments in fixed assets, required to make the enterprise or project a going concern. Yield: In evaluating investments, especially those offered by lending institutions, the true annual rate of return to the investor. In bond valuation, the annual dividend of a bond divided by the current market price and usually expressed as a percent. (See Expected Yield.)

7477_book.fm Page 236 Tuesday, March 13, 2007 3:34 PM

7477_book.fm Page 237 Friday, March 30, 2007 2:27 PM

appendix B

Engineering conversion factors Science constants speed of light

velocity of sound gravity (acceleration)

2.997,925 × 1010 cm/sec 983.6 × 106 ft/sec 186,284 miles/sec 340.3 meters/sec 1116 ft/sec 9.80665 m/sec square 32.174 ft/sec square 386.089 inches/sec square

Numbers and prefixes yotta (1024): zetta (1021): exa (1018): peta (1015): tera (1012): giga (109): mega (106): kilo (103): hecto (102): deca (101): deci (10–1): centi (10–2): milli (10–3): micro (10–6): nano (10–9): pico (10–12): femto (10–15): atto (10–18): zepto (10–21): yacto (10–24): Stringo (10–35):

1 000 000 000 000 000 000 000 000 1 000 000 000 000 000 000 000 1 000 000 000 000 000 000 1 000 000 000 000 000 1 000 000 000 000 1 000 000 000 1 000 000 1 000 100 10 0.1 0.01 0.001 0.000 001 0.000 000 001 0.000 000 000 001 0.000 000 000 000 001 0.000 000 000 000 000 001 0.000 000 000 000 000 000 001 0.000 000 000 000 000 000 000 001 0.000 000 000 000 000 000 000 000 000 000 000 01

237

7477_book.fm Page 238 Friday, March 30, 2007 2:27 PM

238

Computational Economic Analysis for Engineering and Industry

Area conversion factors Multiply

by

to obtain

acres

43,560 4,047 4,840 0.405 0.155 144 0.09290 0.1111 645.16 0.3861 10.764 1.196 640 2.590

sq feet sq meters sq yards hectare sq inches sq inches sq meters sq yards sq millimeters sq miles sq feet sq yards acres sq kilometers

sq cm sq feet

sq inches sq kilometers sq meters sq miles

Volume conversion factors Multiply

by

to obtain

acre-foot cubic cm cubic feet

1233.5 0.06102 1728

cubic meters cubic inches cubic inches

7.480 0.02832 0.03704 1.057 0.908

gallons (U.S.) cubic meters cubic yards liquid quarts dry quarts

61.024 231 3.7854 4 0.833 128 0.9463

cubic inches cubic inches liters quarts British gallons U.S. fluid ounces liters

liter

gallons (U.S.)

quarts (U.S.)

7477_book.fm Page 239 Friday, March 30, 2007 2:27 PM

Appendix B:

Engineering conversion factors

Energy conversion factors Multiply

by

to obtain

Btu

1055.9 0.2520 3600 3.409 746 1055.9 1.00

joules kg-calories joules Btu watts watts joules

watt-hour HP (electric) Btu/second watt-second

Mass conversion factors Multiply

by

to obtain

carat grams kilograms ounces

0.200 0.03527 2.2046 28.350

cubic grams ounces pounds grams

pound

16 453.6 6.35 14 907.2 2000 0.893 0.907 2240 1.12 1.016 2,204.623 0.984 1000

ounces grams kilograms pounds kilograms pounds gross ton metric ton pounds net tons metric tons pounds gross pound kilograms

stone (U.K.) ton (net)

ton (gross)

tonne (metric)

Temperature conversion factors Conversion formulas Celsius to Kelvin Celsius to Fahrenheit Fahrenheit to Celsius Fahrenheit to Kelvin Fahrenheit to Rankin Rankin to Kelvin

K = C + 273.15 F = (9/5)C + 32 C = (5/9)(F – 32) K = (5/9)(F + 459.67) R = F + 459.67 K = (5/9)R

239

7477_book.fm Page 240 Friday, March 30, 2007 2:27 PM

240

Computational Economic Analysis for Engineering and Industry

Velocity conversion factors Multiply

by

to obtain

feet/minute feet/second inches/second km/hour meters/second

5.080 0.3048 0.0254 0.6214 3.2808 2.237 88.0 0.44704 1.6093 0.8684 1.151

mm/second meters/second meters/second miles/hour feet/second miles/hour feet/minute meters/second km/hour knots miles/hour

miles/hour

knot

Pressure conversion factors Multiply

by

to obtain

atmospheres

1.01325 33.90 29.92 760.0 75.01 14.50 0.1 1.450 0.06805 2.036 27.708 68.948

bars feet of water inches of mercury mm of mercury cm of mercury pounds/sq inch N/sq meter pounds/sq inch atmospheres inches of mercury inches of water millibars

51.72

mm of mercury

bar dyne/sq cm newtons/sq cm pounds/sq inch

7477_book.fm Page 241 Friday, March 30, 2007 2:27 PM

Appendix B:

Engineering conversion factors

Distance conversion factors Multiply

by –10

to obtain

angstrom feet

10 0.30480 12

meters meters inches

inches

25.40 0.02540 0.08333 3280.8 0.6214 1094 39.370 3.2808 1.094 5280 1.6093 0.8694 0.03937 6076 1.852 0.9144 3 36

millimeters meters feet feet miles yards inches feet yards feet kilometers nautical miles inches feet kilometers meters feet inches

kilometers

meters

miles

millimeters nautical miles yards

241

7477_book.fm Page 242 Friday, March 30, 2007 2:27 PM

242

Computational Economic Analysis for Engineering and Industry

Physical science equations D density m mass V volume

m V

D=

d=v·t

a=

vf − vi t

d = vi ⋅ t +

1 ⋅ a ⋅ t2 2

F=m·a

d v t a vf vi t d vi t a F m a

 g kg   cm 3 = m 3 

distance m velocity m/s time s acceleration m/s2 final velocity m/s initial velocity m/s time s distance m initial velocity m/s time s acceleration m/s2 net force N(=newtons) mass kg acceleration m/s2

K.E. =

Fe =

G ⋅ m1 ⋅ m 2 d2

p=m·v

W=F·d

K.E.kinetic energy m mass kg v velocity m/s Fe electrical force N k Coulomb’s constant 2   9 N⋅m  k = 9 × 10  c2  

k ⋅ Q1 ⋅ Q 2 d2

V

electrical potential difference V(=volts) W work done J Q electric charge moving C I electric current amperes Q electric charge flowing C t time s

W V= Q

I=

2   −11 N − m  G = 6.67 × 10  kg 2  

m1,m2 masses of the two objects kg d separation distance m p momentum kg·m/s m mass v velocity W work J(=joules) F force N d distance m

1 ⋅ m ⋅ v2 2

W(=watts) J s

Q1 · Q2 are electrical charges C d separation distance m

Fg force of gravity N G universal gravitational constant Fg =

P power W work t time

W t

P=

Q t

W=V·I·t

P=V·I

H = c · m · ∆T

W V I t

electrical energy J voltage V current A time s

P V I H m T c

power W voltage V current A heat energy J mass kg change in temperature °C specific heat J/Kg·°C

Units of measurement English system = 12 inches (in) 1’=12” 1 foot (ft) 1 yard (yd) = 3 feet = 1760 yards 1 mile (mi) = 144 sq. inches 1 sq. foot = 9 sq. feet 1 sq. yard = 4840 sq. yards = 43,560 ft2 1 acre 1 sq. mile = 640 acres Note: Prefixes also apply to l (liter) and g (gram).

Metric mm cm dm m dam hm km

system millimeter centimeter decimeter meter dekameter hectometer kilometer

.001 m .01 m .1 m 1m 10 m 100 m 1000 m

7477_book.fm Page 243 Friday, March 30, 2007 2:27 PM

Appendix B:

Engineering conversion factors

Common Notations Measurement

Notation

meter hectare tonne kilogram nautical mile knot liter second hertz candela degree Celsius kelvin pascal joule newton watt ampere volt ohm coulomb

m ha t kg M kn L s Hz cd °C K Pa J N W A V Ω C

Description length area mass mass distance (navigation) speed (navigation) volume or capacity time frequency luminous intensity temperature thermodynamic temp. pressure, stress energy, work force power, radiant flux electric current electric potential electric resistance electric charge

Household Measurements A pinch ................................................... 1/8 tsp. or less 3 tsp........................................................................1 tbsp. 2 tbsp....................................................................... 1/8 c. 4 tbsp....................................................................... 1/4 c. 16 tbsp..........................................................................1 c. 5 tbsp. + 1 tsp. ....................................................... 1/3 c. 4 oz. ......................................................................... 1/2 c. 8 oz. ..............................................................................1 c. 16 oz. .......................................................................... 1 lb. 1 oz. ................................................. 2 tbsp. fat or liquid 1 c. of liquid......................................................... 1/2 pt. 2 c. ..............................................................................1 pt. 2 pt.............................................................................. 1 qt. 4 c. of liquid.............................................................. 1 qt. 4 qts. .................................................................... 1 gallon 8 qts. ..................... 1 peck (such as apples, pears, etc.) 1 jigger .................................................................1 ∫ fl.oz. 1 jigger ...................................................................3 tbsp.

243

7477_book.fm Page 244 Friday, March 30, 2007 2:27 PM

7477_book.fm Page 245 Tuesday, March 13, 2007 3:34 PM

appendix C

Computational and mathematical formulae ∞

∑ n =0



∑ n= 0

 1  xn = 1n  n  1 − x 

k

xn =

x k +1 − 1 , x−1

x≠1

xn =

x − x k +1 , 1− x

x≠1

xn =

x 2 − x k +1 , 1− x

x≠1

∑ n =0

k

∑ n =1

k



xn = ex n!

n =2



∑p

n

n= 0

=

1 , 1− p

if p < 1



∑ nx = (1 −xx ) n

2

,

x≠1

n= 0

245

7477_book.fm Page 246 Tuesday, March 13, 2007 3:34 PM

246

Computational Economic Analysis for Engineering and Industry ∞



n2 x n =

n =0





(1 − x )

n =0

(

+

x

(1 − x )

2

6 x2

+

4

3

(1 − x )

3

x≠1

,

x

+

(1 − x )

2

)

x 1 − M + 1 x M + Mx M +1 

M

∑ nx

(1 − x )

6x3

n3xn =

=

n

2 x2

(1 − x )

2

n =0



  ∑  r +xx − 1 u = (1 − u ) x

−r

x≠1

,

x≠1

,

if u < 1

,

x =0



∑ ( −1)

1 1 1 1 1 1 = 1 − + − + − + ... = 1n 2 k 2 3 4 5 6

k +1

k =1



∑ ( −1) (2 k 1− 1) = 1 − 13 + 15 − 71 + 91 − ... = π4 k +1

k =1



∑ ( −1) x k

k

=

k =0

n

1 , 1+ x

 

∑ ( −1)  nk = 1 , k

−1 < x < 1

for n ≥ 2

k =1

n

∑ k =0

n

2

 n  2 n  k  =  n 

∑ k = 1 + 2 + 3 + ... + n = k =1

n

(

)

n n+1 2

∑ (2 k ) = 2 + 4 + 6 + ... + 2 n = n ( n − 1) k =1

7477_book.fm Page 247 Tuesday, March 13, 2007 3:34 PM

Appendix C:

Computational and mathematical formulae

247

n

∑ (2 k − 1) = 1 + 3 + 5 + ... + (2 n − 1) = n

2

k =1



∑ ( a + kd ) r

(

)

(

)

= a + a + d r + a + 2 d r 2 + ... + =

k

k =0

2

(

= 1 + 4 + 9 + ... + n 2 =

)(

∑k

3

(

)

n2 n + 1

= 1 + 8 + 27 + ... + n = 3

(

)





∑ x1 = 1 + 21 + 13

+ ...(does not converge)

x =1

k

∑ ma = (1 − a) m

(

a

)

1 − k + 1 a k + ka k +1  =  

2

m =0

n

∑ (1) = n k =0

n

 

∑  nk = 2

n

k =0

(

a+b

)

n

n

=



 

∑  nk a b

k n− k

k =0



an = e

 ∞   1 nan    n=1



n =1

 1n  



∏ n =1

 an  = 

2

)

n n + 1 2   n = =    2    k =1

2

4

k =1

)

6

k =1

n

(

n n + 1 2n + 1

n

∑k

a rd + 1− r 1− r



∑ 1na n =1

n

k

∑ ma m =1

m

 k 

2

7477_book.fm Page 248 Tuesday, March 13, 2007 3:34 PM

248

Computational Economic Analysis for Engineering and Industry ∞

k

1  x − 1 , k  x 



()

1n x =

k =1

(

lim 1 + h h→∞

)

1/h

x≥

1 2

=e

n

 x lim  1 +  = e − x n→∞  n n

lim

n→∞

−n

∑ e Kn!

r

1 2

=

k =0

 xk  lim   = 0 k →∞  k !  x+y ≤ x + y x−y ≥ x − y

(

)

1n 1 + x =



∑ ( −1)

k +1

k =1

 xk   k ,  

 1 Γ  =  2

(

if − 1 < x ≤ 1

π

)

( )

Γ α + 1 = αΓ α

 n Γ  =  2





0

(

)

π n−1 ! 2

n −1

 n − 1  2  !

,

n odd

e − x x n−1 dx = Γ( n)

7477_book.fm Page 249 Tuesday, March 13, 2007 3:34 PM

Appendix C:

Computational and mathematical formulae

(

n −1

) ∑K

 n 1 2  2  = 2 n − n = n +  2

249

k =1

1  n   =  2  + n n

2.4.6.8...2 n =

∏ 2k = 2 n ! n

k =1

(

(2 n − 1) ! = 2 n − 1 2 (2 n − 2 ) ! 2

)

1.3.5.7... 2 n − 1 =

2 n −2

2 n −2

n

Derivation of closed form expression for

∑ kx

k

:

k =1

n

n

∑ kx

k

=x

k =1

∑ kx

k −1

k =1

n

=x

∑ dxd  x  k

k =1

=x

n  d   xk  dx  k =1   

=x

n  d  x 1− x dx  1 − x 

∑ (

( (

)   

) )( (

) )

(

)( )

 1 − n + 1 x n 1 − x − x 1 − x n −1   = x 2   − 1 x   =

(

)

x 1 − n + 1 x n + nx n+1 

(1 − x )

2

,

x≠1

7477_book.fm Page 250 Tuesday, March 13, 2007 3:34 PM

7477_book.fm Page 251 Tuesday, March 13, 2007 3:34 PM

appendix D

Units of measure Acre: An area of 43,560 square feet. Agate: 1/14 inch (used in printing for measuring column length). Ampere: Unit of electric current. Astronomical (A.U.): 93,000,000 miles; the average distance of the Earth from the sun (used in astronomy). Bale: A large bundle of goods. In the United States, the approximate weight of a bale of cotton is 500 pounds. Weight of a bale may vary from country to country. Board Foot: 144 cubic inches (12 by 12 by 1 used for lumber). Bolt: 40 yards (used for measuring cloth). Btu: British thermal unit; amount of heat needed to increase the temperature of one pound of water by one degree Fahrenheit (252 calories). Carat: 200 milligrams or 3,086 troy; used for weighing precious stones (originally the weight of a seed of the carob tree in the Mediterranean region). See also Karat. Chain: 66 feet; used in surveying (one mile = 80 chains). Cubit: 18 inches (derived from the distance between the elbow and the tip of the middle finger). Decibel: Unit of relative loudness. Freight Ton: 40 cubic feet of merchandise (used for cargo freight). Gross: 12 dozen (144). Hertz: Unit of measurement of electromagnetic wave frequencies (measures cycles per second). Hogshead: 2 liquid barrels or 14,653 cubic inches. Horsepower: The power needed to lift 33,000 pounds a distance of one foot in one minute (about 1∫ times the power that an average horse can exert); used for measuring the power of mechanical engines. Karat: A measure of the purity of gold. It indicates how many parts out of 24 are pure. 18 karat gold is Ω pure gold. Knot: Rate of speed of 1 nautical mile per hour; used for measuring speed of ships (not distance). League: Approximately 3 miles.

251

7477_book.fm Page 252 Tuesday, March 13, 2007 3:34 PM

252

Computational Economic Analysis for Engineering and Industry

Light year: 5,880,000,000,000 miles; distance traveled by light in one year at the rate of 186,281.7 miles per second; used for measurement of interstellar space. Magnum: Two-quart bottle; used for measuring wine. Ohm: Unit of electrical resistance. Parsec: Approximately 3.26 light years of 19.2 trillion miles; used for measuring interstellar distances. Pi (π): 3.14159265+; the ratio of the circumference of a circle to its diameter. Pica: 1/6 inch or 12 points; used in printing for measuring column width. Pipe: 2 hogsheads; used for measuring wine and other liquids. Point: 0.013837 (approximately 1/72 inch or 1/12 pica); used in printing for measuring type size. Quintal: 100,000 grams or 220.46 pounds avoirdupois. Quire: 24 or 25 sheets; used for measuring paper (20 quires is one ream). Ream: 480 or 500 sheets; used for measuring paper. Roentgen: Dosage unit of radiation exposure produced by x-rays. Score: 20 units. Span: 9 inches or 22.86 cm; derived from the distance between the end of the thumb and the end of the little finger when both are outstretched. Square: 100 square feet; used in building. Stone: 14 pounds avoirdupois in Great Britain. Therm: 100,000 Btus. Township: U.S. land measurement of almost 36 square miles; used in surveying. Tun: 252 gallons (sometimes larger); used for measuring wine and other liquids. Watt: Unit of power.

7477_book.fm Page 253 Tuesday, March 13, 2007 3:34 PM

appendix E

Interest factors and tables Formulas for interest factor Name of Factor Compound Amount (single payment) Present Worth (single payment)

Formula

Table Notation

(1+i)

(F/P, i, N)

(1+i)-N

(P/F, i, N)

N

Sinking Fund

i ( 1 + i) N − 1

(A/F, i, N)

Capital Recovery

i( 1 + i) N ( 1 + i) N − 1

(A/P, i, N)

Compound Amount (uniform series)

( 1 + i) N − 1 i

(F/A, i, N)

Present Worth (uniform series)

( 1 + i) N − 1 i( 1 + i) N

(P/A, i, N)

Arithmetic Gradient to Uniform Series

( 1 + i) N − iN − 1 i( 1 + i) N − i

(A/G, i, N)

Arithmetic Gradient to Present Worth

( 1 + i) N − iN − 1 i 2 ( 1 + i) N

(P/G, i, N)

Geometric Gradient to Present Worth (for ig)

1 − ( 1 + g ) N ( 1 + i) − N i− g

(P/A, g, i, N)

253

7477_book.fm Page 254 Tuesday, March 13, 2007 3:34 PM

254

Computational Economic Analysis for Engineering and Industry

Continuous Compounding Compound Amount (single payment) Continuous Compounding Present Worth (single payment)

erN

(F/P, r, N)

e-rN

(P/F, r, N)

e rN − 1 e rN ( e r − 1)

(P/A, r, N)

Continuous Compounding Sinking Fund

er − 1 e rN − 1

(A/F, r, N)

Continuous Compounding Capital Recovery

e rN ( e r − 1) e rN − 1

(A/P, r, N)

e rN − 1 er − 1

(F/A, r, N)

i( 1 + i)− N ln( 1 + i)

– (P/F, i, N)

i( 1 + i) N −1 ln( 1 + i)

– (F/P, i, N)

ln( 1 + i) ( 1 + i) N − 1

– (A/F, i, N)

Continuous Compounding Present Worth (uniform series)

Continuous Compounding Compound Amount (uniform series) Continuous Compounding Present Worth (single, continuous payment) Continuous Compounding Compound Amount (single, continuous payment) Continuous Compounding Sinking Fund (continuous, uniform payments)

Continuous Compounding Capital Recovery (continuous, uniform payments) Continuous Compounding Compound Amount (continuous, uniform payments) Continuous Compounding Present Worth (continuous, uniform payments)

(1 + i)N ln(1 + i) (1 + i)N − 1

– (A/P, i, N)

( 1 + i) N − 1 ln( 1 + i)

( F A , i, N)

( 1 + i) N − 1 ( 1 + i) N ln( 1 + i)

( P A , i, N)

7477_book.fm Page 255 Tuesday, March 13, 2007 3:34 PM

Appendix E:

Interest factors and tables

255

Summation formulas for closed-form expressions n



xt =

1 − x n +1 1− x

xt =

1 − xn 1− x

xt =

x − x n +1 1− x

xt =

x − xn 1− x

t= 0

n −1

∑ t= 0

n

∑ t= 1

n −1

∑ t= 1

n

∑x

t

=

t= 2

n



xt =

x 2 − x n +1 1− x

x 1 − ( n + 1) x n + nx n+1  ( 1 − x )2

t= 0

∑ t= 1

tx t =



tx t − 0 x 0 =

t= 0

∑ t= 0

=

x − ( 1 − x )( n + 1) x n+1 − x n+2 ( 1 − x )2

n

n

n

n

=

tx t − 0 =

∑ tx

t

t= 0

x 1 − ( n + 1) x n + nx n+1  (1 − x)

2

=

x − ( 1 − x )( n + 1) x n+1 − x n+2 ( 1 − x )2

7477_book.fm Page 256 Tuesday, March 13, 2007 3:34 PM

256

Computational Economic Analysis for Engineering and Industry

Interest tables 0.25% Period

n 1 2 3 4 5

Compound Interest Factors Single Payment Uniform Payment Series Compound Present Sinking Capital Compound Amount Value Fund Recovery Amount Factor Factor Factor Factor Factor Find F Find P Find A Find A Find F Given P Given F Given F Given P Given A F/P P/F A/F A/P F/A 1.003 0.9975 1.0000 1.0025 1.000 1.005 0.9950 0.4994 0.5019 2.002 1.008 0.9925 0.3325 0.3350 3.008 1.010 0.9901 0.2491 0.2516 4.015 1.013 0.9876 0.1990 0.2015 5.025

0.25%

Arithmetic Gradient Period Present Gradient Gradient Value Uniform Present Factor Series Value Find P Find A Find P Given A Given G Given G P/A A/G P/G n 0.998 0.000 0.000 1 1.993 0.499 0.995 2 2.985 0.998 2.980 3 3.975 1.497 5.950 4 4.963 1.995 9.901 5

6 7 8 9 10

1.015 1.018 1.020 1.023 1.025

0.9851 0.9827 0.9802 0.9778 0.9753

0.1656 0.1418 0.1239 0.1100 0.0989

0.1681 0.1443 0.1264 0.1125 0.1014

6.038 7.053 8.070 9.091 10.113

5.948 6.931 7.911 8.889 9.864

2.493 2.990 3.487 3.983 4.479

14.826 20.722 27.584 35.406 44.184

6 7 8 9 10

11 12 13 14 15

1.028 1.030 1.033 1.036 1.038

0.9729 0.9705 0.9681 0.9656 0.9632

0.0898 0.0822 0.0758 0.0703 0.0655

0.0923 0.0847 0.0783 0.0728 0.0680

11.139 12.166 13.197 14.230 15.265

10.837 11.807 12.775 13.741 14.704

4.975 5.470 5.965 6.459 6.953

53.913 64.589 76.205 88.759 102.244

11 12 13 14 15

16 17 18 19 20

1.041 1.043 1.046 1.049 1.051

0.9608 0.9584 0.9561 0.9537 0.9513

0.0613 0.0577 0.0544 0.0515 0.0488

0.0638 0.0602 0.0569 0.0540 0.0513

16.304 17.344 18.388 19.434 20.482

15.665 16.623 17.580 18.533 19.484

7.447 7.940 8.433 8.925 9.417

116.657 131.992 148.245 165.411 183.485

16 17 18 19 20

21 22 23 24 25

1.054 1.056 1.059 1.062 1.064

0.9489 0.9466 0.9442 0.9418 0.9395

0.0464 0.0443 0.0423 0.0405 0.0388

0.0489 0.0468 0.0448 0.0430 0.0413

21.533 22.587 23.644 24.703 25.765

20.433 21.380 22.324 23.266 24.205

9.908 10.400 10.890 11.380 11.870

202.463 222.341 243.113 264.775 287.323

21 22 23 24 25

26 27 28 29 30

1.067 1.070 1.072 1.075 1.078

0.9371 0.9348 0.9325 0.9301 0.9278

0.0373 0.0358 0.0345 0.0333 0.0321

0.0398 0.0383 0.0370 0.0358 0.0346

26.829 27.896 28.966 30.038 31.113

25.143 26.077 27.010 27.940 28.868

12.360 12.849 13.337 13.825 14.313

310.752 335.057 360.233 386.278 413.185

26 27 28 29 30

31 32 33 34 35

1.080 1.083 1.086 1.089 1.091

0.9255 0.9232 0.9209 0.9186 0.9163

0.0311 0.0301 0.0291 0.0282 0.0274

0.0336 0.0326 0.0316 0.0307 0.0299

32.191 33.272 34.355 35.441 36.529

29.793 30.717 31.638 32.556 33.472

14.800 15.287 15.774 16.260 16.745

440.950 469.570 499.039 529.353 560.508

31 32 33 34 35

36 37 38 39 40

1.094 1.097 1.100 1.102 1.105

0.9140 0.9118 0.9095 0.9072 0.9050

0.0266 0.0258 0.0251 0.0244 0.0238

0.0291 0.0283 0.0276 0.0269 0.0263

37.621 38.715 39.811 40.911 42.013

34.386 35.298 36.208 37.115 38.020

17.231 17.715 18.200 18.684 19.167

592.499 625.322 658.973 693.447 728.740

36 37 38 39 40

41 42 43 44 45

1.108 1.111 1.113 1.116 1.119

0.9027 0.9004 0.8982 0.8960 0.8937

0.0232 0.0226 0.0221 0.0215 0.0210

0.0257 0.0251 0.0246 0.0240 0.0235

43.118 44.226 45.337 46.450 47.566

38.923 39.823 40.721 41.617 42.511

19.650 20.133 20.616 21.097 21.579

764.848 801.766 839.490 878.016 917.340

41 42 43 44 45

46 47 48 49 50

1.122 1.125 1.127 1.130 1.133

0.8915 0.8893 0.8871 0.8848 0.8826

0.0205 0.0201 0.0196 0.0192 0.0188

0.0230 0.0226 0.0221 0.0217 0.0213

48.685 49.807 50.931 52.059 53.189

43.402 44.292 45.179 46.064 46.946

22.060 22.541 23.021 23.501 23.980

957.457 998.364 1040.055 1082.528 1125.777

46 47 48 49 50

7477_book.fm Page 257 Tuesday, March 13, 2007 3:34 PM

Appendix E:

Interest factors and tables

0.5% Period

n 1 2 3 4 5

257

Compound Interest Factors Single Payment Uniform Payment Series Compound Present Sinking Capital Compound Amount Value Fund Recovery Amount Factor Factor Factor Factor Factor Find F Find P Find A Find A Find F Given P Given F Given F Given P Given A F/P P/F A/F A/P F/A 1.005 0.9950 1.0000 1.0050 1.000 1.010 0.9901 0.4988 0.5038 2.005 1.015 0.9851 0.3317 0.3367 3.015 1.020 0.9802 0.2481 0.2531 4.030 1.025 0.9754 0.1980 0.2030 5.050

0.5%

Arithmetic Gradient Period Present Gradient Gradient Value Uniform Present Factor Series Value Find P Find A Find P Given A Given G Given G P/A A/G P/G n 0.995 0.000 0.000 1 1.985 0.499 0.990 2 2.970 0.997 2.960 3 3.950 1.494 5.901 4 4.926 1.990 9.803 5

6 7 8 9 10

1.030 1.036 1.041 1.046 1.051

0.9705 0.9657 0.9609 0.9561 0.9513

0.1646 0.1407 0.1228 0.1089 0.0978

0.1696 0.1457 0.1278 0.1139 0.1028

6.076 7.106 8.141 9.182 10.228

5.896 6.862 7.823 8.779 9.730

2.485 2.980 3.474 3.967 4.459

14.655 20.449 27.176 34.824 43.386

6 7 8 9 10

11 12 13 14 15

1.056 1.062 1.067 1.072 1.078

0.9466 0.9419 0.9372 0.9326 0.9279

0.0887 0.0811 0.0746 0.0691 0.0644

0.0937 0.0861 0.0796 0.0741 0.0694

11.279 12.336 13.397 14.464 15.537

10.677 11.619 12.556 13.489 14.417

4.950 5.441 5.930 6.419 6.907

52.853 63.214 74.460 86.583 99.574

11 12 13 14 15

16 17 18 19 20

1.083 1.088 1.094 1.099 1.105

0.9233 0.9187 0.9141 0.9096 0.9051

0.0602 0.0565 0.0532 0.0503 0.0477

0.0652 0.0615 0.0582 0.0553 0.0527

16.614 17.697 18.786 19.880 20.979

15.340 16.259 17.173 18.082 18.987

7.394 7.880 8.366 8.850 9.334

113.424 128.123 143.663 160.036 177.232

16 17 18 19 20

21 22 23 24 25

1.110 1.116 1.122 1.127 1.133

0.9006 0.8961 0.8916 0.8872 0.8828

0.0453 0.0431 0.0411 0.0393 0.0377

0.0503 0.0481 0.0461 0.0443 0.0427

22.084 23.194 24.310 25.432 26.559

19.888 20.784 21.676 22.563 23.446

9.817 10.299 10.781 11.261 11.741

195.243 214.061 233.677 254.082 275.269

21 22 23 24 25

26 27 28 29 30

1.138 1.144 1.150 1.156 1.161

0.8784 0.8740 0.8697 0.8653 0.8610

0.0361 0.0347 0.0334 0.0321 0.0310

0.0411 0.0397 0.0384 0.0371 0.0360

27.692 28.830 29.975 31.124 32.280

24.324 25.198 26.068 26.933 27.794

12.220 12.698 13.175 13.651 14.126

297.228 319.952 343.433 367.663 392.632

26 27 28 29 30

31 32 33 34 35

1.167 1.173 1.179 1.185 1.191

0.8567 0.8525 0.8482 0.8440 0.8398

0.0299 0.0289 0.0279 0.0271 0.0262

0.0349 0.0339 0.0329 0.0321 0.0312

33.441 34.609 35.782 36.961 38.145

28.651 29.503 30.352 31.196 32.035

14.601 15.075 15.548 16.020 16.492

418.335 444.762 471.906 499.758 528.312

31 32 33 34 35

36 37 38 39 40

1.197 1.203 1.209 1.215 1.221

0.8356 0.8315 0.8274 0.8232 0.8191

0.0254 0.0247 0.0240 0.0233 0.0226

0.0304 0.0297 0.0290 0.0283 0.0276

39.336 40.533 41.735 42.944 44.159

32.871 33.703 34.530 35.353 36.172

16.962 17.432 17.901 18.369 18.836

557.560 587.493 618.105 649.388 681.335

36 37 38 39 40

41 42 43 44 45

1.227 1.233 1.239 1.245 1.252

0.8151 0.8110 0.8070 0.8030 0.7990

0.0220 0.0215 0.0209 0.0204 0.0199

0.0270 0.0265 0.0259 0.0254 0.0249

45.380 46.607 47.840 49.079 50.324

36.987 37.798 38.605 39.408 40.207

19.302 19.768 20.233 20.696 21.159

713.937 747.189 781.081 815.609 850.763

41 42 43 44 45

46 47 48 49 50

1.258 1.264 1.270 1.277 1.283

0.7950 0.7910 0.7871 0.7832 0.7793

0.0194 0.0189 0.0185 0.0181 0.0177

0.0244 0.0239 0.0235 0.0231 0.0227

51.576 52.834 54.098 55.368 56.645

41.002 41.793 42.580 43.364 44.143

21.622 22.083 22.544 23.003 23.462

886.538 922.925 959.919 997.512 1035.697

46 47 48 49 50

7477_book.fm Page 258 Tuesday, March 13, 2007 3:34 PM

258

Computational Economic Analysis for Engineering and Industry

0.75% Period

n 1 2 3 4 5

Compound Interest Factors Single Payment Uniform Payment Series Compound Present Sinking Capital Compound Amount Value Fund Recovery Amount Factor Factor Factor Factor Factor Find F Find P Find A Find A Find F Given P Given F Given F Given P Given A F/P P/F A/F A/P F/A 1.008 0.9926 1.0000 1.0075 1.000 1.015 0.9852 0.4981 0.5056 2.008 1.023 0.9778 0.3308 0.3383 3.023 1.030 0.9706 0.2472 0.2547 4.045 1.038 0.9633 0.1970 0.2045 5.076

0.75%

Arithmetic Gradient Period Present Gradient Gradient Value Uniform Present Factor Series Value Find P Find A Find P Given A Given G Given G P/A A/G P/G n 0.993 0.000 0.000 1 1.978 0.498 0.985 2 2.956 0.995 2.941 3 3.926 1.491 5.852 4 4.889 1.985 9.706 5

6 7 8 9 10

1.046 1.054 1.062 1.070 1.078

0.9562 0.9490 0.9420 0.9350 0.9280

0.1636 0.1397 0.1218 0.1078 0.0967

0.1711 0.1472 0.1293 0.1153 0.1042

6.114 7.159 8.213 9.275 10.344

5.846 6.795 7.737 8.672 9.600

2.478 2.970 3.461 3.950 4.438

14.487 20.181 26.775 34.254 42.606

6 7 8 9 10

11 12 13 14 15

1.086 1.094 1.102 1.110 1.119

0.9211 0.9142 0.9074 0.9007 0.8940

0.0876 0.0800 0.0735 0.0680 0.0632

0.0951 0.0875 0.0810 0.0755 0.0707

11.422 12.508 13.601 14.703 15.814

10.521 11.435 12.342 13.243 14.137

4.925 5.411 5.895 6.379 6.861

51.817 61.874 72.763 84.472 96.988

11 12 13 14 15

16 17 18 19 20

1.127 1.135 1.144 1.153 1.161

0.8873 0.8807 0.8742 0.8676 0.8612

0.0591 0.0554 0.0521 0.0492 0.0465

0.0666 0.0629 0.0596 0.0567 0.0540

16.932 18.059 19.195 20.339 21.491

15.024 15.905 16.779 17.647 18.508

7.341 7.821 8.299 8.776 9.252

110.297 124.389 139.249 154.867 171.230

16 17 18 19 20

21 22 23 24 25

1.170 1.179 1.188 1.196 1.205

0.8548 0.8484 0.8421 0.8358 0.8296

0.0441 0.0420 0.0400 0.0382 0.0365

0.0516 0.0495 0.0475 0.0457 0.0440

22.652 23.822 25.001 26.188 27.385

19.363 20.211 21.053 21.889 22.719

9.726 10.199 10.671 11.142 11.612

188.325 206.142 224.668 243.892 263.803

21 22 23 24 25

26 27 28 29 30

1.214 1.224 1.233 1.242 1.251

0.8234 0.8173 0.8112 0.8052 0.7992

0.0350 0.0336 0.0322 0.0310 0.0298

0.0425 0.0411 0.0397 0.0385 0.0373

28.590 29.805 31.028 32.261 33.503

23.542 24.359 25.171 25.976 26.775

12.080 12.547 13.013 13.477 13.941

284.389 305.639 327.542 350.087 373.263

26 27 28 29 30

31 32 33 34 35

1.261 1.270 1.280 1.289 1.299

0.7932 0.7873 0.7815 0.7757 0.7699

0.0288 0.0278 0.0268 0.0259 0.0251

0.0363 0.0353 0.0343 0.0334 0.0326

34.754 36.015 37.285 38.565 39.854

27.568 28.356 29.137 29.913 30.683

14.403 14.864 15.323 15.782 16.239

397.060 421.468 446.475 472.071 498.247

31 32 33 34 35

36 37 38 39 40

1.309 1.318 1.328 1.338 1.348

0.7641 0.7585 0.7528 0.7472 0.7416

0.0243 0.0236 0.0228 0.0222 0.0215

0.0318 0.0311 0.0303 0.0297 0.0290

41.153 42.461 43.780 45.108 46.446

31.447 32.205 32.958 33.705 34.447

16.695 17.149 17.603 18.055 18.506

524.992 552.297 580.151 608.545 637.469

36 37 38 39 40

41 42 43 44 45

1.358 1.369 1.379 1.389 1.400

0.7361 0.7306 0.7252 0.7198 0.7145

0.0209 0.0203 0.0198 0.0193 0.0188

0.0284 0.0278 0.0273 0.0268 0.0263

47.795 49.153 50.522 51.901 53.290

35.183 35.914 36.639 37.359 38.073

18.956 19.404 19.851 20.297 20.742

666.914 696.871 727.330 758.281 789.717

41 42 43 44 45

46 47 48 49 50

1.410 1.421 1.431 1.442 1.453

0.7091 0.7039 0.6986 0.6934 0.6883

0.0183 0.0178 0.0174 0.0170 0.0166

0.0258 0.0253 0.0249 0.0245 0.0241

54.690 56.100 57.521 58.952 60.394

38.782 39.486 40.185 40.878 41.566

21.186 21.628 22.069 22.509 22.948

821.628 854.006 886.840 920.124 953.849

46 47 48 49 50

7477_book.fm Page 259 Tuesday, March 13, 2007 3:34 PM

Appendix E:

Interest factors and tables

1% Period

n 1 2 3 4 5

259

Compound Interest Factors Single Payment Uniform Payment Series Compound Present Sinking Capital Compound Amount Value Fund Recovery Amount Factor Factor Factor Factor Factor Find F Find P Find A Find A Find F Given P Given F Given F Given P Given A F/P P/F A/F A/P F/A 1.010 0.9901 1.0000 1.0100 1.000 1.020 0.9803 0.4975 0.5075 2.010 1.030 0.9706 0.3300 0.3400 3.030 1.041 0.9610 0.2463 0.2563 4.060 1.051 0.9515 0.1960 0.2060 5.101

1%

Arithmetic Gradient Period Present Gradient Gradient Value Uniform Present Factor Series Value Find P Find A Find P Given A Given G Given G P/A A/G P/G n 0.990 0.000 0.000 1 1.970 0.498 0.980 2 2.941 0.993 2.921 3 3.902 1.488 5.804 4 4.853 1.980 9.610 5

6 7 8 9 10

1.062 1.072 1.083 1.094 1.105

0.9420 0.9327 0.9235 0.9143 0.9053

0.1625 0.1386 0.1207 0.1067 0.0956

0.1725 0.1486 0.1307 0.1167 0.1056

6.152 7.214 8.286 9.369 10.462

5.795 6.728 7.652 8.566 9.471

2.471 2.960 3.448 3.934 4.418

14.321 19.917 26.381 33.696 41.843

6 7 8 9 10

11 12 13 14 15

1.116 1.127 1.138 1.149 1.161

0.8963 0.8874 0.8787 0.8700 0.8613

0.0865 0.0788 0.0724 0.0669 0.0621

0.0965 0.0888 0.0824 0.0769 0.0721

11.567 12.683 13.809 14.947 16.097

10.368 11.255 12.134 13.004 13.865

4.901 5.381 5.861 6.338 6.814

50.807 60.569 71.113 82.422 94.481

11 12 13 14 15

16 17 18 19 20

1.173 1.184 1.196 1.208 1.220

0.8528 0.8444 0.8360 0.8277 0.8195

0.0579 0.0543 0.0510 0.0481 0.0454

0.0679 0.0643 0.0610 0.0581 0.0554

17.258 18.430 19.615 20.811 22.019

14.718 15.562 16.398 17.226 18.046

7.289 7.761 8.232 8.702 9.169

107.273 120.783 134.996 149.895 165.466

16 17 18 19 20

21 22 23 24 25

1.232 1.245 1.257 1.270 1.282

0.8114 0.8034 0.7954 0.7876 0.7798

0.0430 0.0409 0.0389 0.0371 0.0354

0.0530 0.0509 0.0489 0.0471 0.0454

23.239 24.472 25.716 26.973 28.243

18.857 19.660 20.456 21.243 22.023

9.635 10.100 10.563 11.024 11.483

181.695 198.566 216.066 234.180 252.894

21 22 23 24 25

26 27 28 29 30

1.295 1.308 1.321 1.335 1.348

0.7720 0.7644 0.7568 0.7493 0.7419

0.0339 0.0324 0.0311 0.0299 0.0287

0.0439 0.0424 0.0411 0.0399 0.0387

29.526 30.821 32.129 33.450 34.785

22.795 23.560 24.316 25.066 25.808

11.941 12.397 12.852 13.304 13.756

272.196 292.070 312.505 333.486 355.002

26 27 28 29 30

31 32 33 34 35

1.361 1.375 1.389 1.403 1.417

0.7346 0.7273 0.7201 0.7130 0.7059

0.0277 0.0267 0.0257 0.0248 0.0240

0.0377 0.0367 0.0357 0.0348 0.0340

36.133 37.494 38.869 40.258 41.660

26.542 27.270 27.990 28.703 29.409

14.205 14.653 15.099 15.544 15.987

377.039 399.586 422.629 446.157 470.158

31 32 33 34 35

36 37 38 39 40

1.431 1.445 1.460 1.474 1.489

0.6989 0.6920 0.6852 0.6784 0.6717

0.0232 0.0225 0.0218 0.0211 0.0205

0.0332 0.0325 0.0318 0.0311 0.0305

43.077 44.508 45.953 47.412 48.886

30.108 30.800 31.485 32.163 32.835

16.428 16.868 17.306 17.743 18.178

494.621 519.533 544.884 570.662 596.856

36 37 38 39 40

41 42 43 44 45

1.504 1.519 1.534 1.549 1.565

0.6650 0.6584 0.6519 0.6454 0.6391

0.0199 0.0193 0.0187 0.0182 0.0177

0.0299 0.0293 0.0287 0.0282 0.0277

50.375 51.879 53.398 54.932 56.481

33.500 34.158 34.810 35.455 36.095

18.611 19.042 19.472 19.901 20.327

623.456 650.451 677.831 705.585 733.704

41 42 43 44 45

46 47 48 49 50

1.580 1.596 1.612 1.628 1.645

0.6327 0.6265 0.6203 0.6141 0.6080

0.0172 0.0168 0.0163 0.0159 0.0155

0.0272 0.0268 0.0263 0.0259 0.0255

58.046 59.626 61.223 62.835 64.463

36.727 37.354 37.974 38.588 39.196

20.752 21.176 21.598 22.018 22.436

762.176 790.994 820.146 849.624 879.418

46 47 48 49 50

7477_book.fm Page 260 Tuesday, March 13, 2007 3:34 PM

260

Computational Economic Analysis for Engineering and Industry

1.25% Period

n 1 2 3 4 5

Compound Interest Factors Single Payment Uniform Payment Series Compound Present Sinking Capital Compound Amount Value Fund Recovery Amount Factor Factor Factor Factor Factor Find F Find P Find A Find A Find F Given P Given F Given F Given P Given A F/P P/F A/F A/P F/A 1.013 0.9877 1.0000 1.0125 1.000 1.025 0.9755 0.4969 0.5094 2.013 1.038 0.9634 0.3292 0.3417 3.038 1.051 0.9515 0.2454 0.2579 4.076 1.064 0.9398 0.1951 0.2076 5.127

1.25%

Arithmetic Gradient Period Present Gradient Gradient Value Uniform Present Factor Series Value Find P Find A Find P Given A Given G Given G P/A A/G P/G n 0.988 0.000 0.000 1 1.963 0.497 0.975 2 2.927 0.992 2.902 3 3.878 1.484 5.757 4 4.818 1.975 9.516 5

6 7 8 9 10

1.077 1.091 1.104 1.118 1.132

0.9282 0.9167 0.9054 0.8942 0.8832

0.1615 0.1376 0.1196 0.1057 0.0945

0.1740 0.1501 0.1321 0.1182 0.1070

6.191 7.268 8.359 9.463 10.582

5.746 6.663 7.568 8.462 9.346

2.464 2.950 3.435 3.917 4.398

14.157 19.657 25.995 33.149 41.097

6 7 8 9 10

11 12 13 14 15

1.146 1.161 1.175 1.190 1.205

0.8723 0.8615 0.8509 0.8404 0.8300

0.0854 0.0778 0.0713 0.0658 0.0610

0.0979 0.0903 0.0838 0.0783 0.0735

11.714 12.860 14.021 15.196 16.386

10.218 11.079 11.930 12.771 13.601

4.876 5.352 5.826 6.298 6.768

49.820 59.297 69.507 80.432 92.052

11 12 13 14 15

16 17 18 19 20

1.220 1.235 1.251 1.266 1.282

0.8197 0.8096 0.7996 0.7898 0.7800

0.0568 0.0532 0.0499 0.0470 0.0443

0.0693 0.0657 0.0624 0.0595 0.0568

17.591 18.811 20.046 21.297 22.563

14.420 15.230 16.030 16.819 17.599

7.236 7.702 8.166 8.628 9.087

104.348 117.302 130.896 145.111 159.932

16 17 18 19 20

21 22 23 24 25

1.298 1.314 1.331 1.347 1.364

0.7704 0.7609 0.7515 0.7422 0.7330

0.0419 0.0398 0.0378 0.0360 0.0343

0.0544 0.0523 0.0503 0.0485 0.0468

23.845 25.143 26.457 27.788 29.135

18.370 19.131 19.882 20.624 21.357

9.545 10.001 10.454 10.906 11.355

175.339 191.317 207.850 224.920 242.513

21 22 23 24 25

26 27 28 29 30

1.381 1.399 1.416 1.434 1.452

0.7240 0.7150 0.7062 0.6975 0.6889

0.0328 0.0314 0.0300 0.0288 0.0277

0.0453 0.0439 0.0425 0.0413 0.0402

30.500 31.881 33.279 34.695 36.129

22.081 22.796 23.503 24.200 24.889

11.802 12.248 12.691 13.132 13.571

260.613 279.204 298.272 317.802 337.780

26 27 28 29 30

31 32 33 34 35

1.470 1.488 1.507 1.526 1.545

0.6804 0.6720 0.6637 0.6555 0.6474

0.0266 0.0256 0.0247 0.0238 0.0230

0.0391 0.0381 0.0372 0.0363 0.0355

37.581 39.050 40.539 42.045 43.571

25.569 26.241 26.905 27.560 28.208

14.009 14.444 14.877 15.308 15.737

358.191 379.023 400.261 421.892 443.904

31 32 33 34 35

36 37 38 39 40

1.564 1.583 1.603 1.623 1.644

0.6394 0.6315 0.6237 0.6160 0.6084

0.0222 0.0214 0.0207 0.0201 0.0194

0.0347 0.0339 0.0332 0.0326 0.0319

45.116 46.679 48.263 49.866 51.490

28.847 29.479 30.103 30.719 31.327

16.164 16.589 17.012 17.433 17.851

466.283 489.018 512.095 535.504 559.232

36 37 38 39 40

41 42 43 44 45

1.664 1.685 1.706 1.727 1.749

0.6009 0.5935 0.5862 0.5789 0.5718

0.0188 0.0182 0.0177 0.0172 0.0167

0.0313 0.0307 0.0302 0.0297 0.0292

53.133 54.797 56.482 58.188 59.916

31.928 32.521 33.107 33.686 34.258

18.268 18.683 19.096 19.507 19.916

583.268 607.601 632.219 657.113 682.271

41 42 43 44 45

46 47 48 49 50

1.771 1.793 1.815 1.838 1.861

0.5647 0.5577 0.5509 0.5441 0.5373

0.0162 0.0158 0.0153 0.0149 0.0145

0.0287 0.0283 0.0278 0.0274 0.0270

61.665 63.435 65.228 67.044 68.882

34.823 35.381 35.931 36.476 37.013

20.322 20.727 21.130 21.531 21.929

707.683 733.339 759.230 785.344 811.674

46 47 48 49 50

7477_book.fm Page 261 Tuesday, March 13, 2007 3:34 PM

Appendix E:

Interest factors and tables

1.5% Period

n 1 2 3 4 5

261

Compound Interest Factors Single Payment Uniform Payment Series Compound Present Sinking Capital Compound Amount Value Fund Recovery Amount Factor Factor Factor Factor Factor Find F Find P Find A Find A Find F Given P Given F Given F Given P Given A F/P P/F A/F A/P F/A 1.015 0.9852 1.0000 1.0150 1.000 1.030 0.9707 0.4963 0.5113 2.015 1.046 0.9563 0.3284 0.3434 3.045 1.061 0.9422 0.2444 0.2594 4.091 1.077 0.9283 0.1941 0.2091 5.152

1.5%

Arithmetic Gradient Period Present Gradient Gradient Value Uniform Present Factor Series Value Find P Find A Find P Given A Given G Given G P/A A/G P/G n 0.985 0.000 0.000 1 1.956 0.496 0.971 2 2.912 0.990 2.883 3 3.854 1.481 5.710 4 4.783 1.970 9.423 5

6 7 8 9 10

1.093 1.110 1.126 1.143 1.161

0.9145 0.9010 0.8877 0.8746 0.8617

0.1605 0.1366 0.1186 0.1046 0.0934

0.1755 0.1516 0.1336 0.1196 0.1084

6.230 7.323 8.433 9.559 10.703

5.697 6.598 7.486 8.361 9.222

2.457 2.940 3.422 3.901 4.377

13.996 19.402 25.616 32.612 40.367

6 7 8 9 10

11 12 13 14 15

1.178 1.196 1.214 1.232 1.250

0.8489 0.8364 0.8240 0.8118 0.7999

0.0843 0.0767 0.0702 0.0647 0.0599

0.0993 0.0917 0.0852 0.0797 0.0749

11.863 13.041 14.237 15.450 16.682

10.071 10.908 11.732 12.543 13.343

4.851 5.323 5.792 6.258 6.722

48.857 58.057 67.945 78.499 89.697

11 12 13 14 15

16 17 18 19 20

1.269 1.288 1.307 1.327 1.347

0.7880 0.7764 0.7649 0.7536 0.7425

0.0558 0.0521 0.0488 0.0459 0.0432

0.0708 0.0671 0.0638 0.0609 0.0582

17.932 19.201 20.489 21.797 23.124

14.131 14.908 15.673 16.426 17.169

7.184 7.643 8.100 8.554 9.006

101.518 113.940 126.943 140.508 154.615

16 17 18 19 20

21 22 23 24 25

1.367 1.388 1.408 1.430 1.451

0.7315 0.7207 0.7100 0.6995 0.6892

0.0409 0.0387 0.0367 0.0349 0.0333

0.0559 0.0537 0.0517 0.0499 0.0483

24.471 25.838 27.225 28.634 30.063

17.900 18.621 19.331 20.030 20.720

9.455 9.902 10.346 10.788 11.228

169.245 184.380 200.001 216.090 232.631

21 22 23 24 25

26 27 28 29 30

1.473 1.495 1.517 1.540 1.563

0.6790 0.6690 0.6591 0.6494 0.6398

0.0317 0.0303 0.0290 0.0278 0.0266

0.0467 0.0453 0.0440 0.0428 0.0416

31.514 32.987 34.481 35.999 37.539

21.399 22.068 22.727 23.376 24.016

11.665 12.099 12.531 12.961 13.388

249.607 267.000 284.796 302.978 321.531

26 27 28 29 30

31 32 33 34 35

1.587 1.610 1.634 1.659 1.684

0.6303 0.6210 0.6118 0.6028 0.5939

0.0256 0.0246 0.0236 0.0228 0.0219

0.0406 0.0396 0.0386 0.0378 0.0369

39.102 40.688 42.299 43.933 45.592

24.646 25.267 25.879 26.482 27.076

13.813 14.236 14.656 15.073 15.488

340.440 359.691 379.269 399.161 419.352

31 32 33 34 35

36 37 38 39 40

1.709 1.735 1.761 1.787 1.814

0.5851 0.5764 0.5679 0.5595 0.5513

0.0212 0.0204 0.0197 0.0191 0.0184

0.0362 0.0354 0.0347 0.0341 0.0334

47.276 48.985 50.720 52.481 54.268

27.661 28.237 28.805 29.365 29.916

15.901 16.311 16.719 17.125 17.528

439.830 460.582 481.595 502.858 524.357

36 37 38 39 40

41 42 43 44 45

1.841 1.869 1.897 1.925 1.954

0.5431 0.5351 0.5272 0.5194 0.5117

0.0178 0.0173 0.0167 0.0162 0.0157

0.0328 0.0323 0.0317 0.0312 0.0307

56.082 57.923 59.792 61.689 63.614

30.459 30.994 31.521 32.041 32.552

17.928 18.327 18.723 19.116 19.507

546.081 568.020 590.162 612.496 635.011

41 42 43 44 45

46 47 48 49 50

1.984 2.013 2.043 2.074 2.105

0.5042 0.4967 0.4894 0.4821 0.4750

0.0153 0.0148 0.0144 0.0140 0.0136

0.0303 0.0298 0.0294 0.0290 0.0286

65.568 67.552 69.565 71.609 73.683

33.056 33.553 34.043 34.525 35.000

19.896 20.283 20.667 21.048 21.428

657.698 680.546 703.546 726.688 749.964

46 47 48 49 50

7477_book.fm Page 262 Tuesday, March 13, 2007 3:34 PM

262

Computational Economic Analysis for Engineering and Industry

1.75% Period

n 1 2 3 4 5

Compound Interest Factors Single Payment Uniform Payment Series Compound Present Sinking Capital Compound Amount Value Fund Recovery Amount Factor Factor Factor Factor Factor Find F Find P Find A Find A Find F Given P Given F Given F Given P Given A F/P P/F A/F A/P F/A 1.018 0.9828 1.0000 1.0175 1.000 1.035 0.9659 0.4957 0.5132 2.018 1.053 0.9493 0.3276 0.3451 3.053 1.072 0.9330 0.2435 0.2610 4.106 1.091 0.9169 0.1931 0.2106 5.178

1.75%

Arithmetic Gradient Period Present Gradient Gradient Value Uniform Present Factor Series Value Find P Find A Find P Given A Given G Given G P/A A/G P/G n 0.983 0.000 0.000 1 1.949 0.496 0.966 2 2.898 0.988 2.864 3 3.831 1.478 5.663 4 4.748 1.965 9.331 5

6 7 8 9 10

1.110 1.129 1.149 1.169 1.189

0.9011 0.8856 0.8704 0.8554 0.8407

0.1595 0.1355 0.1175 0.1036 0.0924

0.1770 0.1530 0.1350 0.1211 0.1099

6.269 7.378 8.508 9.656 10.825

5.649 6.535 7.405 8.260 9.101

2.449 2.931 3.409 3.884 4.357

13.837 19.151 25.243 32.087 39.654

6 7 8 9 10

11 12 13 14 15

1.210 1.231 1.253 1.275 1.297

0.8263 0.8121 0.7981 0.7844 0.7709

0.0832 0.0756 0.0692 0.0637 0.0589

0.1007 0.0931 0.0867 0.0812 0.0764

12.015 13.225 14.457 15.710 16.984

9.927 10.740 11.538 12.322 13.093

4.827 5.293 5.757 6.218 6.677

47.916 56.849 66.426 76.623 87.415

11 12 13 14 15

16 17 18 19 20

1.320 1.343 1.367 1.390 1.415

0.7576 0.7446 0.7318 0.7192 0.7068

0.0547 0.0510 0.0477 0.0448 0.0422

0.0722 0.0685 0.0652 0.0623 0.0597

18.282 19.602 20.945 22.311 23.702

13.850 14.595 15.327 16.046 16.753

7.132 7.584 8.034 8.480 8.924

98.779 110.693 123.133 136.078 149.508

16 17 18 19 20

21 22 23 24 25

1.440 1.465 1.490 1.516 1.543

0.6947 0.6827 0.6710 0.6594 0.6481

0.0398 0.0377 0.0357 0.0339 0.0322

0.0573 0.0552 0.0532 0.0514 0.0497

25.116 26.556 28.021 29.511 31.027

17.448 18.130 18.801 19.461 20.109

9.365 9.803 10.239 10.671 11.101

163.401 177.738 192.500 207.667 223.221

21 22 23 24 25

26 27 28 29 30

1.570 1.597 1.625 1.654 1.683

0.6369 0.6260 0.6152 0.6046 0.5942

0.0307 0.0293 0.0280 0.0268 0.0256

0.0482 0.0468 0.0455 0.0443 0.0431

32.570 34.140 35.738 37.363 39.017

20.746 21.372 21.987 22.592 23.186

11.527 11.951 12.372 12.791 13.206

239.145 255.421 272.032 288.962 306.195

26 27 28 29 30

31 32 33 34 35

1.712 1.742 1.773 1.804 1.835

0.5840 0.5740 0.5641 0.5544 0.5449

0.0246 0.0236 0.0226 0.0218 0.0210

0.0421 0.0411 0.0401 0.0393 0.0385

40.700 42.412 44.154 45.927 47.731

23.770 24.344 24.908 25.462 26.007

13.619 14.029 14.436 14.840 15.241

323.716 341.510 359.561 377.857 396.382

31 32 33 34 35

36 37 38 39 40

1.867 1.900 1.933 1.967 2.002

0.5355 0.5263 0.5172 0.5083 0.4996

0.0202 0.0194 0.0187 0.0181 0.0175

0.0377 0.0369 0.0362 0.0356 0.0350

49.566 51.434 53.334 55.267 57.234

26.543 27.069 27.586 28.095 28.594

15.640 16.036 16.429 16.819 17.207

415.125 434.071 453.209 472.526 492.011

36 37 38 39 40

41 42 43 44 45

2.037 2.072 2.109 2.145 2.183

0.4910 0.4826 0.4743 0.4661 0.4581

0.0169 0.0163 0.0158 0.0153 0.0148

0.0344 0.0338 0.0333 0.0328 0.0323

59.236 61.272 63.345 65.453 67.599

29.085 29.568 30.042 30.508 30.966

17.591 17.973 18.353 18.729 19.103

511.651 531.436 551.355 571.398 591.554

41 42 43 44 45

46 47 48 49 50

2.221 2.260 2.300 2.340 2.381

0.4502 0.4425 0.4349 0.4274 0.4200

0.0143 0.0139 0.0135 0.0131 0.0127

0.0318 0.0314 0.0310 0.0306 0.0302

69.782 72.003 74.263 76.562 78.902

31.416 31.859 32.294 32.721 33.141

19.474 19.843 20.208 20.571 20.932

611.813 632.167 652.605 673.120 693.701

46 47 48 49 50

7477_book.fm Page 263 Tuesday, March 13, 2007 3:34 PM

Appendix E:

Interest factors and tables

2% Period

n 1 2 3 4 5

263

Compound Interest Factors Single Payment Uniform Payment Series Compound Present Sinking Capital Compound Amount Value Fund Recovery Amount Factor Factor Factor Factor Factor Find F Find P Find A Find A Find F Given P Given F Given F Given P Given A F/P P/F A/F A/P F/A 1.020 0.9804 1.0000 1.0200 1.000 1.040 0.9612 0.4950 0.5150 2.020 1.061 0.9423 0.3268 0.3468 3.060 1.082 0.9238 0.2426 0.2626 4.122 1.104 0.9057 0.1922 0.2122 5.204

2%

Arithmetic Gradient Period Present Gradient Gradient Value Uniform Present Factor Series Value Find P Find A Find P Given A Given G Given G P/A A/G P/G n 0.980 0.000 0.000 1 1.942 0.495 0.961 2 2.884 0.987 2.846 3 3.808 1.475 5.617 4 4.713 1.960 9.240 5

6 7 8 9 10

1.126 1.149 1.172 1.195 1.219

0.8880 0.8706 0.8535 0.8368 0.8203

0.1585 0.1345 0.1165 0.1025 0.0913

0.1785 0.1545 0.1365 0.1225 0.1113

6.308 7.434 8.583 9.755 10.950

5.601 6.472 7.325 8.162 8.983

2.442 2.921 3.396 3.868 4.337

13.680 18.903 24.878 31.572 38.955

6 7 8 9 10

11 12 13 14 15

1.243 1.268 1.294 1.319 1.346

0.8043 0.7885 0.7730 0.7579 0.7430

0.0822 0.0746 0.0681 0.0626 0.0578

0.1022 0.0946 0.0881 0.0826 0.0778

12.169 13.412 14.680 15.974 17.293

9.787 10.575 11.348 12.106 12.849

4.802 5.264 5.723 6.179 6.631

46.998 55.671 64.948 74.800 85.202

11 12 13 14 15

16 17 18 19 20

1.373 1.400 1.428 1.457 1.486

0.7284 0.7142 0.7002 0.6864 0.6730

0.0537 0.0500 0.0467 0.0438 0.0412

0.0737 0.0700 0.0667 0.0638 0.0612

18.639 20.012 21.412 22.841 24.297

13.578 14.292 14.992 15.678 16.351

7.080 7.526 7.968 8.407 8.843

96.129 107.555 119.458 131.814 144.600

16 17 18 19 20

21 22 23 24 25

1.516 1.546 1.577 1.608 1.641

0.6598 0.6468 0.6342 0.6217 0.6095

0.0388 0.0366 0.0347 0.0329 0.0312

0.0588 0.0566 0.0547 0.0529 0.0512

25.783 27.299 28.845 30.422 32.030

17.011 17.658 18.292 18.914 19.523

9.276 9.705 10.132 10.555 10.974

157.796 171.379 185.331 199.630 214.259

21 22 23 24 25

26 27 28 29 30

1.673 1.707 1.741 1.776 1.811

0.5976 0.5859 0.5744 0.5631 0.5521

0.0297 0.0283 0.0270 0.0258 0.0246

0.0497 0.0483 0.0470 0.0458 0.0446

33.671 35.344 37.051 38.792 40.568

20.121 20.707 21.281 21.844 22.396

11.391 11.804 12.214 12.621 13.025

229.199 244.431 259.939 275.706 291.716

26 27 28 29 30

31 32 33 34 35

1.848 1.885 1.922 1.961 2.000

0.5412 0.5306 0.5202 0.5100 0.5000

0.0236 0.0226 0.0217 0.0208 0.0200

0.0436 0.0426 0.0417 0.0408 0.0400

42.379 44.227 46.112 48.034 49.994

22.938 23.468 23.989 24.499 24.999

13.426 13.823 14.217 14.608 14.996

307.954 324.403 341.051 357.882 374.883

31 32 33 34 35

36 37 38 39 40

2.040 2.081 2.122 2.165 2.208

0.4902 0.4806 0.4712 0.4619 0.4529

0.0192 0.0185 0.0178 0.0172 0.0166

0.0392 0.0385 0.0378 0.0372 0.0366

51.994 54.034 56.115 58.237 60.402

25.489 25.969 26.441 26.903 27.355

15.381 15.762 16.141 16.516 16.889

392.040 409.342 426.776 444.330 461.993

36 37 38 39 40

41 42 43 44 45

2.252 2.297 2.343 2.390 2.438

0.4440 0.4353 0.4268 0.4184 0.4102

0.0160 0.0154 0.0149 0.0144 0.0139

0.0360 0.0354 0.0349 0.0344 0.0339

62.610 64.862 67.159 69.503 71.893

27.799 28.235 28.662 29.080 29.490

17.258 17.624 17.987 18.347 18.703

479.754 497.601 515.525 533.517 551.565

41 42 43 44 45

46 47 48 49 50

2.487 2.536 2.587 2.639 2.692

0.4022 0.3943 0.3865 0.3790 0.3715

0.0135 0.0130 0.0126 0.0122 0.0118

0.0335 0.0330 0.0326 0.0322 0.0318

74.331 76.817 79.354 81.941 84.579

29.892 30.287 30.673 31.052 31.424

19.057 19.408 19.756 20.100 20.442

569.662 587.798 605.966 624.156 642.361

46 47 48 49 50

7477_book.fm Page 264 Tuesday, March 13, 2007 3:34 PM

264

Computational Economic Analysis for Engineering and Industry

2.5% Period

n 1 2 3 4 5

Compound Interest Factors Single Payment Uniform Payment Series Compound Present Sinking Capital Compound Amount Value Fund Recovery Amount Factor Factor Factor Factor Factor Find F Find P Find A Find A Find F Given P Given F Given F Given P Given A F/P P/F A/F A/P F/A 1.025 0.9756 1.0000 1.0250 1.000 1.051 0.9518 0.4938 0.5188 2.025 1.077 0.9286 0.3251 0.3501 3.076 1.104 0.9060 0.2408 0.2658 4.153 1.131 0.8839 0.1902 0.2152 5.256

2.5%

Arithmetic Gradient Period Present Gradient Gradient Value Uniform Present Factor Series Value Find P Find A Find P Given A Given G Given G P/A A/G P/G n 0.976 0.000 0.000 1 1.927 0.494 0.952 2 2.856 0.984 2.809 3 3.762 1.469 5.527 4 4.646 1.951 9.062 5

6 7 8 9 10

1.160 1.189 1.218 1.249 1.280

0.8623 0.8413 0.8207 0.8007 0.7812

0.1565 0.1325 0.1145 0.1005 0.0893

0.1815 0.1575 0.1395 0.1255 0.1143

6.388 7.547 8.736 9.955 11.203

5.508 6.349 7.170 7.971 8.752

2.428 2.901 3.370 3.836 4.296

13.374 18.421 24.167 30.572 37.603

6 7 8 9 10

11 12 13 14 15

1.312 1.345 1.379 1.413 1.448

0.7621 0.7436 0.7254 0.7077 0.6905

0.0801 0.0725 0.0660 0.0605 0.0558

0.1051 0.0975 0.0910 0.0855 0.0808

12.483 13.796 15.140 16.519 17.932

9.514 10.258 10.983 11.691 12.381

4.753 5.206 5.655 6.100 6.540

45.225 53.404 62.109 71.309 80.976

11 12 13 14 15

16 17 18 19 20

1.485 1.522 1.560 1.599 1.639

0.6736 0.6572 0.6412 0.6255 0.6103

0.0516 0.0479 0.0447 0.0418 0.0391

0.0766 0.0729 0.0697 0.0668 0.0641

19.380 20.865 22.386 23.946 25.545

13.055 13.712 14.353 14.979 15.589

6.977 7.409 7.838 8.262 8.682

91.080 101.595 112.495 123.755 135.350

16 17 18 19 20

21 22 23 24 25

1.680 1.722 1.765 1.809 1.854

0.5954 0.5809 0.5667 0.5529 0.5394

0.0368 0.0346 0.0327 0.0309 0.0293

0.0618 0.0596 0.0577 0.0559 0.0543

27.183 28.863 30.584 32.349 34.158

16.185 16.765 17.332 17.885 18.424

9.099 9.511 9.919 10.324 10.724

147.257 159.456 171.923 184.639 197.584

21 22 23 24 25

26 27 28 29 30

1.900 1.948 1.996 2.046 2.098

0.5262 0.5134 0.5009 0.4887 0.4767

0.0278 0.0264 0.0251 0.0239 0.0228

0.0528 0.0514 0.0501 0.0489 0.0478

36.012 37.912 39.860 41.856 43.903

18.951 19.464 19.965 20.454 20.930

11.121 11.513 11.902 12.286 12.667

210.740 224.089 237.612 251.295 265.120

26 27 28 29 30

31 32 33 34 35

2.150 2.204 2.259 2.315 2.373

0.4651 0.4538 0.4427 0.4319 0.4214

0.0217 0.0208 0.0199 0.0190 0.0182

0.0467 0.0458 0.0449 0.0440 0.0432

46.000 48.150 50.354 52.613 54.928

21.395 21.849 22.292 22.724 23.145

13.044 13.417 13.786 14.151 14.512

279.074 293.141 307.307 321.560 335.887

31 32 33 34 35

36 37 38 39 40

2.433 2.493 2.556 2.620 2.685

0.4111 0.4011 0.3913 0.3817 0.3724

0.0175 0.0167 0.0161 0.0154 0.0148

0.0425 0.0417 0.0411 0.0404 0.0398

57.301 59.734 62.227 64.783 67.403

23.556 23.957 24.349 24.730 25.103

14.870 15.223 15.573 15.920 16.262

350.275 364.713 379.191 393.697 408.222

36 37 38 39 40

41 42 43 44 45

2.752 2.821 2.892 2.964 3.038

0.3633 0.3545 0.3458 0.3374 0.3292

0.0143 0.0137 0.0132 0.0127 0.0123

0.0393 0.0387 0.0382 0.0377 0.0373

70.088 72.840 75.661 78.552 81.516

25.466 25.821 26.166 26.504 26.833

16.601 16.936 17.267 17.595 17.918

422.756 437.290 451.815 466.323 480.807

41 42 43 44 45

46 47 48 49 50

3.114 3.192 3.271 3.353 3.437

0.3211 0.3133 0.3057 0.2982 0.2909

0.0118 0.0114 0.0110 0.0106 0.0103

0.0368 0.0364 0.0360 0.0356 0.0353

84.554 87.668 90.860 94.131 97.484

27.154 27.467 27.773 28.071 28.362

18.239 18.555 18.868 19.178 19.484

495.259 509.671 524.038 538.352 552.608

46 47 48 49 50

7477_book.fm Page 265 Tuesday, March 13, 2007 3:34 PM

Appendix E:

Interest factors and tables

3% Period

n 1 2 3 4 5

265

Compound Interest Factors Single Payment Uniform Payment Series Compound Present Sinking Capital Compound Amount Value Fund Recovery Amount Factor Factor Factor Factor Factor Find F Find P Find A Find A Find F Given P Given F Given F Given P Given A F/P P/F A/F A/P F/A 1.030 0.9709 1.0000 1.0300 1.000 1.061 0.9426 0.4926 0.5226 2.030 1.093 0.9151 0.3235 0.3535 3.091 1.126 0.8885 0.2390 0.2690 4.184 1.159 0.8626 0.1884 0.2184 5.309

3%

Arithmetic Gradient Period Present Gradient Gradient Value Uniform Present Factor Series Value Find P Find A Find P Given A Given G Given G P/A A/G P/G n 0.971 0.000 0.000 1 1.913 0.493 0.943 2 2.829 0.980 2.773 3 3.717 1.463 5.438 4 4.580 1.941 8.889 5

6 7 8 9 10

1.194 1.230 1.267 1.305 1.344

0.8375 0.8131 0.7894 0.7664 0.7441

0.1546 0.1305 0.1125 0.0984 0.0872

0.1846 0.1605 0.1425 0.1284 0.1172

6.468 7.662 8.892 10.159 11.464

5.417 6.230 7.020 7.786 8.530

2.414 2.882 3.345 3.803 4.256

13.076 17.955 23.481 29.612 36.309

6 7 8 9 10

11 12 13 14 15

1.384 1.426 1.469 1.513 1.558

0.7224 0.7014 0.6810 0.6611 0.6419

0.0781 0.0705 0.0640 0.0585 0.0538

0.1081 0.1005 0.0940 0.0885 0.0838

12.808 14.192 15.618 17.086 18.599

9.253 9.954 10.635 11.296 11.938

4.705 5.148 5.587 6.021 6.450

43.533 51.248 59.420 68.014 77.000

11 12 13 14 15

16 17 18 19 20

1.605 1.653 1.702 1.754 1.806

0.6232 0.6050 0.5874 0.5703 0.5537

0.0496 0.0460 0.0427 0.0398 0.0372

0.0796 0.0760 0.0727 0.0698 0.0672

20.157 21.762 23.414 25.117 26.870

12.561 13.166 13.754 14.324 14.877

6.874 7.294 7.708 8.118 8.523

86.348 96.028 106.014 116.279 126.799

16 17 18 19 20

21 22 23 24 25

1.860 1.916 1.974 2.033 2.094

0.5375 0.5219 0.5067 0.4919 0.4776

0.0349 0.0327 0.0308 0.0290 0.0274

0.0649 0.0627 0.0608 0.0590 0.0574

28.676 30.537 32.453 34.426 36.459

15.415 15.937 16.444 16.936 17.413

8.923 9.319 9.709 10.095 10.477

137.550 148.509 159.657 170.971 182.434

21 22 23 24 25

26 27 28 29 30

2.157 2.221 2.288 2.357 2.427

0.4637 0.4502 0.4371 0.4243 0.4120

0.0259 0.0246 0.0233 0.0221 0.0210

0.0559 0.0546 0.0533 0.0521 0.0510

38.553 40.710 42.931 45.219 47.575

17.877 18.327 18.764 19.188 19.600

10.853 11.226 11.593 11.956 12.314

194.026 205.731 217.532 229.414 241.361

26 27 28 29 30

31 32 33 34 35

2.500 2.575 2.652 2.732 2.814

0.4000 0.3883 0.3770 0.3660 0.3554

0.0200 0.0190 0.0182 0.0173 0.0165

0.0500 0.0490 0.0482 0.0473 0.0465

50.003 52.503 55.078 57.730 60.462

20.000 20.389 20.766 21.132 21.487

12.668 13.017 13.362 13.702 14.037

253.361 265.399 277.464 289.544 301.627

31 32 33 34 35

36 37 38 39 40

2.898 2.985 3.075 3.167 3.262

0.3450 0.3350 0.3252 0.3158 0.3066

0.0158 0.0151 0.0145 0.0138 0.0133

0.0458 0.0451 0.0445 0.0438 0.0433

63.276 66.174 69.159 72.234 75.401

21.832 22.167 22.492 22.808 23.115

14.369 14.696 15.018 15.336 15.650

313.703 325.762 337.796 349.794 361.750

36 37 38 39 40

41 42 43 44 45

3.360 3.461 3.565 3.671 3.782

0.2976 0.2890 0.2805 0.2724 0.2644

0.0127 0.0122 0.0117 0.0112 0.0108

0.0427 0.0422 0.0417 0.0412 0.0408

78.663 82.023 85.484 89.048 92.720

23.412 23.701 23.982 24.254 24.519

15.960 16.265 16.566 16.863 17.156

373.655 385.502 397.285 408.997 420.632

41 42 43 44 45

46 47 48 49 50

3.895 4.012 4.132 4.256 4.384

0.2567 0.2493 0.2420 0.2350 0.2281

0.0104 0.0100 0.0096 0.0092 0.0089

0.0404 0.0400 0.0396 0.0392 0.0389

96.501 100.397 104.408 108.541 112.797

24.775 25.025 25.267 25.502 25.730

17.444 17.729 18.009 18.285 18.558

432.186 443.652 455.025 466.303 477.480

46 47 48 49 50

7477_book.fm Page 266 Tuesday, March 13, 2007 3:34 PM

266

Computational Economic Analysis for Engineering and Industry

3.5% Period

n 1 2 3 4 5

Compound Interest Factors Single Payment Uniform Payment Series Compound Present Sinking Capital Compound Amount Value Fund Recovery Amount Factor Factor Factor Factor Factor Find F Find P Find A Find A Find F Given P Given F Given F Given P Given A F/P P/F A/F A/P F/A 1.035 0.9662 1.0000 1.0350 1.000 1.071 0.9335 0.4914 0.5264 2.035 1.109 0.9019 0.3219 0.3569 3.106 1.148 0.8714 0.2373 0.2723 4.215 1.188 0.8420 0.1865 0.2215 5.362

3.5%

Arithmetic Gradient Period Present Gradient Gradient Value Uniform Present Factor Series Value Find P Find A Find P Given A Given G Given G P/A A/G P/G n 0.966 0.000 0.000 1 1.900 0.491 0.934 2 2.802 0.977 2.737 3 3.673 1.457 5.352 4 4.515 1.931 8.720 5

6 7 8 9 10

1.229 1.272 1.317 1.363 1.411

0.8135 0.7860 0.7594 0.7337 0.7089

0.1527 0.1285 0.1105 0.0964 0.0852

0.1877 0.1635 0.1455 0.1314 0.1202

6.550 7.779 9.052 10.368 11.731

5.329 6.115 6.874 7.608 8.317

2.400 2.863 3.320 3.771 4.217

12.787 17.503 22.819 28.689 35.069

6 7 8 9 10

11 12 13 14 15

1.460 1.511 1.564 1.619 1.675

0.6849 0.6618 0.6394 0.6178 0.5969

0.0761 0.0685 0.0621 0.0566 0.0518

0.1111 0.1035 0.0971 0.0916 0.0868

13.142 14.602 16.113 17.677 19.296

9.002 9.663 10.303 10.921 11.517

4.657 5.091 5.520 5.943 6.361

41.919 49.198 56.871 64.902 73.259

11 12 13 14 15

16 17 18 19 20

1.734 1.795 1.857 1.923 1.990

0.5767 0.5572 0.5384 0.5202 0.5026

0.0477 0.0440 0.0408 0.0379 0.0354

0.0827 0.0790 0.0758 0.0729 0.0704

20.971 22.705 24.500 26.357 28.280

12.094 12.651 13.190 13.710 14.212

6.773 7.179 7.580 7.975 8.365

81.909 90.824 99.977 109.339 118.888

16 17 18 19 20

21 22 23 24 25

2.059 2.132 2.206 2.283 2.363

0.4856 0.4692 0.4533 0.4380 0.4231

0.0330 0.0309 0.0290 0.0273 0.0257

0.0680 0.0659 0.0640 0.0623 0.0607

30.269 32.329 34.460 36.667 38.950

14.698 15.167 15.620 16.058 16.482

8.749 9.128 9.502 9.870 10.233

128.600 138.452 148.424 158.497 168.653

21 22 23 24 25

26 27 28 29 30

2.446 2.532 2.620 2.712 2.807

0.4088 0.3950 0.3817 0.3687 0.3563

0.0242 0.0229 0.0216 0.0204 0.0194

0.0592 0.0579 0.0566 0.0554 0.0544

41.313 43.759 46.291 48.911 51.623

16.890 17.285 17.667 18.036 18.392

10.590 10.942 11.289 11.631 11.967

178.874 189.144 199.448 209.773 220.106

26 27 28 29 30

31 32 33 34 35

2.905 3.007 3.112 3.221 3.334

0.3442 0.3326 0.3213 0.3105 0.3000

0.0184 0.0174 0.0166 0.0158 0.0150

0.0534 0.0524 0.0516 0.0508 0.0500

54.429 57.335 60.341 63.453 66.674

18.736 19.069 19.390 19.701 20.001

12.299 12.625 12.946 13.262 13.573

230.432 240.743 251.026 261.271 271.471

31 32 33 34 35

36 37 38 39 40

3.450 3.571 3.696 3.825 3.959

0.2898 0.2800 0.2706 0.2614 0.2526

0.0143 0.0136 0.0130 0.0124 0.0118

0.0493 0.0486 0.0480 0.0474 0.0468

70.008 73.458 77.029 80.725 84.550

20.290 20.571 20.841 21.102 21.355

13.879 14.180 14.477 14.768 15.055

281.615 291.696 301.707 311.640 321.491

36 37 38 39 40

41 42 43 44 45

4.098 4.241 4.390 4.543 4.702

0.2440 0.2358 0.2278 0.2201 0.2127

0.0113 0.0108 0.0103 0.0099 0.0095

0.0463 0.0458 0.0453 0.0449 0.0445

88.510 92.607 96.849 101.238 105.782

21.599 21.835 22.063 22.283 22.495

15.336 15.613 15.886 16.154 16.417

331.252 340.919 350.487 359.951 369.308

41 42 43 44 45

46 47 48 49 50

4.867 5.037 5.214 5.396 5.585

0.2055 0.1985 0.1918 0.1853 0.1791

0.0091 0.0087 0.0083 0.0080 0.0076

0.0441 0.0437 0.0433 0.0430 0.0426

110.484 115.351 120.388 125.602 130.998

22.701 22.899 23.091 23.277 23.456

16.676 16.930 17.180 17.425 17.666

378.554 387.686 396.701 405.596 414.370

46 47 48 49 50

7477_book.fm Page 267 Tuesday, March 13, 2007 3:34 PM

Appendix E:

Interest factors and tables

4% Period

n 1 2 3 4 5

267

Compound Interest Factors Single Payment Uniform Payment Series Compound Present Sinking Capital Compound Amount Value Fund Recovery Amount Factor Factor Factor Factor Factor Find F Find P Find A Find A Find F Given P Given F Given F Given P Given A F/P P/F A/F A/P F/A 1.040 0.9615 1.0000 1.0400 1.000 1.082 0.9246 0.4902 0.5302 2.040 1.125 0.8890 0.3203 0.3603 3.122 1.170 0.8548 0.2355 0.2755 4.246 1.217 0.8219 0.1846 0.2246 5.416

4%

Arithmetic Gradient Period Present Gradient Gradient Value Uniform Present Factor Series Value Find P Find A Find P Given A Given G Given G P/A A/G P/G n 0.962 0.000 0.000 1 1.886 0.490 0.925 2 2.775 0.974 2.703 3 3.630 1.451 5.267 4 4.452 1.922 8.555 5

6 7 8 9 10

1.265 1.316 1.369 1.423 1.480

0.7903 0.7599 0.7307 0.7026 0.6756

0.1508 0.1266 0.1085 0.0945 0.0833

0.1908 0.1666 0.1485 0.1345 0.1233

6.633 7.898 9.214 10.583 12.006

5.242 6.002 6.733 7.435 8.111

2.386 2.843 3.294 3.739 4.177

12.506 17.066 22.181 27.801 33.881

6 7 8 9 10

11 12 13 14 15

1.539 1.601 1.665 1.732 1.801

0.6496 0.6246 0.6006 0.5775 0.5553

0.0741 0.0666 0.0601 0.0547 0.0499

0.1141 0.1066 0.1001 0.0947 0.0899

13.486 15.026 16.627 18.292 20.024

8.760 9.385 9.986 10.563 11.118

4.609 5.034 5.453 5.866 6.272

40.377 47.248 54.455 61.962 69.735

11 12 13 14 15

16 17 18 19 20

1.873 1.948 2.026 2.107 2.191

0.5339 0.5134 0.4936 0.4746 0.4564

0.0458 0.0422 0.0390 0.0361 0.0336

0.0858 0.0822 0.0790 0.0761 0.0736

21.825 23.698 25.645 27.671 29.778

11.652 12.166 12.659 13.134 13.590

6.672 7.066 7.453 7.834 8.209

77.744 85.958 94.350 102.893 111.565

16 17 18 19 20

21 22 23 24 25

2.279 2.370 2.465 2.563 2.666

0.4388 0.4220 0.4057 0.3901 0.3751

0.0313 0.0292 0.0273 0.0256 0.0240

0.0713 0.0692 0.0673 0.0656 0.0640

31.969 34.248 36.618 39.083 41.646

14.029 14.451 14.857 15.247 15.622

8.578 8.941 9.297 9.648 9.993

120.341 129.202 138.128 147.101 156.104

21 22 23 24 25

26 27 28 29 30

2.772 2.883 2.999 3.119 3.243

0.3607 0.3468 0.3335 0.3207 0.3083

0.0226 0.0212 0.0200 0.0189 0.0178

0.0626 0.0612 0.0600 0.0589 0.0578

44.312 47.084 49.968 52.966 56.085

15.983 16.330 16.663 16.984 17.292

10.331 10.664 10.991 11.312 11.627

165.121 174.138 183.142 192.121 201.062

26 27 28 29 30

31 32 33 34 35

3.373 3.508 3.648 3.794 3.946

0.2965 0.2851 0.2741 0.2636 0.2534

0.0169 0.0159 0.0151 0.0143 0.0136

0.0569 0.0559 0.0551 0.0543 0.0536

59.328 62.701 66.210 69.858 73.652

17.588 17.874 18.148 18.411 18.665

11.937 12.241 12.540 12.832 13.120

209.956 218.792 227.563 236.261 244.877

31 32 33 34 35

36 37 38 39 40

4.104 4.268 4.439 4.616 4.801

0.2437 0.2343 0.2253 0.2166 0.2083

0.0129 0.0122 0.0116 0.0111 0.0105

0.0529 0.0522 0.0516 0.0511 0.0505

77.598 81.702 85.970 90.409 95.026

18.908 19.143 19.368 19.584 19.793

13.402 13.678 13.950 14.216 14.477

253.405 261.840 270.175 278.407 286.530

36 37 38 39 40

41 42 43 44 45

4.993 5.193 5.400 5.617 5.841

0.2003 0.1926 0.1852 0.1780 0.1712

0.0100 0.0095 0.0091 0.0087 0.0083

0.0500 0.0495 0.0491 0.0487 0.0483

99.827 104.820 110.012 115.413 121.029

19.993 20.186 20.371 20.549 20.720

14.732 14.983 15.228 15.469 15.705

294.541 302.437 310.214 317.870 325.403

41 42 43 44 45

46 47 48 49 50

6.075 6.318 6.571 6.833 7.107

0.1646 0.1583 0.1522 0.1463 0.1407

0.0079 0.0075 0.0072 0.0069 0.0066

0.0479 0.0475 0.0472 0.0469 0.0466

126.871 132.945 139.263 145.834 152.667

20.885 21.043 21.195 21.341 21.482

15.936 16.162 16.383 16.600 16.812

332.810 340.091 347.245 354.269 361.164

46 47 48 49 50

7477_book.fm Page 268 Tuesday, March 13, 2007 3:34 PM

268

Computational Economic Analysis for Engineering and Industry

4.5% Period

n 1 2 3 4 5

Compound Interest Factors Single Payment Uniform Payment Series Compound Present Sinking Capital Compound Amount Value Fund Recovery Amount Factor Factor Factor Factor Factor Find F Find P Find A Find A Find F Given P Given F Given F Given P Given A F/P P/F A/F A/P F/A 1.045 0.9569 1.0000 1.0450 1.000 1.092 0.9157 0.4890 0.5340 2.045 1.141 0.8763 0.3188 0.3638 3.137 1.193 0.8386 0.2337 0.2787 4.278 1.246 0.8025 0.1828 0.2278 5.471

4.5%

Arithmetic Gradient Period Present Gradient Gradient Value Uniform Present Factor Series Value Find P Find A Find P Given A Given G Given G P/A A/G P/G n 0.957 0.000 0.000 1 1.873 0.489 0.916 2 2.749 0.971 2.668 3 3.588 1.445 5.184 4 4.390 1.912 8.394 5

6 7 8 9 10

1.302 1.361 1.422 1.486 1.553

0.7679 0.7348 0.7032 0.6729 0.6439

0.1489 0.1247 0.1066 0.0926 0.0814

0.1939 0.1697 0.1516 0.1376 0.1264

6.717 8.019 9.380 10.802 12.288

5.158 5.893 6.596 7.269 7.913

2.372 2.824 3.269 3.707 4.138

12.233 16.642 21.565 26.948 32.743

6 7 8 9 10

11 12 13 14 15

1.623 1.696 1.772 1.852 1.935

0.6162 0.5897 0.5643 0.5400 0.5167

0.0722 0.0647 0.0583 0.0528 0.0481

0.1172 0.1097 0.1033 0.0978 0.0931

13.841 15.464 17.160 18.932 20.784

8.529 9.119 9.683 10.223 10.740

4.562 4.978 5.387 5.789 6.184

38.905 45.391 52.163 59.182 66.416

11 12 13 14 15

16 17 18 19 20

2.022 2.113 2.208 2.308 2.412

0.4945 0.4732 0.4528 0.4333 0.4146

0.0440 0.0404 0.0372 0.0344 0.0319

0.0890 0.0854 0.0822 0.0794 0.0769

22.719 24.742 26.855 29.064 31.371

11.234 11.707 12.160 12.593 13.008

6.572 6.953 7.327 7.695 8.055

73.833 81.404 89.102 96.901 104.780

16 17 18 19 20

21 22 23 24 25

2.520 2.634 2.752 2.876 3.005

0.3968 0.3797 0.3634 0.3477 0.3327

0.0296 0.0275 0.0257 0.0240 0.0224

0.0746 0.0725 0.0707 0.0690 0.0674

33.783 36.303 38.937 41.689 44.565

13.405 13.784 14.148 14.495 14.828

8.409 8.755 9.096 9.429 9.756

112.715 120.689 128.683 136.680 144.665

21 22 23 24 25

26 27 28 29 30

3.141 3.282 3.430 3.584 3.745

0.3184 0.3047 0.2916 0.2790 0.2670

0.0210 0.0197 0.0185 0.0174 0.0164

0.0660 0.0647 0.0635 0.0624 0.0614

47.571 50.711 53.993 57.423 61.007

15.147 15.451 15.743 16.022 16.289

10.077 10.391 10.698 10.999 11.295

152.625 160.547 168.420 176.232 183.975

26 27 28 29 30

31 32 33 34 35

3.914 4.090 4.274 4.466 4.667

0.2555 0.2445 0.2340 0.2239 0.2143

0.0154 0.0146 0.0137 0.0130 0.0123

0.0604 0.0596 0.0587 0.0580 0.0573

64.752 68.666 72.756 77.030 81.497

16.544 16.789 17.023 17.247 17.461

11.583 11.866 12.143 12.414 12.679

191.640 199.220 206.707 214.096 221.380

31 32 33 34 35

36 37 38 39 40

4.877 5.097 5.326 5.566 5.816

0.2050 0.1962 0.1878 0.1797 0.1719

0.0116 0.0110 0.0104 0.0099 0.0093

0.0566 0.0560 0.0554 0.0549 0.0543

86.164 91.041 96.138 101.464 107.030

17.666 17.862 18.050 18.230 18.402

12.938 13.191 13.439 13.681 13.917

228.556 235.619 242.566 249.393 256.099

36 37 38 39 40

41 42 43 44 45

6.078 6.352 6.637 6.936 7.248

0.1645 0.1574 0.1507 0.1442 0.1380

0.0089 0.0084 0.0080 0.0076 0.0072

0.0539 0.0534 0.0530 0.0526 0.0522

112.847 118.925 125.276 131.914 138.850

18.566 18.724 18.874 19.018 19.156

14.148 14.374 14.595 14.810 15.020

262.680 269.135 275.462 281.662 287.732

41 42 43 44 45

46 47 48 49 50

7.574 7.915 8.271 8.644 9.033

0.1320 0.1263 0.1209 0.1157 0.1107

0.0068 0.0065 0.0062 0.0059 0.0056

0.0518 0.0515 0.0512 0.0509 0.0506

146.098 153.673 161.588 169.859 178.503

19.288 19.415 19.536 19.651 19.762

15.225 15.426 15.621 15.812 15.998

293.673 299.485 305.167 310.720 316.145

46 47 48 49 50

7477_book.fm Page 269 Tuesday, March 13, 2007 3:34 PM

Appendix E:

Interest factors and tables

5% Period

n 1 2 3 4 5

269

Compound Interest Factors Single Payment Uniform Payment Series Compound Present Sinking Capital Compound Amount Value Fund Recovery Amount Factor Factor Factor Factor Factor Find F Find P Find A Find A Find F Given P Given F Given F Given P Given A F/P P/F A/F A/P F/A 1.050 0.9524 1.0000 1.0500 1.000 1.103 0.9070 0.4878 0.5378 2.050 1.158 0.8638 0.3172 0.3672 3.153 1.216 0.8227 0.2320 0.2820 4.310 1.276 0.7835 0.1810 0.2310 5.526

5%

Arithmetic Gradient Period Present Gradient Gradient Value Uniform Present Factor Series Value Find P Find A Find P Given A Given G Given G P/A A/G P/G n 0.952 0.000 0.000 1 1.859 0.488 0.907 2 2.723 0.967 2.635 3 3.546 1.439 5.103 4 4.329 1.903 8.237 5

6 7 8 9 10

1.340 1.407 1.477 1.551 1.629

0.7462 0.7107 0.6768 0.6446 0.6139

0.1470 0.1228 0.1047 0.0907 0.0795

0.1970 0.1728 0.1547 0.1407 0.1295

6.802 8.142 9.549 11.027 12.578

5.076 5.786 6.463 7.108 7.722

2.358 2.805 3.245 3.676 4.099

11.968 16.232 20.970 26.127 31.652

6 7 8 9 10

11 12 13 14 15

1.710 1.796 1.886 1.980 2.079

0.5847 0.5568 0.5303 0.5051 0.4810

0.0704 0.0628 0.0565 0.0510 0.0463

0.1204 0.1128 0.1065 0.1010 0.0963

14.207 15.917 17.713 19.599 21.579

8.306 8.863 9.394 9.899 10.380

4.514 4.922 5.322 5.713 6.097

37.499 43.624 49.988 56.554 63.288

11 12 13 14 15

16 17 18 19 20

2.183 2.292 2.407 2.527 2.653

0.4581 0.4363 0.4155 0.3957 0.3769

0.0423 0.0387 0.0355 0.0327 0.0302

0.0923 0.0887 0.0855 0.0827 0.0802

23.657 25.840 28.132 30.539 33.066

10.838 11.274 11.690 12.085 12.462

6.474 6.842 7.203 7.557 7.903

70.160 77.140 84.204 91.328 98.488

16 17 18 19 20

21 22 23 24 25

2.786 2.925 3.072 3.225 3.386

0.3589 0.3418 0.3256 0.3101 0.2953

0.0280 0.0260 0.0241 0.0225 0.0210

0.0780 0.0760 0.0741 0.0725 0.0710

35.719 38.505 41.430 44.502 47.727

12.821 13.163 13.489 13.799 14.094

8.242 8.573 8.897 9.214 9.524

105.667 112.846 120.009 127.140 134.228

21 22 23 24 25

26 27 28 29 30

3.556 3.733 3.920 4.116 4.322

0.2812 0.2678 0.2551 0.2429 0.2314

0.0196 0.0183 0.0171 0.0160 0.0151

0.0696 0.0683 0.0671 0.0660 0.0651

51.113 54.669 58.403 62.323 66.439

14.375 14.643 14.898 15.141 15.372

9.827 10.122 10.411 10.694 10.969

141.259 148.223 155.110 161.913 168.623

26 27 28 29 30

31 32 33 34 35

4.538 4.765 5.003 5.253 5.516

0.2204 0.2099 0.1999 0.1904 0.1813

0.0141 0.0133 0.0125 0.0118 0.0111

0.0641 0.0633 0.0625 0.0618 0.0611

70.761 75.299 80.064 85.067 90.320

15.593 15.803 16.003 16.193 16.374

11.238 11.501 11.757 12.006 12.250

175.233 181.739 188.135 194.417 200.581

31 32 33 34 35

36 37 38 39 40

5.792 6.081 6.385 6.705 7.040

0.1727 0.1644 0.1566 0.1491 0.1420

0.0104 0.0098 0.0093 0.0088 0.0083

0.0604 0.0598 0.0593 0.0588 0.0583

95.836 101.628 107.710 114.095 120.800

16.547 16.711 16.868 17.017 17.159

12.487 12.719 12.944 13.164 13.377

206.624 212.543 218.338 224.005 229.545

36 37 38 39 40

41 42 43 44 45

7.392 7.762 8.150 8.557 8.985

0.1353 0.1288 0.1227 0.1169 0.1113

0.0078 0.0074 0.0070 0.0066 0.0063

0.0578 0.0574 0.0570 0.0566 0.0563

127.840 135.232 142.993 151.143 159.700

17.294 17.423 17.546 17.663 17.774

13.586 13.788 13.986 14.178 14.364

234.956 240.239 245.392 250.417 255.315

41 42 43 44 45

46 47 48 49 50

9.434 9.906 10.401 10.921 11.467

0.1060 0.1009 0.0961 0.0916 0.0872

0.0059 0.0056 0.0053 0.0050 0.0048

0.0559 0.0556 0.0553 0.0550 0.0548

168.685 178.119 188.025 198.427 209.348

17.880 17.981 18.077 18.169 18.256

14.546 14.723 14.894 15.061 15.223

260.084 264.728 269.247 273.642 277.915

46 47 48 49 50

7477_book.fm Page 270 Tuesday, March 13, 2007 3:34 PM

270

Computational Economic Analysis for Engineering and Industry

6% Period

n 1 2 3 4 5

Compound Interest Factors Single Payment Uniform Payment Series Compound Present Sinking Capital Compound Amount Value Fund Recovery Amount Factor Factor Factor Factor Factor Find F Find P Find A Find A Find F Given P Given F Given F Given P Given A F/P P/F A/F A/P F/A 1.060 0.9434 1.0000 1.0600 1.000 1.124 0.8900 0.4854 0.5454 2.060 1.191 0.8396 0.3141 0.3741 3.184 1.262 0.7921 0.2286 0.2886 4.375 1.338 0.7473 0.1774 0.2374 5.637

6%

Arithmetic Gradient Period Present Gradient Gradient Value Uniform Present Factor Series Value Find P Find A Find P Given A Given G Given G P/A A/G P/G n 0.943 0.000 0.000 1 1.833 0.485 0.890 2 2.673 0.961 2.569 3 3.465 1.427 4.946 4 4.212 1.884 7.935 5

6 7 8 9 10

1.419 1.504 1.594 1.689 1.791

0.7050 0.6651 0.6274 0.5919 0.5584

0.1434 0.1191 0.1010 0.0870 0.0759

0.2034 0.1791 0.1610 0.1470 0.1359

6.975 8.394 9.897 11.491 13.181

4.917 5.582 6.210 6.802 7.360

2.330 2.768 3.195 3.613 4.022

11.459 15.450 19.842 24.577 29.602

6 7 8 9 10

11 12 13 14 15

1.898 2.012 2.133 2.261 2.397

0.5268 0.4970 0.4688 0.4423 0.4173

0.0668 0.0593 0.0530 0.0476 0.0430

0.1268 0.1193 0.1130 0.1076 0.1030

14.972 16.870 18.882 21.015 23.276

7.887 8.384 8.853 9.295 9.712

4.421 4.811 5.192 5.564 5.926

34.870 40.337 45.963 51.713 57.555

11 12 13 14 15

16 17 18 19 20

2.540 2.693 2.854 3.026 3.207

0.3936 0.3714 0.3503 0.3305 0.3118

0.0390 0.0354 0.0324 0.0296 0.0272

0.0990 0.0954 0.0924 0.0896 0.0872

25.673 28.213 30.906 33.760 36.786

10.106 10.477 10.828 11.158 11.470

6.279 6.624 6.960 7.287 7.605

63.459 69.401 75.357 81.306 87.230

16 17 18 19 20

21 22 23 24 25

3.400 3.604 3.820 4.049 4.292

0.2942 0.2775 0.2618 0.2470 0.2330

0.0250 0.0230 0.0213 0.0197 0.0182

0.0850 0.0830 0.0813 0.0797 0.0782

39.993 43.392 46.996 50.816 54.865

11.764 12.042 12.303 12.550 12.783

7.915 8.217 8.510 8.795 9.072

93.114 98.941 104.701 110.381 115.973

21 22 23 24 25

26 27 28 29 30

4.549 4.822 5.112 5.418 5.743

0.2198 0.2074 0.1956 0.1846 0.1741

0.0169 0.0157 0.0146 0.0136 0.0126

0.0769 0.0757 0.0746 0.0736 0.0726

59.156 63.706 68.528 73.640 79.058

13.003 13.211 13.406 13.591 13.765

9.341 9.603 9.857 10.103 10.342

121.468 126.860 132.142 137.310 142.359

26 27 28 29 30

31 32 33 34 35

6.088 6.453 6.841 7.251 7.686

0.1643 0.1550 0.1462 0.1379 0.1301

0.0118 0.0110 0.0103 0.0096 0.0090

0.0718 0.0710 0.0703 0.0696 0.0690

84.802 90.890 97.343 104.184 111.435

13.929 14.084 14.230 14.368 14.498

10.574 10.799 11.017 11.228 11.432

147.286 152.090 156.768 161.319 165.743

31 32 33 34 35

36 37 38 39 40

8.147 8.636 9.154 9.704 10.286

0.1227 0.1158 0.1092 0.1031 0.0972

0.0084 0.0079 0.0074 0.0069 0.0065

0.0684 0.0679 0.0674 0.0669 0.0665

119.121 127.268 135.904 145.058 154.762

14.621 14.737 14.846 14.949 15.046

11.630 11.821 12.007 12.186 12.359

170.039 174.207 178.249 182.165 185.957

36 37 38 39 40

41 42 43 44 45

10.903 11.557 12.250 12.985 13.765

0.0917 0.0865 0.0816 0.0770 0.0727

0.0061 0.0057 0.0053 0.0050 0.0047

0.0661 0.0657 0.0653 0.0650 0.0647

165.048 175.951 187.508 199.758 212.744

15.138 15.225 15.306 15.383 15.456

12.526 12.688 12.845 12.996 13.141

189.626 193.173 196.602 199.913 203.110

41 42 43 44 45

46 47 48 49 50

14.590 15.466 16.394 17.378 18.420

0.0685 0.0647 0.0610 0.0575 0.0543

0.0044 0.0041 0.0039 0.0037 0.0034

0.0644 0.0641 0.0639 0.0637 0.0634

226.508 241.099 256.565 272.958 290.336

15.524 15.589 15.650 15.708 15.762

13.282 13.418 13.549 13.675 13.796

206.194 209.168 212.035 214.797 217.457

46 47 48 49 50

7477_book.fm Page 271 Tuesday, March 13, 2007 3:34 PM

Appendix E:

Interest factors and tables

7% Period

n 1 2 3 4 5

271

Compound Interest Factors Single Payment Uniform Payment Series Compound Present Sinking Capital Compound Amount Value Fund Recovery Amount Factor Factor Factor Factor Factor Find F Find P Find A Find A Find F Given P Given F Given F Given P Given A F/P P/F A/F A/P F/A 1.070 0.9346 1.0000 1.0700 1.000 1.145 0.8734 0.4831 0.5531 2.070 1.225 0.8163 0.3111 0.3811 3.215 1.311 0.7629 0.2252 0.2952 4.440 1.403 0.7130 0.1739 0.2439 5.751

7%

Arithmetic Gradient Period Present Gradient Gradient Value Uniform Present Factor Series Value Find P Find A Find P Given A Given G Given G P/A A/G P/G n 0.935 0.000 0.000 1 1.808 0.483 0.873 2 2.624 0.955 2.506 3 3.387 1.416 4.795 4 4.100 1.865 7.647 5

6 7 8 9 10

1.501 1.606 1.718 1.838 1.967

0.6663 0.6227 0.5820 0.5439 0.5083

0.1398 0.1156 0.0975 0.0835 0.0724

0.2098 0.1856 0.1675 0.1535 0.1424

7.153 8.654 10.260 11.978 13.816

4.767 5.389 5.971 6.515 7.024

2.303 2.730 3.147 3.552 3.946

10.978 14.715 18.789 23.140 27.716

6 7 8 9 10

11 12 13 14 15

2.105 2.252 2.410 2.579 2.759

0.4751 0.4440 0.4150 0.3878 0.3624

0.0634 0.0559 0.0497 0.0443 0.0398

0.1334 0.1259 0.1197 0.1143 0.1098

15.784 17.888 20.141 22.550 25.129

7.499 7.943 8.358 8.745 9.108

4.330 4.703 5.065 5.417 5.758

32.466 37.351 42.330 47.372 52.446

11 12 13 14 15

16 17 18 19 20

2.952 3.159 3.380 3.617 3.870

0.3387 0.3166 0.2959 0.2765 0.2584

0.0359 0.0324 0.0294 0.0268 0.0244

0.1059 0.1024 0.0994 0.0968 0.0944

27.888 30.840 33.999 37.379 40.995

9.447 9.763 10.059 10.336 10.594

6.090 6.411 6.722 7.024 7.316

57.527 62.592 67.622 72.599 77.509

16 17 18 19 20

21 22 23 24 25

4.141 4.430 4.741 5.072 5.427

0.2415 0.2257 0.2109 0.1971 0.1842

0.0223 0.0204 0.0187 0.0172 0.0158

0.0923 0.0904 0.0887 0.0872 0.0858

44.865 49.006 53.436 58.177 63.249

10.836 11.061 11.272 11.469 11.654

7.599 7.872 8.137 8.392 8.639

82.339 87.079 91.720 96.255 100.676

21 22 23 24 25

26 27 28 29 30

5.807 6.214 6.649 7.114 7.612

0.1722 0.1609 0.1504 0.1406 0.1314

0.0146 0.0134 0.0124 0.0114 0.0106

0.0846 0.0834 0.0824 0.0814 0.0806

68.676 74.484 80.698 87.347 94.461

11.826 11.987 12.137 12.278 12.409

8.877 9.107 9.329 9.543 9.749

104.981 109.166 113.226 117.162 120.972

26 27 28 29 30

31 32 33 34 35

8.145 8.715 9.325 9.978 10.677

0.1228 0.1147 0.1072 0.1002 0.0937

0.0098 0.0091 0.0084 0.0078 0.0072

0.0798 0.0791 0.0784 0.0778 0.0772

102.073 110.218 118.933 128.259 138.237

12.532 12.647 12.754 12.854 12.948

9.947 10.138 10.322 10.499 10.669

124.655 128.212 131.643 134.951 138.135

31 32 33 34 35

36 37 38 39 40

11.424 12.224 13.079 13.995 14.974

0.0875 0.0818 0.0765 0.0715 0.0668

0.0067 0.0062 0.0058 0.0054 0.0050

0.0767 0.0762 0.0758 0.0754 0.0750

148.913 160.337 172.561 185.640 199.635

13.035 13.117 13.193 13.265 13.332

10.832 10.989 11.140 11.285 11.423

141.199 144.144 146.973 149.688 152.293

36 37 38 39 40

41 42 43 44 45

16.023 17.144 18.344 19.628 21.002

0.0624 0.0583 0.0545 0.0509 0.0476

0.0047 0.0043 0.0040 0.0038 0.0035

0.0747 0.0743 0.0740 0.0738 0.0735

214.610 230.632 247.776 266.121 285.749

13.394 13.452 13.507 13.558 13.606

11.557 11.684 11.807 11.924 12.036

154.789 157.181 159.470 161.661 163.756

41 42 43 44 45

46 47 48 49 50

22.473 24.046 25.729 27.530 29.457

0.0445 0.0416 0.0389 0.0363 0.0339

0.0033 0.0030 0.0028 0.0026 0.0025

0.0733 0.0730 0.0728 0.0726 0.0725

306.752 329.224 353.270 378.999 406.529

13.650 13.692 13.730 13.767 13.801

12.143 12.246 12.345 12.439 12.529

165.758 167.671 169.498 171.242 172.905

46 47 48 49 50

7477_book.fm Page 272 Tuesday, March 13, 2007 3:34 PM

272

Computational Economic Analysis for Engineering and Industry

8% Period

n 1 2 3 4 5

Compound Interest Factors Single Payment Uniform Payment Series Compound Present Sinking Capital Compound Amount Value Fund Recovery Amount Factor Factor Factor Factor Factor Find F Find P Find A Find A Find F Given P Given F Given F Given P Given A F/P P/F A/F A/P F/A 1.080 0.9259 1.0000 1.0800 1.000 1.166 0.8573 0.4808 0.5608 2.080 1.260 0.7938 0.3080 0.3880 3.246 1.360 0.7350 0.2219 0.3019 4.506 1.469 0.6806 0.1705 0.2505 5.867

8%

Arithmetic Gradient Period Present Gradient Gradient Value Uniform Present Factor Series Value Find P Find A Find P Given A Given G Given G P/A A/G P/G n 0.926 0.000 0.000 1 1.783 0.481 0.857 2 2.577 0.949 2.445 3 3.312 1.404 4.650 4 3.993 1.846 7.372 5

6 7 8 9 10

1.587 1.714 1.851 1.999 2.159

0.6302 0.5835 0.5403 0.5002 0.4632

0.1363 0.1121 0.0940 0.0801 0.0690

0.2163 0.1921 0.1740 0.1601 0.1490

7.336 8.923 10.637 12.488 14.487

4.623 5.206 5.747 6.247 6.710

2.276 2.694 3.099 3.491 3.871

10.523 14.024 17.806 21.808 25.977

6 7 8 9 10

11 12 13 14 15

2.332 2.518 2.720 2.937 3.172

0.4289 0.3971 0.3677 0.3405 0.3152

0.0601 0.0527 0.0465 0.0413 0.0368

0.1401 0.1327 0.1265 0.1213 0.1168

16.645 18.977 21.495 24.215 27.152

7.139 7.536 7.904 8.244 8.559

4.240 4.596 4.940 5.273 5.594

30.266 34.634 39.046 43.472 47.886

11 12 13 14 15

16 17 18 19 20

3.426 3.700 3.996 4.316 4.661

0.2919 0.2703 0.2502 0.2317 0.2145

0.0330 0.0296 0.0267 0.0241 0.0219

0.1130 0.1096 0.1067 0.1041 0.1019

30.324 33.750 37.450 41.446 45.762

8.851 9.122 9.372 9.604 9.818

5.905 6.204 6.492 6.770 7.037

52.264 56.588 60.843 65.013 69.090

16 17 18 19 20

21 22 23 24 25

5.034 5.437 5.871 6.341 6.848

0.1987 0.1839 0.1703 0.1577 0.1460

0.0198 0.0180 0.0164 0.0150 0.0137

0.0998 0.0980 0.0964 0.0950 0.0937

50.423 55.457 60.893 66.765 73.106

10.017 10.201 10.371 10.529 10.675

7.294 7.541 7.779 8.007 8.225

73.063 76.926 80.673 84.300 87.804

21 22 23 24 25

26 27 28 29 30

7.396 7.988 8.627 9.317 10.063

0.1352 0.1252 0.1159 0.1073 0.0994

0.0125 0.0114 0.0105 0.0096 0.0088

0.0925 0.0914 0.0905 0.0896 0.0888

79.954 87.351 95.339 103.966 113.283

10.810 10.935 11.051 11.158 11.258

8.435 8.636 8.829 9.013 9.190

91.184 94.439 97.569 100.574 103.456

26 27 28 29 30

31 32 33 34 35

10.868 11.737 12.676 13.690 14.785

0.0920 0.0852 0.0789 0.0730 0.0676

0.0081 0.0075 0.0069 0.0063 0.0058

0.0881 0.0875 0.0869 0.0863 0.0858

123.346 134.214 145.951 158.627 172.317

11.350 11.435 11.514 11.587 11.655

9.358 9.520 9.674 9.821 9.961

106.216 108.857 111.382 113.792 116.092

31 32 33 34 35

36 37 38 39 40

15.968 17.246 18.625 20.115 21.725

0.0626 0.0580 0.0537 0.0497 0.0460

0.0053 0.0049 0.0045 0.0042 0.0039

0.0853 0.0849 0.0845 0.0842 0.0839

187.102 203.070 220.316 238.941 259.057

11.717 11.775 11.829 11.879 11.925

10.095 10.222 10.344 10.460 10.570

118.284 120.371 122.358 124.247 126.042

36 37 38 39 40

41 42 43 44 45

23.462 25.339 27.367 29.556 31.920

0.0426 0.0395 0.0365 0.0338 0.0313

0.0036 0.0033 0.0030 0.0028 0.0026

0.0836 0.0833 0.0830 0.0828 0.0826

280.781 304.244 329.583 356.950 386.506

11.967 12.007 12.043 12.077 12.108

10.675 10.774 10.869 10.959 11.045

127.747 129.365 130.900 132.355 133.733

41 42 43 44 45

46 47 48 49 50

34.474 37.232 40.211 43.427 46.902

0.0290 0.0269 0.0249 0.0230 0.0213

0.0024 0.0022 0.0020 0.0019 0.0017

0.0824 0.0822 0.0820 0.0819 0.0817

418.426 452.900 490.132 530.343 573.770

12.137 12.164 12.189 12.212 12.233

11.126 11.203 11.276 11.345 11.411

135.038 136.274 137.443 138.548 139.593

46 47 48 49 50

7477_book.fm Page 273 Tuesday, March 13, 2007 3:34 PM

Appendix E:

Interest factors and tables

9% Period

n 1 2 3 4 5

273

Compound Interest Factors Single Payment Uniform Payment Series Compound Present Sinking Capital Compound Amount Value Fund Recovery Amount Factor Factor Factor Factor Factor Find F Find P Find A Find A Find F Given P Given F Given F Given P Given A F/P P/F A/F A/P F/A 1.090 0.9174 1.0000 1.0900 1.000 1.188 0.8417 0.4785 0.5685 2.090 1.295 0.7722 0.3051 0.3951 3.278 1.412 0.7084 0.2187 0.3087 4.573 1.539 0.6499 0.1671 0.2571 5.985

9%

Arithmetic Gradient Period Present Gradient Gradient Value Uniform Present Factor Series Value Find P Find A Find P Given A Given G Given G P/A A/G P/G n 0.917 0.000 0.000 1 1.759 0.478 0.842 2 2.531 0.943 2.386 3 3.240 1.393 4.511 4 3.890 1.828 7.111 5

6 7 8 9 10

1.677 1.828 1.993 2.172 2.367

0.5963 0.5470 0.5019 0.4604 0.4224

0.1329 0.1087 0.0907 0.0768 0.0658

0.2229 0.1987 0.1807 0.1668 0.1558

7.523 9.200 11.028 13.021 15.193

4.486 5.033 5.535 5.995 6.418

2.250 2.657 3.051 3.431 3.798

10.092 13.375 16.888 20.571 24.373

6 7 8 9 10

11 12 13 14 15

2.580 2.813 3.066 3.342 3.642

0.3875 0.3555 0.3262 0.2992 0.2745

0.0569 0.0497 0.0436 0.0384 0.0341

0.1469 0.1397 0.1336 0.1284 0.1241

17.560 20.141 22.953 26.019 29.361

6.805 7.161 7.487 7.786 8.061

4.151 4.491 4.818 5.133 5.435

28.248 32.159 36.073 39.963 43.807

11 12 13 14 15

16 17 18 19 20

3.970 4.328 4.717 5.142 5.604

0.2519 0.2311 0.2120 0.1945 0.1784

0.0303 0.0270 0.0242 0.0217 0.0195

0.1203 0.1170 0.1142 0.1117 0.1095

33.003 36.974 41.301 46.018 51.160

8.313 8.544 8.756 8.950 9.129

5.724 6.002 6.269 6.524 6.767

47.585 51.282 54.886 58.387 61.777

16 17 18 19 20

21 22 23 24 25

6.109 6.659 7.258 7.911 8.623

0.1637 0.1502 0.1378 0.1264 0.1160

0.0176 0.0159 0.0144 0.0130 0.0118

0.1076 0.1059 0.1044 0.1030 0.1018

56.765 62.873 69.532 76.790 84.701

9.292 9.442 9.580 9.707 9.823

7.001 7.223 7.436 7.638 7.832

65.051 68.205 71.236 74.143 76.926

21 22 23 24 25

26 27 28 29 30

9.399 10.245 11.167 12.172 13.268

0.1064 0.0976 0.0895 0.0822 0.0754

0.0107 0.0097 0.0089 0.0081 0.0073

0.1007 0.0997 0.0989 0.0981 0.0973

93.324 102.723 112.968 124.135 136.308

9.929 10.027 10.116 10.198 10.274

8.016 8.191 8.357 8.515 8.666

79.586 82.124 84.542 86.842 89.028

26 27 28 29 30

31 32 33 34 35

14.462 15.763 17.182 18.728 20.414

0.0691 0.0634 0.0582 0.0534 0.0490

0.0067 0.0061 0.0056 0.0051 0.0046

0.0967 0.0961 0.0956 0.0951 0.0946

149.575 164.037 179.800 196.982 215.711

10.343 10.406 10.464 10.518 10.567

8.808 8.944 9.072 9.193 9.308

91.102 93.069 94.931 96.693 98.359

31 32 33 34 35

36 37 38 39 40

22.251 24.254 26.437 28.816 31.409

0.0449 0.0412 0.0378 0.0347 0.0318

0.0042 0.0039 0.0035 0.0032 0.0030

0.0942 0.0939 0.0935 0.0932 0.0930

236.125 258.376 282.630 309.066 337.882

10.612 10.653 10.691 10.726 10.757

9.417 9.520 9.617 9.709 9.796

99.932 101.416 102.816 104.135 105.376

36 37 38 39 40

41 42 43 44 45

34.236 37.318 40.676 44.337 48.327

0.0292 0.0268 0.0246 0.0226 0.0207

0.0027 0.0025 0.0023 0.0021 0.0019

0.0927 0.0925 0.0923 0.0921 0.0919

369.292 403.528 440.846 481.522 525.859

10.787 10.813 10.838 10.861 10.881

9.878 9.955 10.027 10.096 10.160

106.545 107.643 108.676 109.646 110.556

41 42 43 44 45

46 47 48 49 50

52.677 57.418 62.585 68.218 74.358

0.0190 0.0174 0.0160 0.0147 0.0134

0.0017 0.0016 0.0015 0.0013 0.0012

0.0917 0.0916 0.0915 0.0913 0.0912

574.186 626.863 684.280 746.866 815.084

10.900 10.918 10.934 10.948 10.962

10.221 10.278 10.332 10.382 10.430

111.410 112.211 112.962 113.666 114.325

46 47 48 49 50

7477_book.fm Page 274 Tuesday, March 13, 2007 3:34 PM

274

Computational Economic Analysis for Engineering and Industry

10% Period

n 1 2 3 4 5

Compound Interest Factors Single Payment Uniform Payment Series Compound Present Sinking Capital Compound Amount Value Fund Recovery Amount Factor Factor Factor Factor Factor Find F Find P Find A Find A Find F Given P Given F Given F Given P Given A F/P P/F A/F A/P F/A 1.100 0.9091 1.0000 1.1000 1.000 1.210 0.8264 0.4762 0.5762 2.100 1.331 0.7513 0.3021 0.4021 3.310 1.464 0.6830 0.2155 0.3155 4.641 1.611 0.6209 0.1638 0.2638 6.105

10%

Arithmetic Gradient Period Present Gradient Gradient Value Uniform Present Factor Series Value Find P Find A Find P Given A Given G Given G P/A A/G P/G n 0.909 0.000 0.000 1 1.736 0.476 0.826 2 2.487 0.937 2.329 3 3.170 1.381 4.378 4 3.791 1.810 6.862 5

6 7 8 9 10

1.772 1.949 2.144 2.358 2.594

0.5645 0.5132 0.4665 0.4241 0.3855

0.1296 0.1054 0.0874 0.0736 0.0627

0.2296 0.2054 0.1874 0.1736 0.1627

7.716 9.487 11.436 13.579 15.937

4.355 4.868 5.335 5.759 6.145

2.224 2.622 3.004 3.372 3.725

9.684 12.763 16.029 19.421 22.891

6 7 8 9 10

11 12 13 14 15

2.853 3.138 3.452 3.797 4.177

0.3505 0.3186 0.2897 0.2633 0.2394

0.0540 0.0468 0.0408 0.0357 0.0315

0.1540 0.1468 0.1408 0.1357 0.1315

18.531 21.384 24.523 27.975 31.772

6.495 6.814 7.103 7.367 7.606

4.064 4.388 4.699 4.996 5.279

26.396 29.901 33.377 36.800 40.152

11 12 13 14 15

16 17 18 19 20

4.595 5.054 5.560 6.116 6.727

0.2176 0.1978 0.1799 0.1635 0.1486

0.0278 0.0247 0.0219 0.0195 0.0175

0.1278 0.1247 0.1219 0.1195 0.1175

35.950 40.545 45.599 51.159 57.275

7.824 8.022 8.201 8.365 8.514

5.549 5.807 6.053 6.286 6.508

43.416 46.582 49.640 52.583 55.407

16 17 18 19 20

21 22 23 24 25

7.400 8.140 8.954 9.850 10.835

0.1351 0.1228 0.1117 0.1015 0.0923

0.0156 0.0140 0.0126 0.0113 0.0102

0.1156 0.1140 0.1126 0.1113 0.1102

64.002 71.403 79.543 88.497 98.347

8.649 8.772 8.883 8.985 9.077

6.719 6.919 7.108 7.288 7.458

58.110 60.689 63.146 65.481 67.696

21 22 23 24 25

26 27 28 29 30

11.918 13.110 14.421 15.863 17.449

0.0839 0.0763 0.0693 0.0630 0.0573

0.0092 0.0083 0.0075 0.0067 0.0061

0.1092 0.1083 0.1075 0.1067 0.1061

109.182 121.100 134.210 148.631 164.494

9.161 9.237 9.307 9.370 9.427

7.619 7.770 7.914 8.049 8.176

69.794 71.777 73.650 75.415 77.077

26 27 28 29 30

31 32 33 34 35

19.194 21.114 23.225 25.548 28.102

0.0521 0.0474 0.0431 0.0391 0.0356

0.0055 0.0050 0.0045 0.0041 0.0037

0.1055 0.1050 0.1045 0.1041 0.1037

181.943 201.138 222.252 245.477 271.024

9.479 9.526 9.569 9.609 9.644

8.296 8.409 8.515 8.615 8.709

78.640 80.108 81.486 82.777 83.987

31 32 33 34 35

36 37 38 39 40

30.913 34.004 37.404 41.145 45.259

0.0323 0.0294 0.0267 0.0243 0.0221

0.0033 0.0030 0.0027 0.0025 0.0023

0.1033 0.1030 0.1027 0.1025 0.1023

299.127 330.039 364.043 401.448 442.593

9.677 9.706 9.733 9.757 9.779

8.796 8.879 8.956 9.029 9.096

85.119 86.178 87.167 88.091 88.953

36 37 38 39 40

41 42 43 44 45

49.785 54.764 60.240 66.264 72.890

0.0201 0.0183 0.0166 0.0151 0.0137

0.0020 0.0019 0.0017 0.0015 0.0014

0.1020 0.1019 0.1017 0.1015 0.1014

487.852 537.637 592.401 652.641 718.905

9.799 9.817 9.834 9.849 9.863

9.160 9.219 9.274 9.326 9.374

89.756 90.505 91.202 91.851 92.454

41 42 43 44 45

46 47 48 49 50

80.180 88.197 97.017 106.719 117.391

0.0125 0.0113 0.0103 0.0094 0.0085

0.0013 0.0011 0.0010 0.0009 0.0009

0.1013 0.1011 0.1010 0.1009 0.1009

791.795 871.975 960.172 1057.190 1163.909

9.875 9.887 9.897 9.906 9.915

9.419 9.461 9.500 9.537 9.570

93.016 93.537 94.022 94.471 94.889

46 47 48 49 50

7477_book.fm Page 275 Tuesday, March 13, 2007 3:34 PM

Appendix E:

Interest factors and tables

12% Period

n 1 2 3 4 5

275

Compound Interest Factors Single Payment Uniform Payment Series Compound Present Sinking Capital Compound Amount Value Fund Recovery Amount Factor Factor Factor Factor Factor Find F Find P Find A Find A Find F Given P Given F Given F Given P Given A F/P P/F A/F A/P F/A 1.120 0.8929 1.0000 1.1200 1.000 1.254 0.7972 0.4717 0.5917 2.120 1.405 0.7118 0.2963 0.4163 3.374 1.574 0.6355 0.2092 0.3292 4.779 1.762 0.5674 0.1574 0.2774 6.353

12%

Arithmetic Gradient Period Present Gradient Gradient Value Uniform Present Factor Series Value Find P Find A Find P Given A Given G Given G P/A A/G P/G n 0.893 0.000 0.000 1 1.690 0.472 0.797 2 2.402 0.925 2.221 3 3.037 1.359 4.127 4 3.605 1.775 6.397 5

6 7 8 9 10

1.974 2.211 2.476 2.773 3.106

0.5066 0.4523 0.4039 0.3606 0.3220

0.1232 0.0991 0.0813 0.0677 0.0570

0.2432 0.2191 0.2013 0.1877 0.1770

8.115 10.089 12.300 14.776 17.549

4.111 4.564 4.968 5.328 5.650

2.172 2.551 2.913 3.257 3.585

8.930 11.644 14.471 17.356 20.254

6 7 8 9 10

11 12 13 14 15

3.479 3.896 4.363 4.887 5.474

0.2875 0.2567 0.2292 0.2046 0.1827

0.0484 0.0414 0.0357 0.0309 0.0268

0.1684 0.1614 0.1557 0.1509 0.1468

20.655 24.133 28.029 32.393 37.280

5.938 6.194 6.424 6.628 6.811

3.895 4.190 4.468 4.732 4.980

23.129 25.952 28.702 31.362 33.920

11 12 13 14 15

16 17 18 19 20

6.130 6.866 7.690 8.613 9.646

0.1631 0.1456 0.1300 0.1161 0.1037

0.0234 0.0205 0.0179 0.0158 0.0139

0.1434 0.1405 0.1379 0.1358 0.1339

42.753 48.884 55.750 63.440 72.052

6.974 7.120 7.250 7.366 7.469

5.215 5.435 5.643 5.838 6.020

36.367 38.697 40.908 42.998 44.968

16 17 18 19 20

21 22 23 24 25

10.804 12.100 13.552 15.179 17.000

0.0926 0.0826 0.0738 0.0659 0.0588

0.0122 0.0108 0.0096 0.0085 0.0075

0.1322 0.1308 0.1296 0.1285 0.1275

81.699 92.503 104.603 118.155 133.334

7.562 7.645 7.718 7.784 7.843

6.191 6.351 6.501 6.641 6.771

46.819 48.554 50.178 51.693 53.105

21 22 23 24 25

26 27 28 29 30

19.040 21.325 23.884 26.750 29.960

0.0525 0.0469 0.0419 0.0374 0.0334

0.0067 0.0059 0.0052 0.0047 0.0041

0.1267 0.1259 0.1252 0.1247 0.1241

150.334 169.374 190.699 214.583 241.333

7.896 7.943 7.984 8.022 8.055

6.892 7.005 7.110 7.207 7.297

54.418 55.637 56.767 57.814 58.782

26 27 28 29 30

31 32 33 34 35

33.555 37.582 42.092 47.143 52.800

0.0298 0.0266 0.0238 0.0212 0.0189

0.0037 0.0033 0.0029 0.0026 0.0023

0.1237 0.1233 0.1229 0.1226 0.1223

271.293 304.848 342.429 384.521 431.663

8.085 8.112 8.135 8.157 8.176

7.381 7.459 7.530 7.596 7.658

59.676 60.501 61.261 61.961 62.605

31 32 33 34 35

36 37 38 39 40

59.136 66.232 74.180 83.081 93.051

0.0169 0.0151 0.0135 0.0120 0.0107

0.0021 0.0018 0.0016 0.0015 0.0013

0.1221 0.1218 0.1216 0.1215 0.1213

484.463 543.599 609.831 684.010 767.091

8.192 8.208 8.221 8.233 8.244

7.714 7.766 7.814 7.858 7.899

63.197 63.741 64.239 64.697 65.116

36 37 38 39 40

41 42 43 44 45

104.217 116.723 130.730 146.418 163.988

0.0096 0.0086 0.0076 0.0068 0.0061

0.0012 0.0010 0.0009 0.0008 0.0007

0.1212 0.1210 0.1209 0.1208 0.1207

860.142 964.359 1081.083 1211.813 1358.230

8.253 8.262 8.270 8.276 8.283

7.936 7.970 8.002 8.031 8.057

65.500 65.851 66.172 66.466 66.734

41 42 43 44 45

46 47 48 49 50

183.666 205.706 230.391 258.038 289.002

0.0054 0.0049 0.0043 0.0039 0.0035

0.0007 0.0006 0.0005 0.0005 0.0004

0.1207 0.1206 0.1205 0.1205 0.1204

1522.218 1705.884 1911.590 2141.981 2400.018

8.288 8.293 8.297 8.301 8.304

8.082 8.104 8.124 8.143 8.160

66.979 67.203 67.407 67.593 67.762

46 47 48 49 50

7477_book.fm Page 276 Tuesday, March 13, 2007 3:34 PM

276

Computational Economic Analysis for Engineering and Industry

14% Period

n 1 2 3 4 5

Compound Interest Factors Single Payment Uniform Payment Series Compound Present Sinking Capital Compound Amount Value Fund Recovery Amount Factor Factor Factor Factor Factor Find F Find P Find A Find A Find F Given P Given F Given F Given P Given A F/P P/F A/F A/P F/A 1.140 0.8772 1.0000 1.1400 1.000 1.300 0.7695 0.4673 0.6073 2.140 1.482 0.6750 0.2907 0.4307 3.440 1.689 0.5921 0.2032 0.3432 4.921 1.925 0.5194 0.1513 0.2913 6.610

14%

Arithmetic Gradient Period Present Gradient Gradient Value Uniform Present Factor Series Value Find P Find A Find P Given A Given G Given G P/A A/G P/G n 0.877 0.000 0.000 1 1.647 0.467 0.769 2 2.322 0.913 2.119 3 2.914 1.337 3.896 4 3.433 1.740 5.973 5

6 7 8 9 10

2.195 2.502 2.853 3.252 3.707

0.4556 0.3996 0.3506 0.3075 0.2697

0.1172 0.0932 0.0756 0.0622 0.0517

0.2572 0.2332 0.2156 0.2022 0.1917

8.536 10.730 13.233 16.085 19.337

3.889 4.288 4.639 4.946 5.216

2.122 2.483 2.825 3.146 3.449

8.251 10.649 13.103 15.563 17.991

6 7 8 9 10

11 12 13 14 15

4.226 4.818 5.492 6.261 7.138

0.2366 0.2076 0.1821 0.1597 0.1401

0.0434 0.0367 0.0312 0.0266 0.0228

0.1834 0.1767 0.1712 0.1666 0.1628

23.045 27.271 32.089 37.581 43.842

5.453 5.660 5.842 6.002 6.142

3.733 4.000 4.249 4.482 4.699

20.357 22.640 24.825 26.901 28.862

11 12 13 14 15

16 17 18 19 20

8.137 9.276 10.575 12.056 13.743

0.1229 0.1078 0.0946 0.0829 0.0728

0.0196 0.0169 0.0146 0.0127 0.0110

0.1596 0.1569 0.1546 0.1527 0.1510

50.980 59.118 68.394 78.969 91.025

6.265 6.373 6.467 6.550 6.623

4.901 5.089 5.263 5.424 5.573

30.706 32.430 34.038 35.531 36.914

16 17 18 19 20

21 22 23 24 25

15.668 17.861 20.362 23.212 26.462

0.0638 0.0560 0.0491 0.0431 0.0378

0.0095 0.0083 0.0072 0.0063 0.0055

0.1495 0.1483 0.1472 0.1463 0.1455

104.768 120.436 138.297 158.659 181.871

6.687 6.743 6.792 6.835 6.873

5.711 5.838 5.955 6.062 6.161

38.190 39.366 40.446 41.437 42.344

21 22 23 24 25

26 27 28 29 30

30.167 34.390 39.204 44.693 50.950

0.0331 0.0291 0.0255 0.0224 0.0196

0.0048 0.0042 0.0037 0.0032 0.0028

0.1448 0.1442 0.1437 0.1432 0.1428

208.333 238.499 272.889 312.094 356.787

6.906 6.935 6.961 6.983 7.003

6.251 6.334 6.410 6.479 6.542

43.173 43.929 44.618 45.244 45.813

26 27 28 29 30

31 32 33 34 35

58.083 66.215 75.485 86.053 98.100

0.0172 0.0151 0.0132 0.0116 0.0102

0.0025 0.0021 0.0019 0.0016 0.0014

0.1425 0.1421 0.1419 0.1416 0.1414

407.737 465.820 532.035 607.520 693.573

7.020 7.035 7.048 7.060 7.070

6.600 6.652 6.700 6.743 6.782

46.330 46.798 47.222 47.605 47.952

31 32 33 34 35

36 37 38 39 40

111.834 127.491 145.340 165.687 188.884

0.0089 0.0078 0.0069 0.0060 0.0053

0.0013 0.0011 0.0010 0.0009 0.0007

0.1413 0.1411 0.1410 0.1409 0.1407

791.673 903.507 1030.998 1176.338 1342.025

7.079 7.087 7.094 7.100 7.105

6.818 6.850 6.880 6.906 6.930

48.265 48.547 48.802 49.031 49.238

36 37 38 39 40

41 42 43 44 45

215.327 245.473 279.839 319.017 363.679

0.0046 0.0041 0.0036 0.0031 0.0027

0.0007 0.0006 0.0005 0.0004 0.0004

0.1407 0.1406 0.1405 0.1404 0.1404

1530.909 1746.236 1991.709 2271.548 2590.565

7.110 7.114 7.117 7.120 7.123

6.952 6.971 6.989 7.004 7.019

49.423 49.590 49.741 49.875 49.996

41 42 43 44 45

46 47 48 49 50

414.594 472.637 538.807 614.239 700.233

0.0024 0.0021 0.0019 0.0016 0.0014

0.0003 0.0003 0.0003 0.0002 0.0002

0.1403 0.1403 0.1403 0.1402 0.1402

2954.244 3368.838 3841.475 4380.282 4994.521

7.126 7.128 7.130 7.131 7.133

7.032 7.043 7.054 7.063 7.071

50.105 50.202 50.289 50.368 50.438

46 47 48 49 50

7477_book.fm Page 277 Tuesday, March 13, 2007 3:34 PM

Appendix E:

Interest factors and tables

16% Period

n 1 2 3 4 5

277

Compound Interest Factors Single Payment Uniform Payment Series Compound Present Sinking Capital Compound Amount Value Fund Recovery Amount Factor Factor Factor Factor Factor Find F Find P Find A Find A Find F Given P Given F Given F Given P Given A F/P P/F A/F A/P F/A 1.160 0.8621 1.0000 1.1600 1.000 1.346 0.7432 0.4630 0.6230 2.160 1.561 0.6407 0.2853 0.4453 3.506 1.811 0.5523 0.1974 0.3574 5.066 2.100 0.4761 0.1454 0.3054 6.877

16%

Arithmetic Gradient Period Present Gradient Gradient Value Uniform Present Factor Series Value Find P Find A Find P Given A Given G Given G P/A A/G P/G n 0.862 0.000 0.000 1 1.605 0.463 0.743 2 2.246 0.901 2.024 3 2.798 1.316 3.681 4 3.274 1.706 5.586 5

6 7 8 9 10

2.436 2.826 3.278 3.803 4.411

0.4104 0.3538 0.3050 0.2630 0.2267

0.1114 0.0876 0.0702 0.0571 0.0469

0.2714 0.2476 0.2302 0.2171 0.2069

8.977 11.414 14.240 17.519 21.321

3.685 4.039 4.344 4.607 4.833

2.073 2.417 2.739 3.039 3.319

7.638 9.761 11.896 14.000 16.040

6 7 8 9 10

11 12 13 14 15

5.117 5.936 6.886 7.988 9.266

0.1954 0.1685 0.1452 0.1252 0.1079

0.0389 0.0324 0.0272 0.0229 0.0194

0.1989 0.1924 0.1872 0.1829 0.1794

25.733 30.850 36.786 43.672 51.660

5.029 5.197 5.342 5.468 5.575

3.578 3.819 4.041 4.246 4.435

17.994 19.847 21.590 23.217 24.728

11 12 13 14 15

16 17 18 19 20

10.748 12.468 14.463 16.777 19.461

0.0930 0.0802 0.0691 0.0596 0.0514

0.0164 0.0140 0.0119 0.0101 0.0087

0.1764 0.1740 0.1719 0.1701 0.1687

60.925 71.673 84.141 98.603 115.380

5.668 5.749 5.818 5.877 5.929

4.609 4.768 4.913 5.046 5.167

26.124 27.407 28.583 29.656 30.632

16 17 18 19 20

21 22 23 24 25

22.574 26.186 30.376 35.236 40.874

0.0443 0.0382 0.0329 0.0284 0.0245

0.0074 0.0064 0.0054 0.0047 0.0040

0.1674 0.1664 0.1654 0.1647 0.1640

134.841 157.415 183.601 213.978 249.214

5.973 6.011 6.044 6.073 6.097

5.277 5.377 5.467 5.549 5.623

31.518 32.320 33.044 33.697 34.284

21 22 23 24 25

26 27 28 29 30

47.414 55.000 63.800 74.009 85.850

0.0211 0.0182 0.0157 0.0135 0.0116

0.0034 0.0030 0.0025 0.0022 0.0019

0.1634 0.1630 0.1625 0.1622 0.1619

290.088 337.502 392.503 456.303 530.312

6.118 6.136 6.152 6.166 6.177

5.690 5.750 5.804 5.853 5.896

34.811 35.284 35.707 36.086 36.423

26 27 28 29 30

31 32 33 34 35

99.586 115.520 134.003 155.443 180.314

0.0100 0.0087 0.0075 0.0064 0.0055

0.0016 0.0014 0.0012 0.0010 0.0009

0.1616 0.1614 0.1612 0.1610 0.1609

616.162 715.747 831.267 965.270 1120.713

6.187 6.196 6.203 6.210 6.215

5.936 5.971 6.002 6.030 6.055

36.725 36.993 37.232 37.444 37.633

31 32 33 34 35

36 37 38 39 40

209.164 242.631 281.452 326.484 378.721

0.0048 0.0041 0.0036 0.0031 0.0026

0.0008 0.0007 0.0006 0.0005 0.0004

0.1608 0.1607 0.1606 0.1605 0.1604

1301.027 1510.191 1752.822 2034.273 2360.757

6.220 6.224 6.228 6.231 6.233

6.077 6.097 6.115 6.130 6.144

37.800 37.948 38.080 38.196 38.299

36 37 38 39 40

41 42 43 44 45

439.317 509.607 591.144 685.727 795.444

0.0023 0.0020 0.0017 0.0015 0.0013

0.0004 0.0003 0.0003 0.0002 0.0002

0.1604 0.1603 0.1603 0.1602 0.1602

2739.478 3178.795 3688.402 4279.546 4965.274

6.236 6.238 6.239 6.241 6.242

6.156 6.167 6.177 6.186 6.193

38.390 38.471 38.542 38.605 38.660

41 42 43 44 45

46 47 48 49 50

922.715 1070.349 1241.605 1440.262 1670.704

0.0011 0.0009 0.0008 0.0007 0.0006

0.0002 0.0001 0.0001 0.0001 0.0001

0.1602 0.1601 0.1601 0.1601 0.1601

5760.718 6683.433 7753.782 8995.387 10435.649

6.243 6.244 6.245 6.246 6.246

6.200 6.206 6.211 6.216 6.220

38.709 38.752 38.789 38.823 38.852

46 47 48 49 50

7477_book.fm Page 278 Tuesday, March 13, 2007 3:34 PM

278

Computational Economic Analysis for Engineering and Industry

18% Period

n 1 2 3 4 5

Compound Interest Factors Single Payment Uniform Payment Series Compound Present Sinking Capital Compound Amount Value Fund Recovery Amount Factor Factor Factor Factor Factor Find F Find P Find A Find A Find F Given P Given F Given F Given P Given A F/P P/F A/F A/P F/A 1.180 0.8475 1.0000 1.1800 1.000 1.392 0.7182 0.4587 0.6387 2.180 1.643 0.6086 0.2799 0.4599 3.572 1.939 0.5158 0.1917 0.3717 5.215 2.288 0.4371 0.1398 0.3198 7.154

18%

Arithmetic Gradient Period Present Gradient Gradient Value Uniform Present Factor Series Value Find P Find A Find P Given A Given G Given G P/A A/G P/G n 0.847 0.000 0.000 1 1.566 0.459 0.718 2 2.174 0.890 1.935 3 2.690 1.295 3.483 4 3.127 1.673 5.231 5

6 7 8 9 10

2.700 3.185 3.759 4.435 5.234

0.3704 0.3139 0.2660 0.2255 0.1911

0.1059 0.0824 0.0652 0.0524 0.0425

0.2859 0.2624 0.2452 0.2324 0.2225

9.442 12.142 15.327 19.086 23.521

3.498 3.812 4.078 4.303 4.494

2.025 2.353 2.656 2.936 3.194

7.083 8.967 10.829 12.633 14.352

6 7 8 9 10

11 12 13 14 15

6.176 7.288 8.599 10.147 11.974

0.1619 0.1372 0.1163 0.0985 0.0835

0.0348 0.0286 0.0237 0.0197 0.0164

0.2148 0.2086 0.2037 0.1997 0.1964

28.755 34.931 42.219 50.818 60.965

4.656 4.793 4.910 5.008 5.092

3.430 3.647 3.845 4.025 4.189

15.972 17.481 18.877 20.158 21.327

11 12 13 14 15

16 17 18 19 20

14.129 16.672 19.673 23.214 27.393

0.0708 0.0600 0.0508 0.0431 0.0365

0.0137 0.0115 0.0096 0.0081 0.0068

0.1937 0.1915 0.1896 0.1881 0.1868

72.939 87.068 103.740 123.414 146.628

5.162 5.222 5.273 5.316 5.353

4.337 4.471 4.592 4.700 4.798

22.389 23.348 24.212 24.988 25.681

16 17 18 19 20

21 22 23 24 25

32.324 38.142 45.008 53.109 62.669

0.0309 0.0262 0.0222 0.0188 0.0160

0.0057 0.0048 0.0041 0.0035 0.0029

0.1857 0.1848 0.1841 0.1835 0.1829

174.021 206.345 244.487 289.494 342.603

5.384 5.410 5.432 5.451 5.467

4.885 4.963 5.033 5.095 5.150

26.300 26.851 27.339 27.772 28.155

21 22 23 24 25

26 27 28 29 30

73.949 87.260 102.967 121.501 143.371

0.0135 0.0115 0.0097 0.0082 0.0070

0.0025 0.0021 0.0018 0.0015 0.0013

0.1825 0.1821 0.1818 0.1815 0.1813

405.272 479.221 566.481 669.447 790.948

5.480 5.492 5.502 5.510 5.517

5.199 5.243 5.281 5.315 5.345

28.494 28.791 29.054 29.284 29.486

26 27 28 29 30

31 32 33 34 35

169.177 199.629 235.563 277.964 327.997

0.0059 0.0050 0.0042 0.0036 0.0030

0.0011 0.0009 0.0008 0.0006 0.0006

0.1811 0.1809 0.1808 0.1806 0.1806

934.319 1103.496 1303.125 1538.688 1816.652

5.523 5.528 5.532 5.536 5.539

5.371 5.394 5.415 5.433 5.449

29.664 29.819 29.955 30.074 30.177

31 32 33 34 35

36 37 38 39 40

387.037 456.703 538.910 635.914 750.378

0.0026 0.0022 0.0019 0.0016 0.0013

0.0005 0.0004 0.0003 0.0003 0.0002

0.1805 0.1804 0.1803 0.1803 0.1802

2144.649 2531.686 2988.389 3527.299 4163.213

5.541 5.543 5.545 5.547 5.548

5.462 5.474 5.485 5.494 5.502

30.268 30.347 30.415 30.475 30.527

36 37 38 39 40

41 42 43 44 45

885.446 1044.827 1232.896 1454.817 1716.684

0.0011 0.0010 0.0008 0.0007 0.0006

0.0002 0.0002 0.0001 0.0001 0.0001

0.1802 0.1802 0.1801 0.1801 0.1801

4913.591 5799.038 6843.865 8076.760 9531.577

5.549 5.550 5.551 5.552 5.552

5.509 5.515 5.521 5.525 5.529

30.572 30.611 30.645 30.675 30.701

41 42 43 44 45

46 47 48 49 50

2025.687 2390.311 2820.567 3328.269 3927.357

0.0005 0.0004 0.0004 0.0003 0.0003

0.0001 0.0001 0.0001 0.0001 0.0000

0.1801 0.1801 0.1801 0.1801 0.1800

11248.261 13273.948 15664.259 18484.825 21813.094

5.553 5.553 5.554 5.554 5.554

5.533 5.536 5.539 5.541 5.543

30.723 30.742 30.759 30.773 30.786

46 47 48 49 50

7477_book.fm Page 279 Tuesday, March 13, 2007 3:34 PM

Appendix E:

Interest factors and tables

20% Period

n 1 2 3 4 5

279

Compound Interest Factors Single Payment Uniform Payment Series Compound Present Sinking Capital Compound Amount Value Fund Recovery Amount Factor Factor Factor Factor Factor Find F Find P Find A Find A Find F Given P Given F Given F Given P Given A F/P P/F A/F A/P F/A 1.200 0.8333 1.0000 1.2000 1.000 1.440 0.6944 0.4545 0.6545 2.200 1.728 0.5787 0.2747 0.4747 3.640 2.074 0.4823 0.1863 0.3863 5.368 2.488 0.4019 0.1344 0.3344 7.442

20%

Arithmetic Gradient Period Present Gradient Gradient Value Uniform Present Factor Series Value Find P Find A Find P Given A Given G Given G P/A A/G P/G n 0.833 0.000 0.000 1 1.528 0.455 0.694 2 2.106 0.879 1.852 3 2.589 1.274 3.299 4 2.991 1.641 4.906 5

6 7 8 9 10

2.986 3.583 4.300 5.160 6.192

0.3349 0.2791 0.2326 0.1938 0.1615

0.1007 0.0774 0.0606 0.0481 0.0385

0.3007 0.2774 0.2606 0.2481 0.2385

9.930 12.916 16.499 20.799 25.959

3.326 3.605 3.837 4.031 4.192

1.979 2.290 2.576 2.836 3.074

6.581 8.255 9.883 11.434 12.887

6 7 8 9 10

11 12 13 14 15

7.430 8.916 10.699 12.839 15.407

0.1346 0.1122 0.0935 0.0779 0.0649

0.0311 0.0253 0.0206 0.0169 0.0139

0.2311 0.2253 0.2206 0.2169 0.2139

32.150 39.581 48.497 59.196 72.035

4.327 4.439 4.533 4.611 4.675

3.289 3.484 3.660 3.817 3.959

14.233 15.467 16.588 17.601 18.509

11 12 13 14 15

16 17 18 19 20

18.488 22.186 26.623 31.948 38.338

0.0541 0.0451 0.0376 0.0313 0.0261

0.0114 0.0094 0.0078 0.0065 0.0054

0.2114 0.2094 0.2078 0.2065 0.2054

87.442 105.931 128.117 154.740 186.688

4.730 4.775 4.812 4.843 4.870

4.085 4.198 4.298 4.386 4.464

19.321 20.042 20.680 21.244 21.739

16 17 18 19 20

21 22 23 24 25

46.005 55.206 66.247 79.497 95.396

0.0217 0.0181 0.0151 0.0126 0.0105

0.0044 0.0037 0.0031 0.0025 0.0021

0.2044 0.2037 0.2031 0.2025 0.2021

225.026 271.031 326.237 392.484 471.981

4.891 4.909 4.925 4.937 4.948

4.533 4.594 4.647 4.694 4.735

22.174 22.555 22.887 23.176 23.428

21 22 23 24 25

26 27 28 29 30

114.475 137.371 164.845 197.814 237.376

0.0087 0.0073 0.0061 0.0051 0.0042

0.0018 0.0015 0.0012 0.0010 0.0008

0.2018 0.2015 0.2012 0.2010 0.2008

567.377 681.853 819.223 984.068 1181.882

4.956 4.964 4.970 4.975 4.979

4.771 4.802 4.829 4.853 4.873

23.646 23.835 23.999 24.141 24.263

26 27 28 29 30

31 32 33 34 35

284.852 341.822 410.186 492.224 590.668

0.0035 0.0029 0.0024 0.0020 0.0017

0.0007 0.0006 0.0005 0.0004 0.0003

0.2007 0.2006 0.2005 0.2004 0.2003

1419.258 1704.109 2045.931 2456.118 2948.341

4.982 4.985 4.988 4.990 4.992

4.891 4.906 4.919 4.931 4.941

24.368 24.459 24.537 24.604 24.661

31 32 33 34 35

36 37 38 39 40

708.802 850.562 1020.675 1224.810 1469.772

0.0014 0.0012 0.0010 0.0008 0.0007

0.0003 0.0002 0.0002 0.0002 0.0001

0.2003 0.2002 0.2002 0.2002 0.2001

3539.009 4247.811 5098.373 6119.048 7343.858

4.993 4.994 4.995 4.996 4.997

4.949 4.956 4.963 4.968 4.973

24.711 24.753 24.789 24.820 24.847

36 37 38 39 40

41 42 43 44 45

1763.726 2116.471 2539.765 3047.718 3657.262

0.0006 0.0005 0.0004 0.0003 0.0003

0.0001 0.0001 0.0001 0.0001 0.0001

0.2001 0.2001 0.2001 0.2001 0.2001

8813.629 10577.355 12693.826 15233.592 18281.310

4.997 4.998 4.998 4.998 4.999

4.977 4.980 4.983 4.986 4.988

24.870 24.889 24.906 24.920 24.932

41 42 43 44 45

46 47 48 49 50

4388.714 5266.457 6319.749 7583.698 9100.438

0.0002 0.0002 0.0002 0.0001 0.0001

0.0000 0.0000 0.0000 0.0000 0.0000

0.2000 0.2000 0.2000 0.2000 0.2000

21938.572 26327.286 31593.744 37913.492 45497.191

4.999 4.999 4.999 4.999 4.999

4.990 4.991 4.992 4.994 4.995

24.942 24.951 24.958 24.964 24.970

46 47 48 49 50

7477_book.fm Page 280 Tuesday, March 13, 2007 3:34 PM

280

Computational Economic Analysis for Engineering and Industry

25% Period

n 1 2 3 4 5

Compound Interest Factors Single Payment Uniform Payment Series Compound Present Sinking Capital Compound Amount Value Fund Recovery Amount Factor Factor Factor Factor Factor Find F Find P Find A Find A Find F Given P Given F Given F Given P Given A F/P P/F A/F A/P F/A 1.250 0.8000 1.0000 1.2500 1.000 1.563 0.6400 0.4444 0.6944 2.250 1.953 0.5120 0.2623 0.5123 3.813 2.441 0.4096 0.1734 0.4234 5.766 3.052 0.3277 0.1218 0.3718 8.207

25%

Arithmetic Gradient Period Present Gradient Gradient Value Uniform Present Factor Series Value Find P Find A Find P Given A Given G Given G P/A A/G P/G n 0.800 0.000 0.000 1 1.440 0.444 0.640 2 1.952 0.852 1.664 3 2.362 1.225 2.893 4 2.689 1.563 4.204 5

6 7 8 9 10

3.815 4.768 5.960 7.451 9.313

0.2621 0.2097 0.1678 0.1342 0.1074

0.0888 0.0663 0.0504 0.0388 0.0301

0.3388 0.3163 0.3004 0.2888 0.2801

11.259 15.073 19.842 25.802 33.253

2.951 3.161 3.329 3.463 3.571

1.868 2.142 2.387 2.605 2.797

5.514 6.773 7.947 9.021 9.987

6 7 8 9 10

11 12 13 14 15

11.642 14.552 18.190 22.737 28.422

0.0859 0.0687 0.0550 0.0440 0.0352

0.0235 0.0184 0.0145 0.0115 0.0091

0.2735 0.2684 0.2645 0.2615 0.2591

42.566 54.208 68.760 86.949 109.687

3.656 3.725 3.780 3.824 3.859

2.966 3.115 3.244 3.356 3.453

10.846 11.602 12.262 12.833 13.326

11 12 13 14 15

16 17 18 19 20

35.527 44.409 55.511 69.389 86.736

0.0281 0.0225 0.0180 0.0144 0.0115

0.0072 0.0058 0.0046 0.0037 0.0029

0.2572 0.2558 0.2546 0.2537 0.2529

138.109 173.636 218.045 273.556 342.945

3.887 3.910 3.928 3.942 3.954

3.537 3.608 3.670 3.722 3.767

13.748 14.108 14.415 14.674 14.893

16 17 18 19 20

21 22 23 24 25

108.420 135.525 169.407 211.758 264.698

0.0092 0.0074 0.0059 0.0047 0.0038

0.0023 0.0019 0.0015 0.0012 0.0009

0.2523 0.2519 0.2515 0.2512 0.2509

429.681 538.101 673.626 843.033 1054.791

3.963 3.970 3.976 3.981 3.985

3.805 3.836 3.863 3.886 3.905

15.078 15.233 15.362 15.471 15.562

21 22 23 24 25

26 27 28 29 30

330.872 413.590 516.988 646.235 807.794

0.0030 0.0024 0.0019 0.0015 0.0012

0.0008 0.0006 0.0005 0.0004 0.0003

0.2508 0.2506 0.2505 0.2504 0.2503

1319.489 1650.361 2063.952 2580.939 3227.174

3.988 3.990 3.992 3.994 3.995

3.921 3.935 3.946 3.955 3.963

15.637 15.700 15.752 15.796 15.832

26 27 28 29 30

31 32 33 34 35

1009.742 1262.177 1577.722 1972.152 2465.190

0.0010 0.0008 0.0006 0.0005 0.0004

0.0002 0.0002 0.0002 0.0001 0.0001

0.2502 0.2502 0.2502 0.2501 0.2501

4034.968 5044.710 6306.887 7884.609 9856.761

3.996 3.997 3.997 3.998 3.998

3.969 3.975 3.979 3.983 3.986

15.861 15.886 15.906 15.923 15.937

31 32 33 34 35

36 37 38 39 40

3081.488 3851.860 4814.825 6018.531 7523.164

0.0003 0.0003 0.0002 0.0002 0.0001

0.0001 0.0001 0.0001 0.0000 0.0000

0.2501 0.2501 0.2501 0.2500 0.2500

12321.952 15403.440 19255.299 24070.124 30088.655

3.999 3.999 3.999 3.999 3.999

3.988 3.990 3.992 3.994 3.995

15.948 15.957 15.965 15.971 15.977

36 37 38 39 40

41 42 43 44 45

9403.955 11754.944 14693.679 18367.099 22958.874

0.0001 0.0001 0.0001 0.0001 0.0000

0.0000 0.0000 0.0000 0.0000 0.0000

0.2500 0.2500 0.2500 0.2500 0.2500

37611.819 47015.774 58770.718 73464.397 91831.496

4.000 4.000 4.000 4.000 4.000

3.996 3.996 3.997 3.998 3.998

15.981 15.984 15.987 15.990 15.991

41 42 43 44 45

46 47 48 49 50

28698.593 35873.241 44841.551 56051.939 70064.923

0.0000 0.0000 0.0000 0.0000 0.0000

0.0000 0.0000 0.0000 0.0000 0.0000

0.2500 0.2500 0.2500 0.2500 0.2500

114790.370 143488.963 179362.203 224203.754 280255.693

4.000 4.000 4.000 4.000 4.000

3.998 3.999 3.999 3.999 3.999

15.993 15.994 15.995 15.996 15.997

46 47 48 49 50

7477_book.fm Page 281 Tuesday, March 13, 2007 3:34 PM

Appendix E:

Interest factors and tables

30% Period

n 1 2 3 4 5

281

Compound Interest Factors Single Payment Uniform Payment Series Compound Present Sinking Capital Compound Amount Value Fund Recovery Amount Factor Factor Factor Factor Factor Find F Find P Find A Find A Find F Given P Given F Given F Given P Given A F/P P/F A/F A/P F/A 1.300 0.7692 1.0000 1.3000 1.000 1.690 0.5917 0.4348 0.7348 2.300 2.197 0.4552 0.2506 0.5506 3.990 2.856 0.3501 0.1616 0.4616 6.187 3.713 0.2693 0.1106 0.4106 9.043

30%

Arithmetic Gradient Period Present Gradient Gradient Value Uniform Present Factor Series Value Find P Find A Find P Given A Given G Given G P/A A/G P/G n 0.769 0.000 0.000 1 1.361 0.435 0.592 2 1.816 0.827 1.502 3 2.166 1.178 2.552 4 2.436 1.490 3.630 5

6 7 8 9 10

4.827 6.275 8.157 10.604 13.786

0.2072 0.1594 0.1226 0.0943 0.0725

0.0784 0.0569 0.0419 0.0312 0.0235

0.3784 0.3569 0.3419 0.3312 0.3235

12.756 17.583 23.858 32.015 42.619

2.643 2.802 2.925 3.019 3.092

1.765 2.006 2.216 2.396 2.551

4.666 5.622 6.480 7.234 7.887

6 7 8 9 10

11 12 13 14 15

17.922 23.298 30.288 39.374 51.186

0.0558 0.0429 0.0330 0.0254 0.0195

0.0177 0.0135 0.0102 0.0078 0.0060

0.3177 0.3135 0.3102 0.3078 0.3060

56.405 74.327 97.625 127.913 167.286

3.147 3.190 3.223 3.249 3.268

2.683 2.795 2.889 2.969 3.034

8.445 8.917 9.314 9.644 9.917

11 12 13 14 15

16 17 18 19 20

66.542 86.504 112.455 146.192 190.050

0.0150 0.0116 0.0089 0.0068 0.0053

0.0046 0.0035 0.0027 0.0021 0.0016

0.3046 0.3035 0.3027 0.3021 0.3016

218.472 285.014 371.518 483.973 630.165

3.283 3.295 3.304 3.311 3.316

3.089 3.135 3.172 3.202 3.228

10.143 10.328 10.479 10.602 10.702

16 17 18 19 20

21 22 23 24 25

247.065 321.184 417.539 542.801 705.641

0.0040 0.0031 0.0024 0.0018 0.0014

0.0012 0.0009 0.0007 0.0006 0.0004

0.3012 0.3009 0.3007 0.3006 0.3004

820.215 1067.280 1388.464 1806.003 2348.803

3.320 3.323 3.325 3.327 3.329

3.248 3.265 3.278 3.289 3.298

10.783 10.848 10.901 10.943 10.977

21 22 23 24 25

26 27 28 29 30

917.333 1192.533 1550.293 2015.381 2619.996

0.0011 0.0008 0.0006 0.0005 0.0004

0.0003 0.0003 0.0002 0.0001 0.0001

0.3003 0.3003 0.3002 0.3001 0.3001

3054.444 3971.778 5164.311 6714.604 8729.985

3.330 3.331 3.331 3.332 3.332

3.305 3.311 3.315 3.319 3.322

11.005 11.026 11.044 11.058 11.069

26 27 28 29 30

31 32 33 34 35

3405.994 4427.793 5756.130 7482.970 9727.860

0.0003 0.0002 0.0002 0.0001 0.0001

0.0001 0.0001 0.0001 0.0000 0.0000

0.3001 0.3001 0.3001 0.3000 0.3000

11349.981 14755.975 19183.768 24939.899 32422.868

3.332 3.333 3.333 3.333 3.333

3.324 3.326 3.328 3.329 3.330

11.078 11.085 11.090 11.094 11.098

31 32 33 34 35

36 37 38 39 40

12646.219 16440.084 21372.109 27783.742 36118.865

0.0001 0.0001 0.0000 0.0000 0.0000

0.0000 0.0000 0.0000 0.0000 0.0000

0.3000 0.3000 0.3000 0.3000 0.3000

42150.729 54796.947 71237.031 92609.141 120392.883

3.333 3.333 3.333 3.333 3.333

3.330 3.331 3.332 3.332 3.332

11.101 11.103 11.105 11.106 11.107

36 37 38 39 40

41 42 43 44 45

46954.524 61040.882 79353.146 103159.090 134106.817

0.0000 0.0000 0.0000 0.0000 0.0000

0.0000 0.0000 0.0000 0.0000 0.0000

0.3000 0.3000 0.3000 0.3000 0.3000

156511.748 203466.272 264507.153 343860.299 447019.389

3.333 3.333 3.333 3.333 3.333

3.332 3.333 3.333 3.333 3.333

11.108 11.109 11.109 11.110 11.110

41 42 43 44 45

46 47 48 49 50

174338.862 226640.520 294632.676 383022.479 497929.223

0.0000 0.0000 0.0000 0.0000 0.0000

0.0000 0.0000 0.0000 0.0000 0.0000

0.3000 0.3000 0.3000 0.3000 0.3000

581126.206 755465.067 982105.588 1276738.264 1659760.743

3.333 3.333 3.333 3.333 3.333

3.333 3.333 3.333 3.333 3.333

11.110 11.110 11.111 11.111 11.111

46 47 48 49 50

7477_book.fm Page 282 Tuesday, March 13, 2007 3:34 PM

7477_Idx.fm Page 283 Friday, March 30, 2007 2:31 PM

Index A ABC, 26 Activity-based costing, 26 Advanced cash-flow analysis, 135 After-tax cash flow, 130, 132 After-tax computations, 132 AHP, 170 Alternative depreciation system, 107 Amortization of capitals, 135 Analytic hierarchy process, 170 Applied economic analysis, 1 Arithmetic gradient series, 47 Asset life, 87 Asset replacement analysis, 85 Assignment problem, 4

B Before-tax computations, 132 Benchmarking, 175 Benefit/cost ratio analysis, 195 Benefit/cost ratio, 52 Bonds, 25 Book depreciation, 100 Book value, 100 Bottom-up budgeting, 179 Break-even analysis, 113 Break-even point, 113 Budgeting, 179

C C/SCSC, 22 Capital allocation, 179 Capital gain, 132 Capital rationing, 180 Capital, sources of, 24 Capitalized cost, 45

Case Study, cost benchmarking, 205 Cash-flow analysis, 190 Cash-flow patterns, 39 Challenger, 86 Combined interest rate, 123 Commodity escalation, 123, 124 Comparison of alternatives, 61, 76 Compound amount factor, 41 Compound interest rate, 34 Computational formulas, 245 Constant-worth cash flow, 124 Consumer price index, 123 Conversion factors, 237 Cost and schedule control systems criteria, 22 Cost concepts, 17 Cost definitions, 1 Cost estimates, 20 Cost formulas, 27 Cost monitoring, 21 Cost performance index, 2 Cost performance index, 21 Cost variance, 2 Cost-driven inflation, 125 Cost-push inflation, 125 Cost-time-productivity formulas, 27 CPI, 2 Cumulative cost performance index, 3 Cumulative CPI, 2

D Davis Bacon Wage incentives, 214 Declining balance method, 102 Defender, 86 Definitions, 221 Demand-driven inflation, 126 Demand-pull inflation, 126 Depreciation analysis, 193 Depreciation, methods, 100 Depreciation, terminology, 99

283

7477_Idx.fm Page 284 Friday, March 30, 2007 2:31 PM

284

Computational Economic Analysis for Engineering and Industry

Descartes’ rule of signs, 71 Detailed cost estimates, 19 Direct cost, 17 Discounted payback period, 55 Do-nothing option, 61

E EAC, 3 Earned value analysis, 2 Earned value, 24 Economic analysis, 1 Economic analysis, 33 Economic service life, 87 Economics, 1 Economies of scale, 17 Effective interest rate, 36 ENGINEA software, 189 Engineering economic analysis, 1 Equity break-even point, 135, 139 Equivalence, 39 ESL, 87 Estimate at completion, 3 External rate of return analysis, 74

Incremental cost, 18 Indirect cost, 18 Industrial economics, 1 Industrial enterprises, 1 Industrial operations, 6 Inflation, 123 Interest calculator, 194 Interest factors, 253 Interest rates, 34 Interest rates, variable, 57 Interest tables, 256 Internal rate of return analysis, 70 Internal rate of return, 52 Investment benchmarking, 175 Investment life, 36 Investment selection, 151 Investment value model, 161 Investment, foreign, 25

L Life-cycle cost, 18 Loan analysis, 196 Loans, commercial, 25

F

M

First cost, 18, 87 Fixed cost, 18 Flow-through method, 8 Foreign-exchange rates, 129 Formulas, computational, 245 Funds, mutual, 26

MACRS, 106 Maintenance cost, 18 Marginal cost, 18 Marginal cost, 87 Market value, 100 MARR (Minimum Annual Revenue Requirement Analysis), 7 Minimum annual revenue requirement analysis, 7 Modified accelerated cost recovery, 106 Mortgage analysis, 196 Multi-attribute investment analysis, 151 Mutual funds, 26

G General depreciation system, 107 Geometric cash flow, 49 Gradient cash flow, 47 Guidelines for comparison of alternatives, 76

H Half-year convention, 100 Hard costs, 206 Hungarian method, 5 Hyperinflation, 127

I Impact of PLA, 214 Incremental analysis, 75

N Net present value analysis, 61 Nominal interest rate, 36 Normalizing method, 8

O Operating cost, 18 Opportunity cost, 18 Optimistic estimates, 20 Order-of-magnitude estimates, 19

7477_Idx.fm Page 285 Friday, March 30, 2007 2:31 PM

Index Outsider viewpoint, 87 Overhead cost, 18

P Payback period, 54 Periodic earned values, 2 Permanent investment, 46 Personal property, 110 Pessimistic estimates, 20 PLA (Project Labor Agreement), 211 Polar plots, 163 Preliminary cost estimates, 19 Present value analysis, 61 Present value, 4 Producer price index, 123 Productivity formulas, 27 Profit ratio analysis, 117 Project balance technique, 21 Project cost estimation, 19 Project Labor Agreement (PLA), 211 Property classes, 110

R Real interest rate, 123, 124 Real property, 110 Recovery period, 100 Replacement analysis computation, 88 Replacement analysis, 86, 190 Resource utilization, 6 Retention analysis, 85

S Salvage value, 100 Schedule performance index, 3 Schedule variance, 2

285 Service life, 87 Simple interest rate, 34 Soft costs, 206 Software, 189 Sports contracts, 142 Standard cost, 19 Stocks, 25 Straight-line method, 101 Sunk cost, 19

T Tax depreciation, 100 Taxes, 123 Taxes, 126 Tent cash-flow analysis, 142 Then-current cash flow, 124 Time formulas, 27 Top-down budgeting, 179 Total cost, 19

U Uniform series, 42 Utility models, 151

V Value definitions, 1 Variable cost, 19

W Wage-driven inflation, 126 Wage-push inflation, 126 Wholesale price index, 123 Worker assignment, 4

7477_Idx.fm Page 286 Friday, March 30, 2007 2:31 PM